[QE-users] Web site issue

2024-05-22 Thread Giovanni Cantele
Hi,

I would like to ask some Quantum ESPRESSO website administrator to help me.
Unfortunately, trying to download QE on a new computer, I misspelled the
passwd maybe too many times (3 or 4). Now I'm locked and I cannot access
the website anymore.

The odd thing is that trying to access the site from the old computer,
having a different IP, i'm blocked too!

I hope some can help me.

Thanks.
Giovanni

-- 

Giovanni Cantele, PhD
CNR-SPIN
c/o Dipartimento di Fisica
Universita' di Napoli "Federico II"
Complesso Universitario M. S. Angelo - Ed. 6
Via Cintia, I-80126, Napoli, Italy
e-mail: giovanni.cant...@spin.cnr.it 
Phone: +39 081 676910
Skype contact: giocan74

ResearcherID: http://www.researcherid.com/rid/A-1951-2009
Web page: https://sites.google.com/view/giovanni-cantele/home
___
The Quantum ESPRESSO community stands by the Ukrainian
people and expresses its concerns about the devastating
effects that the Russian military offensive has on their
country and on the free and peaceful scientific, cultural,
and economic cooperation amongst peoples
___
Quantum ESPRESSO is supported by MaX (www.max-centre.eu)
users mailing list users@lists.quantum-espresso.org
https://lists.quantum-espresso.org/mailman/listinfo/users

Re: [QE-users] how to choose the number of processors (-np with -npool)

2024-05-20 Thread Giovanni Cantele
Dear Bouafia,

you might look for parallelization hints  on the website. The correct use
of the available resources depends on the architecture of your computer and
on the size of the calculation.

You should first understand if the result you find in terms of
computational time are due to the fact that the size of your system is too
small (I would not use 18 processors for bulk silicon, for example)
or to the fact that the two nodes are not efficient in terms of inter-node
communication.

This being said, if 18 processors are a good starting point, you might try
with 36 with -npool 2. This will partition your 36 processors into two
groups of 18 processors, each dealing with one half of the k-points.
This will work only if your calculation comprises at least two k-points.
Giovanni

-- 

Giovanni Cantele, PhD
CNR-SPIN
c/o Dipartimento di Fisica
Universita' di Napoli "Federico II"
Complesso Universitario M. S. Angelo - Ed. 6
Via Cintia, I-80126, Napoli, Italy
e-mail: giovanni.cant...@spin.cnr.it 
Phone: +39 081 676910
Skype contact: giocan74

ResearcherID: http://www.researcherid.com/rid/A-1951-2009
Web page: https://sites.google.com/view/giovanni-cantele/home


Il giorno lun 20 mag 2024 alle ore 13:26 H. BOUAFIA <
hamza_boua...@hotmail.com> ha scritto:

> Hi
> For two nodes of 18 processors each, how to choose the number of
> processors (-np with -npool) because the calculation with a single node
> (-np 18) is much faster than that with two nodes (-np 36).
> Sincerely
> H Bouafia
> ___
> The Quantum ESPRESSO community stands by the Ukrainian
> people and expresses its concerns about the devastating
> effects that the Russian military offensive has on their
> country and on the free and peaceful scientific, cultural,
> and economic cooperation amongst peoples
> ___
> Quantum ESPRESSO is supported by MaX (www.max-centre.eu)
> users mailing list users@lists.quantum-espresso.org
> https://lists.quantum-espresso.org/mailman/listinfo/users
>
___
The Quantum ESPRESSO community stands by the Ukrainian
people and expresses its concerns about the devastating
effects that the Russian military offensive has on their
country and on the free and peaceful scientific, cultural,
and economic cooperation amongst peoples
___
Quantum ESPRESSO is supported by MaX (www.max-centre.eu)
users mailing list users@lists.quantum-espresso.org
https://lists.quantum-espresso.org/mailman/listinfo/users

Re: [QE-users] N2 unit cell

2024-05-20 Thread Giovanni Cantele
Dear Vishva,

I'm maybe misunderstanding your question, but are you looking for the
properties of a N2 molecule?
You're speaking about the magnetization of N2 that indeed should turn out
to be zero. However your input file contains 20 atoms.
If you try to visualize your structure, that is always a good practice
before running any calculation,
it appears like a small N20 cluster, where some N atoms seem to have just
two bonds, other N atoms three bonds. Moreover, the size of the cubic
cell seems to be sufficiently small to make those clusters at list
partially interacting with each other. If this is the case, no warranty that
the system you're simulating is not magnetic.

N2 molecule is correctly reproduced (with its zero magnetization and its
1.10 bond length) by this file

calculation = 'relax'
pseudo_dir =  './'
outdir =  './tmp'
 forc_conv_thr = 1.0d-3
 etot_conv_thr = 1.0d-4
/


  ibrav = 1
  A=  10
  nat = 2
  ntyp = 1
  nspin=2
  starting_magnetization(1)=1.0
  degauss =  1.0e-02
  ecutrho =  450
  ecutwfc =  50
  occupations = "smearing"
  smearing= "gaussian"
/


conv_thr =  1.0e-06
electron_maxstep = 500
mixing_beta  =  7.0e-01
startingpot  = "atomic"
startingwfc  = "atomic+random"
/

/


ATOMIC_SPECIES
N 14.0067 N.pbe-n-rrkjus_psl.1.0.0.UPF

ATOMIC_POSITIONS {angstrom}
 N0.0  0.0  0.0
 N    1.5  0.0  0.0

K_POINTS {gamma}


--
Giovanni Cantele, PhD
CNR-SPIN
c/o Dipartimento di Fisica
Universita' di Napoli "Federico II"
Complesso Universitario M. S. Angelo - Ed. 6
Via Cintia, I-80126, Napoli, Italy
e-mail: giovanni.cant...@spin.cnr.it 
Phone: +39 081 676910
Skype contact: giocan74

ResearcherID: http://www.researcherid.com/rid/A-1951-2009
Web page: https://sites.google.com/view/giovanni-cantele/home


Il giorno lun 20 mag 2024 alle ore 08:24 VISHVA JEET ANAND via users <
users@lists.quantum-espresso.org> ha scritto:

> Dear users
> I have two quick queries, first can we relax
> N2 (diatom) unit cell in QE and second is I attached here my input and
> output file of the N2 unit cell please check these files. In the
> magnetization calculation I got 10.51 bohr /unit cell but N2 should have
> 0.0 magnetization.
>
> --
> With Regards
> Vishva Jeet Anand
> Research Scholar
> Department of Chemistry
>
> ___
> The Quantum ESPRESSO community stands by the Ukrainian
> people and expresses its concerns about the devastating
> effects that the Russian military offensive has on their
> country and on the free and peaceful scientific, cultural,
> and economic cooperation amongst peoples
> ___
> Quantum ESPRESSO is supported by MaX (www.max-centre.eu)
> users mailing list users@lists.quantum-espresso.org
> https://lists.quantum-espresso.org/mailman/listinfo/users
___
The Quantum ESPRESSO community stands by the Ukrainian
people and expresses its concerns about the devastating
effects that the Russian military offensive has on their
country and on the free and peaceful scientific, cultural,
and economic cooperation amongst peoples
___
Quantum ESPRESSO is supported by MaX (www.max-centre.eu)
users mailing list users@lists.quantum-espresso.org
https://lists.quantum-espresso.org/mailman/listinfo/users

Re: [QE-users] Feo unit sell relax

2024-05-15 Thread Giovanni Cantele
You're selecting, through ibrav = 2,  cubic F (fcc) lattice with lattice
constant a = 4.3988580704 A.
As the error, quite clearly shows, is with atoms 1 and 2 (of course similar
errors may show up for the other atoms),
let us consider the positions in input:
Fe0.000.000.00
Fe0.002.1994292.199429 -> 0 a/2 a/2

So, atoms 1 and 2 differ by the vector (0,a/2,a/2) that is a Bravais
lattice translation of cubic fcc. This means that in the input there are
atoms that are equivalent for periodicity.

What you should do depends on what you would do. Either you're adding atoms
that are not needed in the input or you are trying to build a supercell
and you have forgotten to change the lattice parameter accordingly.

The number of atoms depends on which system you want to simulate.

Giovanni

-- 

Giovanni Cantele, PhD
CNR-SPIN
c/o Dipartimento di Fisica
Universita' di Napoli "Federico II"
Complesso Universitario M. S. Angelo - Ed. 6
Via Cintia, I-80126, Napoli, Italy
e-mail: giovanni.cant...@spin.cnr.it 
Phone: +39 081 676910
Skype contact: giocan74

ResearcherID: http://www.researcherid.com/rid/A-1951-2009
Web page: https://sites.google.com/view/giovanni-cantele/home


Il giorno mer 15 mag 2024 alle ore 13:28 VISHVA JEET ANAND via users <
users@lists.quantum-espresso.org> ha scritto:

> Dear Users
> I try to relax the FeO unit cell but I face a problem that is attached
> here in the output file and I also attached the input file.
> In the input file are no. of atoms right with ibrav value? In input i am
> using angstrom atomic position exactly at the same position as crystal.
> Please help to resolve this problem.
>
> --
> With Regards
> Vishva Jeet Anand
> Research Scholar
> Department of Chemistry
>
> ___
> The Quantum ESPRESSO community stands by the Ukrainian
> people and expresses its concerns about the devastating
> effects that the Russian military offensive has on their
> country and on the free and peaceful scientific, cultural,
> and economic cooperation amongst peoples
> ___
> Quantum ESPRESSO is supported by MaX (www.max-centre.eu)
> users mailing list users@lists.quantum-espresso.org
> https://lists.quantum-espresso.org/mailman/listinfo/users
___
The Quantum ESPRESSO community stands by the Ukrainian
people and expresses its concerns about the devastating
effects that the Russian military offensive has on their
country and on the free and peaceful scientific, cultural,
and economic cooperation amongst peoples
___
Quantum ESPRESSO is supported by MaX (www.max-centre.eu)
users mailing list users@lists.quantum-espresso.org
https://lists.quantum-espresso.org/mailman/listinfo/users

Re: [QE-users] Supercell relaxation

2024-05-13 Thread Giovanni Cantele
Dear Vishva,

before running any calculation it is a good practice to visualize your
input structure, because many times convergence issues derive
from errors in the input geometry.

If you do so with yours, you see atoms at extremely small distances from
each other, which prevents pw.x from reasonably converging
in an acceptable number of steps.

The second step is to understand why the structure is wrong. In your case
my guess is that you tell pw.x that you're providing positions in alat
units,
whereas those units are Angstrom.

The number of atoms depends on what you want to do. If you what to use a
simple cubic unit cell (as in your input file) and the bcc unit cell is
composed by N atoms,
then the 1x1x1 CONVENTIONAL unit cell will contain 2N atoms. If, on top of
that, you want also build a 2x2x2 unit cell, than that number has to be
multiplied by 2x2x2.

Giovanmi
-- 

Giovanni Cantele, PhD
CNR-SPIN
c/o Dipartimento di Fisica
Universita' di Napoli "Federico II"
Complesso Universitario M. S. Angelo - Ed. 6
Via Cintia, I-80126, Napoli, Italy
e-mail: giovanni.cant...@spin.cnr.it 
Phone: +39 081 676910
Skype contact: giocan74

ResearcherID: http://www.researcherid.com/rid/A-1951-2009
Web page: https://sites.google.com/view/giovanni-cantele/home


Il giorno lun 13 mag 2024 alle ore 12:08 VISHVA JEET ANAND via users <
users@lists.quantum-espresso.org> ha scritto:

> Dear Users
> I try to run Fe (bcc) structure 2x2x2 supercell for relaxation
> calculation but scf does not converge in 1000 iterations. Secondally how
> many no. of atoms in Fe (bcc) structure. Here I attached my input file.
>
> --
> With Regards
> Vishva Jeet Anand
> Research Scholar
> Department of Chemistry
>
> ___
> The Quantum ESPRESSO community stands by the Ukrainian
> people and expresses its concerns about the devastating
> effects that the Russian military offensive has on their
> country and on the free and peaceful scientific, cultural,
> and economic cooperation amongst peoples
> ___
> Quantum ESPRESSO is supported by MaX (www.max-centre.eu)
> users mailing list users@lists.quantum-espresso.org
> https://lists.quantum-espresso.org/mailman/listinfo/users
___
The Quantum ESPRESSO community stands by the Ukrainian
people and expresses its concerns about the devastating
effects that the Russian military offensive has on their
country and on the free and peaceful scientific, cultural,
and economic cooperation amongst peoples
___
Quantum ESPRESSO is supported by MaX (www.max-centre.eu)
users mailing list users@lists.quantum-espresso.org
https://lists.quantum-espresso.org/mailman/listinfo/users

Re: [QE-users] Creating a 2 × 2 × 3 Surface of Crystal Structure with 10 Å Vacuum in Burai or XCrysDen

2024-04-08 Thread Giovanni Cantele
Dear Moses,

there is not actually a universal recipe and the means used depends on your
feeling. I'm not an expert about Burai. As far as XCrysDen is concerned, I
use it a lot to visualize the structure, not to build it.

Personally, I've always found it instructive try, to the best of what I can
do, to build structures "by hand", because this is in my opinion the best
way to "enter a crystal" and know how it is done!

Few general hints, whatever tool you use:
1) first identify the direction of the surface, e.g. 100, 110, 111, ...
2) identify the "in-plane" lattice vectors. This can coincide with those of
the bulk lattice or not. For example, should you want to build the 001
surface of a simple cubic crystal, the 1x1 surface will have a1 = (a,0,0)
and a2 = (0,a,0) as surface in-plane vectors
3) as a corollary of point two there are cases where complex
reconstructions (e.g. 2x2, sqrt(3) x sqrt(3), ) might occur. In these
cases, there are the "simplest" systems where you should just take the
in-plane vectors of the 1x1 surface unit cell and double, triple, 
them. The trickiest cases are instead when the in-plane vectors of the
reconstructed surface ar not only scaled but also rotated with respect to
the starting ones.
4) build, according to the chosen a1 and a2, slab models consisting of n
planes, obtained as a periodic repetition of the first plane along the 3d
direction.
5) take the resulting thickness in the correct units, and add the vacuum in
the same units, this will be the length of the 3d vector.

Be careful: sometimes one has coordinates in Ang, other times in 2 pi / a
units, other times in crystal units. It is important that one makes the new
coordinates consistent with the units used for the surface.

Imagine, for example, that you have a simple cubic system with one atom in
the unit cell at (0,0,0) and lattice constant a. A 4-slab model will have a
unit cell with atoms at
(0,0,0)
(0,0,a)
(0,0,2a)
(0,0,3a)
The in-plane lattice vectors will be (a,0,0) and (0,a,0). The lattice
vector orthogonal to the surface will be (0,0,c) where c = 3a + vacuum
space.

Always visualize your new structure before running any calculation!

More specific help could be provided for specific structures.
Giovanni

-- 

Giovanni Cantele, PhD
CNR-SPIN
c/o Dipartimento di Fisica
Universita' di Napoli "Federico II"
Complesso Universitario M. S. Angelo - Ed. 6
Via Cintia, I-80126, Napoli, Italy
e-mail: giovanni.cant...@spin.cnr.it 
Phone: +39 081 676910
Skype contact: giocan74

ResearcherID: http://www.researcherid.com/rid/A-1951-2009
Web page: https://sites.google.com/view/giovanni-cantele/home


Il giorno lun 8 apr 2024 alle ore 17:26 MOSES NTSIFUL <
moses.ntsiful...@stu.ucc.edu.gh> ha scritto:

> Dear All,
>
> I hope this email finds you well. I am writing to request your assistance
> with a specific task related to crystal structure visualization and
> manipulation. I am currently working on a project that requires the
> creation of a 2 × 2 × 3 surface of a crystal structure with a 10 Å vacuum
> using either Burai or XCrysDen. Despite my efforts, I have not been able to
> successfully complete this task.
>
> I was wondering if I could be provided with some guidance on how to
> proceed. Specifically, I would appreciate any insights you might have on:
>
>1.
>
>How to create a 2 × 2 × 3 surface of a crystal structure in Burai or
>XCrysDen.
>2.
>
>How to ensure that the resulting surface has a 10 Å vacuum.
>
> Thank you in advance for the assistance. I look forward to hearing from
> you soon.
>
>
>
> Best regards,
>
> Moses Ntsiful
> ___
> The Quantum ESPRESSO community stands by the Ukrainian
> people and expresses its concerns about the devastating
> effects that the Russian military offensive has on their
> country and on the free and peaceful scientific, cultural,
> and economic cooperation amongst peoples
> ___
> Quantum ESPRESSO is supported by MaX (www.max-centre.eu)
> users mailing list users@lists.quantum-espresso.org
> https://lists.quantum-espresso.org/mailman/listinfo/users
___
The Quantum ESPRESSO community stands by the Ukrainian
people and expresses its concerns about the devastating
effects that the Russian military offensive has on their
country and on the free and peaceful scientific, cultural,
and economic cooperation amongst peoples
___
Quantum ESPRESSO is supported by MaX (www.max-centre.eu)
users mailing list users@lists.quantum-espresso.org
https://lists.quantum-espresso.org/mailman/listinfo/users

Re: [QE-users] Convergence not achieved

2024-03-18 Thread Giovanni Cantele
Dear Saiyed,
it is better if next time you also include output, just to understand what
is not converging.

In this case:
i) for convergence issues of the electronic self-consistent cycle, you
should always try to reduce mixing_beta to 0.3, 0.1, 0.05 ...

ii) before running ANY calculations, mostly if you're dealing with a
material that is novel for you, it is quite a good practice to visualize
your structure.
I'm not expert about this material, however what I can notice is that, if
you're running the same calculation as that you described in your first
email,
that is, 3 layers of calcite, I cannot see three layers but just one and
the one I see I suspect does not resemble that much to the models in the
paper you mention.
You could figure out that you're likely not simulating three layers, by
looking at your coordinates: the z coordinates range from 0.46234
to 0.53766, so they span an
interval 0.07532 wide. Because these are crystal units, this in Angstrom is
0.07532 *  43.2578 Ang ~ 3.258 Ang. It is quite unlikely that three
layers of any material
(even graphene. the flattest you can think about) are just 3 Ang thick.

Giovanni
-- 

Giovanni Cantele, PhD
CNR-SPIN
c/o Dipartimento di Fisica
Universita' di Napoli "Federico II"
Complesso Universitario M. S. Angelo - Ed. 6
Via Cintia, I-80126, Napoli, Italy
e-mail: giovanni.cant...@spin.cnr.it 
Phone: +39 081 676910
Skype contact: giocan74

ResearcherID: http://www.researcherid.com/rid/A-1951-2009
Web page: https://sites.google.com/view/giovanni-cantele/home


Il giorno sab 16 mar 2024 alle ore 22:45 Saiyed Tasnim Md Fahim <
sfa...@usc.edu> ha scritto:

> Dear Dr. Cantele,
>
> Thank you for your response.
> I tried the steps that you mentioned. However, it still didn't converge.
> I am following this paper as the benchmark :
> https://doi.org/10.1016/j.cej.2024.148940
> I have used the norm conserving pseudopotentials from Pseudodojo website
> as USPP/PAW wasn't implemented for TS dispersion. I used the ASE (atomic
> simulation environment) to generate the atomic positions of the 2x2
> supercell.
>
> I have attached the new input file with this mail. Could you please tell
> me if any parameters I have missed or need to be changed or removed? Thank
> you.
>
>
> Sincerely,
> Saiyed Tasnim Md Fahim,
> Ph.D. Student,
> Environmental Engineering,
> University of Southern California.
>
>
> On Wed, Feb 28, 2024 at 12:27 AM Giovanni Cantele <
> giovanni.cant...@spin.cnr.it> wrote:
>
>> Dear Saiyed,
>>
>> when convergence issues show up there are several tricks you might
>> consider, one of these is certainly reducing mixing_beta,
>> another could be increasing degauss.
>>
>> However, before trying any trick to cure a calculation particularly
>> difficult to converge, as a rule of thumb the first step is to
>> check your input file parameters including cut-off, k-point grid,
>> degauss, GEOMETRY, pseudopotentials, etc.
>>
>> After a quick look to your input file I can notice:
>> - an incredibly huge value of degauss (1d-1 Ry = 0.1 Ry = 1.36 eV, this
>> would smear even the band gap of silicon I think)
>> - a mixing beta of 0.6, actually the advise of reducing it in cases of
>> difficult convergence is meant to go down to even 0.1 or 0.05
>> - but MOST IMPORTANTLY you specify ATOMIC_POSITIONS in crystal units,
>> that are usually expected to be numbers in the range
>> [-1,1] because in units of the lattice vectors there should be no reason
>> to specify the position of an atom in a unit cell far apart from the
>> origin. On the other hand, the numbers in your input file seem to
>> indicate that you're actually giving the positions in Angstrom, because
>> they are
>> huge. If you try to visualize your input file using, for example
>> XCrysDen, you can easily see atoms at incredibly huge distances. The
>> geometry
>> becomes much more reasonables if you replace "crystal" with angstrom.
>> After doing that, recheck the geometry (with XCrysDen) to understand if it
>> is
>> what you expect or if there are still issues, before running any
>> calculation.
>>
>> Giovanni
>>
>>
>> --
>>
>> Giovanni Cantele, PhD
>> CNR-SPIN
>> c/o Dipartimento di Fisica
>> Universita' di Napoli "Federico II"
>> Complesso Universitario M. S. Angelo - Ed. 6
>> Via Cintia, I-80126, Napoli, Italy
>> e-mail: giovanni.cant...@spin.cnr.it 
>> Phone: +39 081 676910
>> Skype contact: giocan74
>>
>> ResearcherID: http://www.researcherid.com/rid/A-1951-2009
>> <https://urldefense.com/v3/__http://www.researcherid.com/rid/A-1951-2009__;!!LIr3w8kk_Xxm!uHeu46wuGTYOEVDT0_V9s92ksXMy-0mbaezxExpDSYH

Re: [QE-users] error

2024-03-16 Thread Giovanni Cantele
There is likely an error in your input file, no way to figure out which
without more details.
Please kindly  provide (see post guidelines here:
https://www.quantum-espresso.org/users-forum/ ):
1) name
2) affiliation
3) input file.

Giovanni
-- 

Giovanni Cantele, PhD
CNR-SPIN
c/o Dipartimento di Fisica
Universita' di Napoli "Federico II"
Complesso Universitario M. S. Angelo - Ed. 6
Via Cintia, I-80126, Napoli, Italy
e-mail: giovanni.cant...@spin.cnr.it 
Phone: +39 081 676910
Skype contact: giocan74

ResearcherID: http://www.researcherid.com/rid/A-1951-2009
Web page: https://sites.google.com/view/giovanni-cantele/home


Il giorno sab 16 mar 2024 alle ore 19:52 Muhammad Ishfaq via users <
users@lists.quantum-espresso.org> ha scritto:

>   kindly,help me solve :
>  Error in routine  card_atomic_species (5010):
>  cannot read atomic specie from: ATOMIC_POSITIONS crystal
>
> Also i want to know,my compound is antiferromagnetic,what can be the total
> magnetization?
> ___
> The Quantum ESPRESSO community stands by the Ukrainian
> people and expresses its concerns about the devastating
> effects that the Russian military offensive has on their
> country and on the free and peaceful scientific, cultural,
> and economic cooperation amongst peoples
> ___
> Quantum ESPRESSO is supported by MaX (www.max-centre.eu)
> users mailing list users@lists.quantum-espresso.org
> https://lists.quantum-espresso.org/mailman/listinfo/users
___
The Quantum ESPRESSO community stands by the Ukrainian
people and expresses its concerns about the devastating
effects that the Russian military offensive has on their
country and on the free and peaceful scientific, cultural,
and economic cooperation amongst peoples
___
Quantum ESPRESSO is supported by MaX (www.max-centre.eu)
users mailing list users@lists.quantum-espresso.org
https://lists.quantum-espresso.org/mailman/listinfo/users

Re: [QE-users] Too few bands

2024-03-14 Thread Giovanni Cantele
Some remarks about your input file:

1) the error message means that the number of bands you specify is not
enough to accommodate all the electrons,
usually you do not need to specify nbnd in scf/relax calculations (whereas
it is needed in bands/dos calculations if you want
to span a given rage of unoccupied states). In my experience, it is much
better to leave out all the variables you do not need explicitly.

2) I would use the most recent version of QE

3) wf_collect I think is no longer used

4) you are using PAW pseudopotentials but you are using the default value
of ecutrho = 4 * ecutwfc. While this might be enough in some
cases, in many others values as large als 8*ecutwfc (or even more) might be
needed. Did you check it?

5) as it is, it is correct, but if you need an orthorombic unit cell you
may use the corresponding value of ibrav (with the corresponding parameters)
in place of the CELL_PARAMETERS card. The case in which you want to
optimize the unit cell is of course different

6) extension in pseudopotential file names refer to the versions of the
code used to generate them. Something more about the naming convention
might be found here:
https://pseudopotentials.quantum-espresso.org/home/naming-convention

7) you do not need to specify symmetries in the input file. In your case
you are getting rid of them, instead ( nosym = .TRUE., noinv = .TRUE.).
Check if this is exactly what you want, because using symmetries (if any)
might (even significantly) reduce the workload of your calculation

8) in order to better control and quantify the convergence of your
calculations, you might want to sample each direction in reciprocal space
with a similar
way (that means that the distance between two neighbor k-points located
along the three reciprocal vector directions is as much similar as
possible).
For this reason, a better choice than an n x n x n grid is an n x m x l
grid, where the three integers are in the inverse ratios of the
corresponding lattice parameters
in real space. Example: a = 8.4, n = 8 => b = 11.1, m = 8 * 8.4/11 ~ 6

Giovanni

-- 

Giovanni Cantele, PhD
CNR-SPIN
c/o Dipartimento di Fisica
Universita' di Napoli "Federico II"
Complesso Universitario M. S. Angelo - Ed. 6
Via Cintia, I-80126, Napoli, Italy
e-mail: giovanni.cant...@spin.cnr.it 
Phone: +39 081 676910
Skype contact: giocan74

ResearcherID: http://www.researcherid.com/rid/A-1951-2009
Web page: https://sites.google.com/view/giovanni-cantele/home


Il giorno gio 14 mar 2024 alle ore 13:31 aleksandr.doma--- via users <
users@lists.quantum-espresso.org> ha scritto:

> Hello!
>
> I am new in QE. After running a simple SCF calculation on the NCI cluster,
> I got an error message in the output file:
>
>  *Program PWSCF v.6.6 starts on 14Mar2024 at 23:16:11 *
>
> * This program is part of the open-source Quantum ESPRESSO suite*
> * for quantum simulation of materials; please cite*
> * "P. Giannozzi et al., J. Phys.:Condens. Matter 21 395502 (2009);*
> * "P. Giannozzi et al., J. Phys.:Condens. Matter 29 465901 (2017);*
> *  URL http://www.quantum-espresso.org
> <http://www.quantum-espresso.org>", *
> * in publications or presentations arising from this work. More
> details at*
> * http://www.quantum-espresso.org/quote
> <http://www.quantum-espresso.org/quote>*
>
> * Parallel version (MPI), running on 1 processors*
>
> * MPI processes distributed on 1 nodes*
> * Fft bands division: nmany =   1*
> * Waiting for input...*
> * Reading input from standard input*
>
> * Current dimensions of program PWSCF are:*
> * Max number of different atomic species (ntypx) = 10*
> * Max number of k-points (npk) =  4*
> * Max angular momentum in pseudopotentials (lmaxx) =  3*
> * file K.pbe-spn-kjpaw_psl.1.0.0.UPF: wavefunction(s)  3S renormalized*
> * file O.pbe-n-kjpaw_psl.1.0.0.UPF: wavefunction(s)  2S 2P
> renormalized*
>
>
> * 
> %%*
> * Error in routine setup (1):*
> * too few bands*
>
> * 
> %%*
>
> * stopping ...*
>
> *--*
>
> This is my input file:
>
> **
> *prefix = 'lkbo',*
> *pseudo_dir='./'*
> *outdir = './', wf_collect = .true.*
> * /*
> * *
> *ibrav=  0, nat=  104, ntyp= 4,*
> *ecutwfc = 33.0, nbnd = 224, *
> *nosym = .TRUE., noinv = .TRUE.*
> *occupations = 'tetrahedra'*
> * /*
> * *
> * /*
> *CELL_PARAMETERS angstrom*
> *8.4915 0. 0.*
> *0. 11.1415000 0.*
> *0. 0. 12.6558000*

Re: [QE-users] Convergence not achieved

2024-02-28 Thread Giovanni Cantele
Dear Saiyed,

when convergence issues show up there are several tricks you might
consider, one of these is certainly reducing mixing_beta,
another could be increasing degauss.

However, before trying any trick to cure a calculation particularly
difficult to converge, as a rule of thumb the first step is to
check your input file parameters including cut-off, k-point grid, degauss,
GEOMETRY, pseudopotentials, etc.

After a quick look to your input file I can notice:
- an incredibly huge value of degauss (1d-1 Ry = 0.1 Ry = 1.36 eV, this
would smear even the band gap of silicon I think)
- a mixing beta of 0.6, actually the advise of reducing it in cases of
difficult convergence is meant to go down to even 0.1 or 0.05
- but MOST IMPORTANTLY you specify ATOMIC_POSITIONS in crystal units, that
are usually expected to be numbers in the range
[-1,1] because in units of the lattice vectors there should be no reason to
specify the position of an atom in a unit cell far apart from the
origin. On the other hand, the numbers in your input file seem to indicate
that you're actually giving the positions in Angstrom, because they are
huge. If you try to visualize your input file using, for example XCrysDen,
you can easily see atoms at incredibly huge distances. The geometry
becomes much more reasonables if you replace "crystal" with angstrom. After
doing that, recheck the geometry (with XCrysDen) to understand if it is
what you expect or if there are still issues, before running any
calculation.

Giovanni


-- 

Giovanni Cantele, PhD
CNR-SPIN
c/o Dipartimento di Fisica
Universita' di Napoli "Federico II"
Complesso Universitario M. S. Angelo - Ed. 6
Via Cintia, I-80126, Napoli, Italy
e-mail: giovanni.cant...@spin.cnr.it 
Phone: +39 081 676910
Skype contact: giocan74

ResearcherID: http://www.researcherid.com/rid/A-1951-2009
Web page: https://sites.google.com/view/giovanni-cantele/home


Il giorno mer 28 feb 2024 alle ore 08:41 Saiyed Tasnim Md Fahim <
sfa...@usc.edu> ha scritto:

> Dear users,
> I am working with calcite in Quantum Espresso 7.1. I want to perform a
> vc-relax calculation for 3 layers of calcite but after 100 iterations
> convergence is not achieved. I changed the mixing beta but it did not work.
>
> I have attached the input file with this mail. Kindly let me know how I
> should proceed for both the convergence.
>
> Sincerely,
> Saiyed Tasnim Md Fahim,
> Ph.D. Student,
> Environmental Engineering,
> University of Southern California.
> ___
> The Quantum ESPRESSO community stands by the Ukrainian
> people and expresses its concerns about the devastating
> effects that the Russian military offensive has on their
> country and on the free and peaceful scientific, cultural,
> and economic cooperation amongst peoples
> ___
> Quantum ESPRESSO is supported by MaX (www.max-centre.eu)
> users mailing list users@lists.quantum-espresso.org
> https://lists.quantum-espresso.org/mailman/listinfo/users
___
The Quantum ESPRESSO community stands by the Ukrainian
people and expresses its concerns about the devastating
effects that the Russian military offensive has on their
country and on the free and peaceful scientific, cultural,
and economic cooperation amongst peoples
___
Quantum ESPRESSO is supported by MaX (www.max-centre.eu)
users mailing list users@lists.quantum-espresso.org
https://lists.quantum-espresso.org/mailman/listinfo/users

Re: [QE-users] Simple cubic to fcc cubic

2024-02-27 Thread Giovanni Cantele
I would add, to support this suggestion, that this forum is, in my opinion,
a very precious resource, for both those who are beginners and those who
are less beginners
because not only there is a more than reasonable response "rate" to posed
problems and questions but also, browsing past threads, you might discover
lot of
tricks, explanations, suggestions that might result helpful for you!
Giovanni

-- 

Giovanni Cantele, PhD
CNR-SPIN
c/o Dipartimento di Fisica
Universita' di Napoli "Federico II"
Complesso Universitario M. S. Angelo - Ed. 6
Via Cintia, I-80126, Napoli, Italy
e-mail: giovanni.cant...@spin.cnr.it 
Phone: +39 081 676910
Skype contact: giocan74

ResearcherID: http://www.researcherid.com/rid/A-1951-2009
Web page: https://sites.google.com/view/giovanni-cantele/home


Il giorno mar 27 feb 2024 alle ore 11:46 Stefano Baroni 
ha scritto:

> Thank you Giovanni. May I suggest that in this and other similar cases
> help from the Quantum ESPRESSO Community is properly acknowledged in any
> work (thesis, paper, report) that has benefited from it? Thanks — Stefano
>
>
> On 27 Feb 2024, at 11:36, Giovanni Cantele 
> wrote:
>
> Let us suppose that we have a fcc crystal, with one atom per cell, at
> (0,0,0), for the sake of simplicity.
>
> The unit cell sides are in the form (a/2,a/2,0) (a=side of the cubic
> cell), with unit cell volume a^3 / 4.
>
> Now, let us move from fcc to sc unit cell.
> Each of the 8 atoms at the corners is shared by 8 cubic cells, and thus we
> should consider only one of these atoms.
> On the other hand, atoms at the center of the cube faces are shared by two
> unit cells, and count as 1/2. Since we have
> 6 faces, 3 out of these 6 atoms must be considered.
> As a result, the sc unit cell will contain 4 atoms and a volume 4 times
> larger, a^3 as you would expect.
>
> To go back from sc to fcc you should therefore keep 1/4 of your atoms,
> because those that are associated to the centers of three faces,
> that should be explicitly considered in sc, now become equivalent (that
> is, connected by a lattice vector) to other atoms in fcc.
>
> The code correctly finds overlapped atoms, e.g. in the example of one atom
> per unit cell, in sc you would have (0,0,0), (a/2,a/2,0), (a/2,0,a/2),
> (0,a/2,a/2)
> and the last three in fcc are obtained from (0,0,0) after a lattice
> translation by (a/2,a/2,0), (a/2,0,a/2), (0,a/2,a/2), respectively.
>
> Giovanni
>
> --
>
> Giovanni Cantele, PhD
> CNR-SPIN
> c/o Dipartimento di Fisica
> Universita' di Napoli "Federico II"
> Complesso Universitario M. S. Angelo - Ed. 6
> Via Cintia, I-80126, Napoli, Italy
> e-mail: giovanni.cant...@spin.cnr.it 
> Phone: +39 081 676910
> Skype contact: giocan74
>
> ResearcherID: http://www.researcherid.com/rid/A-1951-2009
> Web page: https://sites.google.com/view/giovanni-cantele/home
>
>
> Il giorno mar 27 feb 2024 alle ore 11:14 Ms. Chandrika K. via users <
> users@lists.quantum-espresso.org> ha scritto:
>
>> Need help in understanding
>>
>>1. I am currently working on Co3O4 structure,from literature it is
>>very evident that it belongs to space group Fd3m(cubic fcc).
>>2. I have referred to Co3O4 Fd3m cubic structure from materials
>>project it is represented as simple cubic structure but Fd3m space group
>>refers to cubic fcc.
>>3. As it is simple cubic ibrav =1,when i change ibrav =2(cubic fcc) i
>>am getting warning message
>>"WARNING: Atom 381 and atom 444 are very close !!!   Atom 444 deleted
>>!!!"
>>ATOMIC_POSITIONS (angstrom)
>>
>> I would like to know how to change the structure from simple cubic to fcc
>> cubic
>>
>>
>> chandrika yadav k
>>
>>
>> ___
>> The Quantum ESPRESSO community stands by the Ukrainian
>> people and expresses its concerns about the devastating
>> effects that the Russian military offensive has on their
>> country and on the free and peaceful scientific, cultural,
>> and economic cooperation amongst peoples
>> ___
>> Quantum ESPRESSO is supported by MaX (www.max-centre.eu)
>> users mailing list users@lists.quantum-espresso.org
>> https://lists.quantum-espresso.org/mailman/listinfo/users
>>
> ___
> The Quantum ESPRESSO community stands by the Ukrainian
> people and expresses its concerns about the devastating
> effects that the Russian military offensive has on their
> country and on the free and peaceful scientific, cultural,
> and economic cooperation amongst peoples
> __

Re: [QE-users] Simple cubic to fcc cubic

2024-02-27 Thread Giovanni Cantele
Let us suppose that we have a fcc crystal, with one atom per cell, at
(0,0,0), for the sake of simplicity.

The unit cell sides are in the form (a/2,a/2,0) (a=side of the cubic cell),
with unit cell volume a^3 / 4.

Now, let us move from fcc to sc unit cell.
Each of the 8 atoms at the corners is shared by 8 cubic cells, and thus we
should consider only one of these atoms.
On the other hand, atoms at the center of the cube faces are shared by two
unit cells, and count as 1/2. Since we have
6 faces, 3 out of these 6 atoms must be considered.
As a result, the sc unit cell will contain 4 atoms and a volume 4 times
larger, a^3 as you would expect.

To go back from sc to fcc you should therefore keep 1/4 of your atoms,
because those that are associated to the centers of three faces,
that should be explicitly considered in sc, now become equivalent (that is,
connected by a lattice vector) to other atoms in fcc.

The code correctly finds overlapped atoms, e.g. in the example of one atom
per unit cell, in sc you would have (0,0,0), (a/2,a/2,0), (a/2,0,a/2),
(0,a/2,a/2)
and the last three in fcc are obtained from (0,0,0) after a lattice
translation by (a/2,a/2,0), (a/2,0,a/2), (0,a/2,a/2), respectively.

Giovanni

-- 

Giovanni Cantele, PhD
CNR-SPIN
c/o Dipartimento di Fisica
Universita' di Napoli "Federico II"
Complesso Universitario M. S. Angelo - Ed. 6
Via Cintia, I-80126, Napoli, Italy
e-mail: giovanni.cant...@spin.cnr.it 
Phone: +39 081 676910
Skype contact: giocan74

ResearcherID: http://www.researcherid.com/rid/A-1951-2009
Web page: https://sites.google.com/view/giovanni-cantele/home


Il giorno mar 27 feb 2024 alle ore 11:14 Ms. Chandrika K. via users <
users@lists.quantum-espresso.org> ha scritto:

> Need help in understanding
>
>1. I am currently working on Co3O4 structure,from literature it is
>very evident that it belongs to space group Fd3m(cubic fcc).
>2. I have referred to Co3O4 Fd3m cubic structure from materials
>project it is represented as simple cubic structure but Fd3m space group
>refers to cubic fcc.
>3. As it is simple cubic ibrav =1,when i change ibrav =2(cubic fcc) i
>am getting warning message
>"WARNING: Atom 381 and atom 444 are very close !!!   Atom 444 deleted
>!!!"
>ATOMIC_POSITIONS (angstrom)
>
> I would like to know how to change the structure from simple cubic to fcc
> cubic
>
>
> chandrika yadav k
>
>
> ___
> The Quantum ESPRESSO community stands by the Ukrainian
> people and expresses its concerns about the devastating
> effects that the Russian military offensive has on their
> country and on the free and peaceful scientific, cultural,
> and economic cooperation amongst peoples
> ___
> Quantum ESPRESSO is supported by MaX (www.max-centre.eu)
> users mailing list users@lists.quantum-espresso.org
> https://lists.quantum-espresso.org/mailman/listinfo/users
>
___
The Quantum ESPRESSO community stands by the Ukrainian
people and expresses its concerns about the devastating
effects that the Russian military offensive has on their
country and on the free and peaceful scientific, cultural,
and economic cooperation amongst peoples
___
Quantum ESPRESSO is supported by MaX (www.max-centre.eu)
users mailing list users@lists.quantum-espresso.org
https://lists.quantum-espresso.org/mailman/listinfo/users

Re: [QE-users] Too few bands

2024-02-16 Thread Giovanni Cantele
Hi,

it could be worth signing messages with name and affiliation.

There are many checks in QE at runtime, that print quite self-explanatory
errors, the one you mention is an example, it seems.

The reason why pw.x seems to complain is that you're trying to parallelize
the code using -ndiag, but the number of bands to be computed is
small and so this parallelization cannot be done.
Remove the -ndiag or make it to be followed by 1 and it should work.

I suggest to read documentation and follow the tutorials, before using
features you're not yet aware of.
Giovanni

-- 

Giovanni Cantele, PhD
CNR-SPIN
c/o Dipartimento di Fisica
Universita' di Napoli "Federico II"
Complesso Universitario M. S. Angelo - Ed. 6
Via Cintia, I-80126, Napoli, Italy
e-mail: giovanni.cant...@spin.cnr.it 
Phone: +39 081 676910
Skype contact: giocan74

ResearcherID: http://www.researcherid.com/rid/A-1951-2009
Web page: https://sites.google.com/view/giovanni-cantele/home


Il giorno ven 16 feb 2024 alle ore 13:00 wangzongyi via users <
users@lists.quantum-espresso.org> ha scritto:

> Dear all
> Thank you for your help, after revise my scf.in file, I am facing another
> problem, the program told me that I have too few bands:
> Error in routine check_para_diag (4):
>  Too few bands for required ndiag
> what should I do.
> This is my Si.scf.in file
> 
> calculation='scf',
> restart_mode='from_scratch',
> prefix='si',
> pseudo_dir='./'
> outdir='../tmp/',
> /
> 
> ibrav=2,
> celldm(1)=10.2625,
> nat=2,
> ntyp=1,
> ecutwfc=60.0,
> ecutrho=720.0,
> /
> 
> mixing_beta=0.7,
> conv_thr=1d-8
> /
> ATOMIC_SPECIES
> Si 28.0855 Si.pbe-n-rrkjus_psl.1.0.0.UPF
> ATOMIC_POSITIONS alat
> Si  0.00  0.00  0.00
> Si  0.25  0.25  0.25
> K_POINTS automatic
> 4 4 4 1 1 1
>
> Thank you for your help
> ___
> The Quantum ESPRESSO community stands by the Ukrainian
> people and expresses its concerns about the devastating
> effects that the Russian military offensive has on their
> country and on the free and peaceful scientific, cultural,
> and economic cooperation amongst peoples
> ___
> Quantum ESPRESSO is supported by MaX (www.max-centre.eu)
> users mailing list users@lists.quantum-espresso.org
> https://lists.quantum-espresso.org/mailman/listinfo/users
___
The Quantum ESPRESSO community stands by the Ukrainian
people and expresses its concerns about the devastating
effects that the Russian military offensive has on their
country and on the free and peaceful scientific, cultural,
and economic cooperation amongst peoples
___
Quantum ESPRESSO is supported by MaX (www.max-centre.eu)
users mailing list users@lists.quantum-espresso.org
https://lists.quantum-espresso.org/mailman/listinfo/users

Re: [QE-users] Fwd: under estimate the bandgap in Quantum espresso

2024-02-16 Thread Giovanni Cantele
Dear
Prasad, there are not sufficient data to understand what's happening.

1) DFT systematically underestimates band gaps, to what extent depends on
the material. If this is the case there is nothing to do unless moving to
other approximations that at least partially solve this problem
(DFT+U,hybrid functionals,GW,)
2) do the references you mention concern theoretical estimations within DFT
or experimental data? While when comparing with experiments you
might experient the limit of the previous point, whereas if you find other
calculations, within the same level of approximation, you should find at
least
similar (if not equal) results
3) in the case you have reference calculations to compare with and there is
no agreement, check your input geometry and unit cell (to understand if they
are wrong) as well as the convergence parameters (Brillouin zone sampling,
wave function cutoff, charge density cutoff if it applies, )

Giovanni

-- 

Giovanni Cantele, PhD
CNR-SPIN
c/o Dipartimento di Fisica
Universita' di Napoli "Federico II"
Complesso Universitario M. S. Angelo - Ed. 6
Via Cintia, I-80126, Napoli, Italy
e-mail: giovanni.cant...@spin.cnr.it 
Phone: +39 081 676910
Skype contact: giocan74

ResearcherID: http://www.researcherid.com/rid/A-1951-2009
Web page: https://sites.google.com/view/giovanni-cantele/home


Il giorno ven 16 feb 2024 alle ore 10:40 PRASAD SANKALPA WANNINAYAKA <
2019s17...@stu.cmb.ac.lk> ha scritto:

> Dear QE users,
> I am studying the electronic properties of FeTiO3 using QE when
> calculating the band structure using 'bands.x' calculation. I obtained the
> bandgap as 0.01eV but according to references that value is 2.54 eV. i need
> to know why this is happening.
>
> Thank you
> Best regards
> Prasad Sankalpa
> Physics Undergraduate
>
> --
>
> The information of this email is confidential. If you have received it by
> error, please inform us by email and then delete the message. It is illegal
> to disclose the contents of this message to anyone. The integrity or
> security of this email cannot be guaranteed over the Internet.  Therefore,
> the sender/university will not be responsible for any damage caused by this
> email.
>
> SINHALA <https://cmb.ac.lk/email-policy/sinhala#disclaimer> | TAMIL
> <https://cmb.ac.lk/email-policy/tamil#disclaimer>
> ___
> The Quantum ESPRESSO community stands by the Ukrainian
> people and expresses its concerns about the devastating
> effects that the Russian military offensive has on their
> country and on the free and peaceful scientific, cultural,
> and economic cooperation amongst peoples
> ___
> Quantum ESPRESSO is supported by MaX (www.max-centre.eu)
> users mailing list users@lists.quantum-espresso.org
> https://lists.quantum-espresso.org/mailman/listinfo/users
___
The Quantum ESPRESSO community stands by the Ukrainian
people and expresses its concerns about the devastating
effects that the Russian military offensive has on their
country and on the free and peaceful scientific, cultural,
and economic cooperation amongst peoples
___
Quantum ESPRESSO is supported by MaX (www.max-centre.eu)
users mailing list users@lists.quantum-espresso.org
https://lists.quantum-espresso.org/mailman/listinfo/users

Re: [QE-users] wrong plane average electrostatic potential. !! Sorry, I forgot to upload the files in the first message.

2024-01-13 Thread Giovanni Cantele
Consider also that the paper you mention shows the potential in eV units,
whereas QE output is in Ry.
If you look at the energy window spanned by the potential, in the paper it
ranges from about (very rough estimates) -12 to about 3 eV, ~15 eV in
total, whereas your potential ranges from -0.9 to 0.3 Ry, for a total of
1.2 Ry, ~16 eV in total, maybe a careful calculation would show a closer
match between the two.
Another term of comparison, is the difference between top and bottom
constant values, reported to be 1.207 eV in the paper. Your calculation
shows something like ~0.1 Ry => 1.36 eV, again not that different (and with
a better estimation that can be done using the data).

Giovanni

-- 

Giovanni Cantele, PhD
CNR-SPIN
c/o Dipartimento di Fisica
Universita' di Napoli "Federico II"
Complesso Universitario M. S. Angelo - Ed. 6
Via Cintia, I-80126, Napoli, Italy
e-mail: giovanni.cant...@spin.cnr.it 
Phone: +39 081 676910
Skype contact: giocan74

ResearcherID: http://www.researcherid.com/rid/A-1951-2009
Web page: https://sites.google.com/view/giovanni-cantele/home


Il giorno sab 13 gen 2024 alle ore 10:18 Thomas Brumme <
thomas.bru...@tu-dresden.de> ha scritto:

> Dear Wilbur,
>
> Can you please explain what should be wrong? You have the same number of
> minima, the relative depths seem to match. The absolut position is not
> important but only relative energies.
>
> Cheerio
>
> Thomas
> 
> From: users  on behalf of
> Wilber Muriel 
> Sent: Saturday, January 13, 2024 5:33:54 AM
> To: Quantum ESPRESSO users Forum
> Subject: [QE-users] wrong plane average electrostatic potential. !! Sorry,
> I forgot to upload the files in the first message.
>
> Dear QE users and depeveloper:
>
> I am trying to reproduce the planar average electrostatic potential of
> In2Se3, as in figure 3B of the article
> https://www.frontiersin.org/articles/10.3389/fchem.2023.1278370/full, but
> I get a wrong result. Attached are the input files and the graph I get.
>
> I don't know if the error is due to my input, or some software bug.
>
> Can someone tell me if I am doing something wrong?
> ___
> The Quantum ESPRESSO community stands by the Ukrainian
> people and expresses its concerns about the devastating
> effects that the Russian military offensive has on their
> country and on the free and peaceful scientific, cultural,
> and economic cooperation amongst peoples
> ___
> Quantum ESPRESSO is supported by MaX (www.max-centre.eu)
> users mailing list users@lists.quantum-espresso.org
> https://lists.quantum-espresso.org/mailman/listinfo/users
>
___
The Quantum ESPRESSO community stands by the Ukrainian
people and expresses its concerns about the devastating
effects that the Russian military offensive has on their
country and on the free and peaceful scientific, cultural,
and economic cooperation amongst peoples
___
Quantum ESPRESSO is supported by MaX (www.max-centre.eu)
users mailing list users@lists.quantum-espresso.org
https://lists.quantum-espresso.org/mailman/listinfo/users

Re: [QE-users] ph.x

2024-01-10 Thread Giovanni Cantele
Dear Elham,

you can browse the source distribution, look for example
to PHonon/examples/README, where the provided examples are listed. Then you
can try to study
those examples that fit your needs (many are for crystal structures).

Moreover, there are tutorials available online, besides Google Search, you
might find helpful the related QE web site page:
https://www.quantum-espresso.org/tutorials/

Giovanni

-- 

Giovanni Cantele, PhD
CNR-SPIN
c/o Dipartimento di Fisica
Universita' di Napoli "Federico II"
Complesso Universitario M. S. Angelo - Ed. 6
Via Cintia, I-80126, Napoli, Italy
e-mail: giovanni.cant...@spin.cnr.it 
Phone: +39 081 676910
Skype contact: giocan74

ResearcherID: http://www.researcherid.com/rid/A-1951-2009
Web page: https://sites.google.com/view/giovanni-cantele/home


Il giorno mer 10 gen 2024 alle ore 16:51 Elham Rezaee 
ha scritto:

> Dear Researchers,
>
> Does anyone have an input file of ph.x  for a crystal structure? and any
> reference that helps me to learn this part.
>
> Thank you
>
> Elham Rezaee, PhD student
> University of New Brunswick, Canada
> ___
> The Quantum ESPRESSO community stands by the Ukrainian
> people and expresses its concerns about the devastating
> effects that the Russian military offensive has on their
> country and on the free and peaceful scientific, cultural,
> and economic cooperation amongst peoples
> ___
> Quantum ESPRESSO is supported by MaX (www.max-centre.eu)
> users mailing list users@lists.quantum-espresso.org
> https://lists.quantum-espresso.org/mailman/listinfo/users
>
___
The Quantum ESPRESSO community stands by the Ukrainian
people and expresses its concerns about the devastating
effects that the Russian military offensive has on their
country and on the free and peaceful scientific, cultural,
and economic cooperation amongst peoples
___
Quantum ESPRESSO is supported by MaX (www.max-centre.eu)
users mailing list users@lists.quantum-espresso.org
https://lists.quantum-espresso.org/mailman/listinfo/users

Re: [QE-users] Inquiry Regarding Crystal Structure Discrepancy in Quantum Espresso

2023-12-30 Thread Giovanni Cantele
As I infer from the input you sent me, if you take atomic positions,
designed to fill the unit cell of a simple tetragonal crystal and use the
same to fill the sites of a body centered tetragonal lattice, you'll
definitely obtain two different structures, one should worry in the case
they look equal!

What you should observe is that your atomic positions are "redundant" in
that if instead of replicating them on the sites of a simple tetragonal
lattice you replicate a fraction of them on the sites of a body centered
lattice, then the two resulting structures are the same. In this case,
since the st primitive cell has twice the volume of a bct primitive cell,
you should halve the positions (of course removing the ones equivalente by
a translation of the bct lattice), then ibrav=7 will definitely work.
BEWARE: your positions are in crystal units. That means they are expressed
in units of a1,a2,a3 of the st lattice. As such, besides halving the number
of atoms, you have to change also the positions of the remaining ones, if
you keep crystal as units (because a1',a2',a3' of bct are different from
a1,a2,a3 of st), otherwise convert them in Angstrom first.

Giovanni
-- 

Giovanni Cantele, PhD
CNR-SPIN
c/o Dipartimento di Fisica
Universita' di Napoli "Federico II"
Complesso Universitario M. S. Angelo - Ed. 6
Via Cintia, I-80126, Napoli, Italy
e-mail: giovanni.cant...@spin.cnr.it 
Phone: +39 081 676910
Skype contact: giocan74

ResearcherID: http://www.researcherid.com/rid/A-1951-2009
Web page: https://sites.google.com/view/giovanni-cantele/home


Il giorno sab 30 dic 2023 alle ore 17:03 houssam eddine hailouf <
h.hail...@yahoo.com> ha scritto:

> Please sir if I can proceed my calculations with ibrav =6 and if this
> state is correct " 7 is for body centered tetragonal. 6 is for primitive
> one"
>
> My problem,
> but when I write Ibrav =7 and I use xcrysden to visualize the structure I
> found other structure and when I write Ibra =6 I find the relevant
> structure.
>
> Sincerely
> Hailouf
>
>
> Le sam., déc. 30, 2023 à 16:20, Giovanni Cantele
>  a écrit:
> and what is the file with ibrav=7 that gives you incorrect results?
> GC
> --
>
> Giovanni Cantele, PhD
> CNR-SPIN
> c/o Dipartimento di Fisica
> Universita' di Napoli "Federico II"
> Complesso Universitario M. S. Angelo - Ed. 6
> Via Cintia, I-80126, Napoli, Italy
> e-mail: giovanni.cant...@spin.cnr.it 
> Phone: +39 081 676910
> Skype contact: giocan74
>
> ResearcherID: http://www.researcherid.com/rid/A-1951-2009
> Web page: https://sites.google.com/view/giovanni-cantele/home
>
>
> Il giorno sab 30 dic 2023 alle ore 16:07 houssam eddine hailouf <
> h.hail...@yahoo.com> ha scritto:
>
> Hi dear Giovanni
>
> Please find my input file attached below such as the system under
> consideration is body-centered tetragonal crystal structure (I41md), but
> when i display the crystal structure with ibrav= 7 i get wrong crystal
> structure but it's fine with ibrav=6.
>
> Sincerely
> Hailouf
> Le samedi 30 décembre 2023 à 14:38:13 UTC+1, Giovanni Cantele <
> giovanni.cant...@spin.cnr.it> a écrit :
>
>
> Dear Hailouf,
>
> I think it is not easy to help you with the information you provided.
> 1) What do you mean with "an incorrect crystal structure"? Is
> Quantum-ESPRESSO giving a structure you do not expect or the calculation
> fails for some other reason? In the case, what is the "error" provided by
> the code?
> 2) In ibrav=6 produces the "correct structure", then your structure is
> simple tetragonal? Or you are trying to reproduce a body-centered
> tetragonal structure using a simple tetragonal unit cell?
>
> With some more detail (and/or input/output files), maybe a more helpful
> answer could be issued.
>
> Giovanni
>
> --
>
> Giovanni Cantele, PhD
> CNR-SPIN
> c/o Dipartimento di Fisica
> Universita' di Napoli "Federico II"
> Complesso Universitario M. S. Angelo - Ed. 6
> Via Cintia, I-80126, Napoli, Italy
> e-mail: giovanni.cant...@spin.cnr.it 
> Phone: +39 081 676910
> Skype contact: giocan74
>
> ResearcherID: http://www.researcherid.com/rid/A-1951-2009
> Web page: https://sites.google.com/view/giovanni-cantele/home
>
>
> Il giorno sab 30 dic 2023 alle ore 14:05 houssam eddine hailouf via users <
> users@lists.quantum-espresso.org> ha scritto:
>
> Dear Quantum Espresso Group,
>
> I am currently studying systems within the body-centered tetragonal
> structure. I've encountered an issue where setting ibrav = 7 yields an
> incorrect crystal structure, while using ibrav = 6 produces the correct
> structure, and calculations proceed smoothly. Could you provide any
> insights or explanations regardin

Re: [QE-users] Inquiry Regarding Crystal Structure Discrepancy in Quantum Espresso

2023-12-30 Thread Giovanni Cantele
Dear Hailouf,

I think it is not easy to help you with the information you provided.
1) What do you mean with "an incorrect crystal structure"? Is
Quantum-ESPRESSO giving a structure you do not expect or the calculation
fails for some other reason? In the case, what is the "error" provided by
the code?
2) In ibrav=6 produces the "correct structure", then your structure is
simple tetragonal? Or you are trying to reproduce a body-centered
tetragonal structure using a simple tetragonal unit cell?

With some more detail (and/or input/output files), maybe a more helpful
answer could be issued.

Giovanni

-- 

Giovanni Cantele, PhD
CNR-SPIN
c/o Dipartimento di Fisica
Universita' di Napoli "Federico II"
Complesso Universitario M. S. Angelo - Ed. 6
Via Cintia, I-80126, Napoli, Italy
e-mail: giovanni.cant...@spin.cnr.it 
Phone: +39 081 676910
Skype contact: giocan74

ResearcherID: http://www.researcherid.com/rid/A-1951-2009
Web page: https://sites.google.com/view/giovanni-cantele/home


Il giorno sab 30 dic 2023 alle ore 14:05 houssam eddine hailouf via users <
users@lists.quantum-espresso.org> ha scritto:

> Dear Quantum Espresso Group,
>
> I am currently studying systems within the body-centered tetragonal
> structure. I've encountered an issue where setting ibrav = 7 yields an
> incorrect crystal structure, while using ibrav = 6 produces the correct
> structure, and calculations proceed smoothly. Could you provide any
> insights or explanations regarding this situation?
>
> Thank you,
> Hailouf houssam
> Materials Science and Informatics Laboratory, Djelfa algerie.
> ___
> The Quantum ESPRESSO community stands by the Ukrainian
> people and expresses its concerns about the devastating
> effects that the Russian military offensive has on their
> country and on the free and peaceful scientific, cultural,
> and economic cooperation amongst peoples
> ___
> Quantum ESPRESSO is supported by MaX (www.max-centre.eu)
> users mailing list users@lists.quantum-espresso.org
> https://lists.quantum-espresso.org/mailman/listinfo/users
___
The Quantum ESPRESSO community stands by the Ukrainian
people and expresses its concerns about the devastating
effects that the Russian military offensive has on their
country and on the free and peaceful scientific, cultural,
and economic cooperation amongst peoples
___
Quantum ESPRESSO is supported by MaX (www.max-centre.eu)
users mailing list users@lists.quantum-espresso.org
https://lists.quantum-espresso.org/mailman/listinfo/users

Re: [QE-users] SCF convergence issue after first ionic step

2023-12-19 Thread Giovanni Cantele
Hi, I do not have a precise explanation of that, but I can say that I found
such a behavior several times, in the years,
for completely different systems.
And this concerns not only the convergence of the scf step, but also the
relaxation
of the atomic positions.
At some iteration it seems that the algorithm falls in some "potential
well" and cannot exit it at all.

In this respect, it helps often just to restart the calculation from
scratch using one of the last coordinate sets found by
the pw.x, or in some cases restart the code introducing in the coordinates
even very small perturbations (e.g. delete the
digits from the fourth to the last). In other cases, you might try to
increase the smearing, even just for a few steps, mostly if
the problem is the convergence of the scf cycle.
This is just empirical, but it is the signature for tricky calculations
(where there is no parameter
to identify when a calculation is going to become "tricky"!), where the
presence of local minima or other numerically hard problems,
either with respect to the charge density or with respect to the atomic
positions, can make the convergence non as easy as expected.

Maybe, someone more expert might add some more specific hints.

Giovanni

-- 

Giovanni Cantele, PhD
CNR-SPIN
c/o Dipartimento di Fisica
Universita' di Napoli "Federico II"
Complesso Universitario M. S. Angelo - Ed. 6
Via Cintia, I-80126, Napoli, Italy
e-mail: giovanni.cant...@spin.cnr.it 
Phone: +39 081 676910
Skype contact: giocan74

ResearcherID: http://www.researcherid.com/rid/A-1951-2009
Web page: https://sites.google.com/view/giovanni-cantele/home


Il giorno mar 19 dic 2023 alle ore 08:49 Laurent Pizzagalli <
laurent.pizzaga...@univ-poitiers.fr> ha scritto:

> Dear all,
>
> I came across an issue I never encountered before. I performed a simple
> ionic relaxation of a W cluster. The initial scf convergence is
> achieved, the ionic forces are calculated (and are reasonable for a
> first ionic relaxation step) and the new atomic positions are obtained.
> The issue arises during the next scf calculation, which diverges in a
> few steps. However, if I start a new calculation with the updated ionic
> positions as input structure, the scf calculation runs smoothly and
> converges (but the issue occurred again for the next ionic relaxation
> steps).
> I obtained the same behavior with two different pw.x versions (6.7 and
> 7.1). I also tried to set 'pot_extrapolation = none', but with no
> difference...
>
> Do any of you have an idea about the origin (and solution) of this issue ?
>
> Thanks
>
> Laurent
>
> --
> ,,, __,
>/'^'\   |__|
>   ( o o )  |
> --oOOO--(_)--OO|o--
> 
> http://laurent.pizzagalli.free.fr/Tel +33 549 49 74 99
> --Fax +33 549 49 66 92
> Institut P'
> Departement de Physique et de Mécanique des Matériaux
> CNRS UPR 3346
> Université de Poitiers
> SP2MI
> TSA 41123  .oooO
> 86073 Poitiers Cedex 9, FRANCE (   )   Oooo.
> \ ((   )---
>   \_)) /
> (_/
>
> ___
> The Quantum ESPRESSO community stands by the Ukrainian
> people and expresses its concerns about the devastating
> effects that the Russian military offensive has on their
> country and on the free and peaceful scientific, cultural,
> and economic cooperation amongst peoples
> ___
> Quantum ESPRESSO is supported by MaX (www.max-centre.eu)
> users mailing list users@lists.quantum-espresso.org
> https://lists.quantum-espresso.org/mailman/listinfo/users
___
The Quantum ESPRESSO community stands by the Ukrainian
people and expresses its concerns about the devastating
effects that the Russian military offensive has on their
country and on the free and peaceful scientific, cultural,
and economic cooperation amongst peoples
___
Quantum ESPRESSO is supported by MaX (www.max-centre.eu)
users mailing list users@lists.quantum-espresso.org
https://lists.quantum-espresso.org/mailman/listinfo/users

Re: [QE-users] Spatial dependency of the wave functions for the spin-orbit coupling calculation

2023-12-11 Thread Giovanni Cantele
I'm not expert about HDF5 files, however because you are seeking for wave
function coefficients, the answer might be that, as explained for example
here
https://www.quantum-espresso.org/Doc/pw_user_guide/node10.html#SECTION00043030
when spin-orbit is turned on the wave functions are two-component spinors.
This also reflect the fact that, with nelec electrons, the ground-state
density suffices nelec/2 Kohn-Sham states (using spin degeneracy) in the
absence of spin-orbit
coupling, whereas including the latter you need to compute (for
semiconductors) at least nelec states.

Giovanni

-- 

Giovanni Cantele, PhD
CNR-SPIN
c/o Dipartimento di Fisica
Universita' di Napoli "Federico II"
Complesso Universitario M. S. Angelo - Ed. 6
Via Cintia, I-80126, Napoli, Italy
e-mail: giovanni.cant...@spin.cnr.it 
Phone: +39 081 676910
Skype contact: giocan74

ResearcherID: http://www.researcherid.com/rid/A-1951-2009
Web page: https://sites.google.com/view/giovanni-cantele/home


Il giorno lun 11 dic 2023 alle ore 13:49 Alireza Shabani via users <
users@lists.quantum-espresso.org> ha scritto:

> Dear QE users,
>
> I am going to calculate the spatial dependency of the wave function from
> the output HDF5 files from QE (for one specific k-point, such as Gamma). I
> know that the number 'evc' should be twice the 'Miller Indices' because it
> includes real and imaginary coefficients in the wave function: psi = sigma
> (C_real + i C_imag) * exp(iG.r).
>
> Everything looks fine, and I can calculate the wave function in a normal
> DFT calculation (I mean without spin-orbit coupling!!). But, when I use
> spin-orbit coupling, the number of 'evc' in the final .hdf5 file becomes
> four times the 'Miller Indices.' For instance, if I have 100 'Miller
> Indices, ' I will have 400 'evc' which is a bit confusing. Does anybody
> know how we can interpret it?
>
> Thank you for your suggestions.
>
>
>
> Kind regards,
>
> Alireza Shabani
>
> Postdoc researcher,
> Denmark Technical University
> Copenhagen, Denmark
>
>
> ___
> The Quantum ESPRESSO community stands by the Ukrainian
> people and expresses its concerns about the devastating
> effects that the Russian military offensive has on their
> country and on the free and peaceful scientific, cultural,
> and economic cooperation amongst peoples
> ___
> Quantum ESPRESSO is supported by MaX (www.max-centre.eu)
> users mailing list users@lists.quantum-espresso.org
> https://lists.quantum-espresso.org/mailman/listinfo/users
>
___
The Quantum ESPRESSO community stands by the Ukrainian
people and expresses its concerns about the devastating
effects that the Russian military offensive has on their
country and on the free and peaceful scientific, cultural,
and economic cooperation amongst peoples
___
Quantum ESPRESSO is supported by MaX (www.max-centre.eu)
users mailing list users@lists.quantum-espresso.org
https://lists.quantum-espresso.org/mailman/listinfo/users

Re: [QE-users] Missing d orbital in projected band structure of CaC6

2023-07-25 Thread Giovanni Cantele
Dear Bruce,

if I'm not wrong you're using this pseudopotential:
http://pseudopotentials.quantum-espresso.org/upf_files/Ca.pbe-spn-rrkjus_psl.1.0.0.UPF

If you look inside you see that it has been generate with this valence
configuration:
nl pn  l   occ   RcutRcut US   E pseu
3S  1  0  2.00  1.200  1.300-3.461142
4S  2  0  2.00  1.200  1.300-0.276804
3P  2  1  6.00  1.400  1.600-2.058527
4P  3  1  0.00  1.400  1.600-0.103085

As such, when you try to project a band structure of a system containing
this atom, it will only be able to project on 3S, 4S, 3P and 4P states.
You may also check this by looking, inside the UPF file, for the string
PP_GIPAW_ORBITAL
After lines such that

you find the radial part of the corresponding atomic wfc to be used for the
projection. Therefore, if an orbital is not included, the code does not
have access to the radial part of the wfc and will not be able to make any
projection.

Giovanni

-- 

Giovanni Cantele, PhD
CNR-SPIN
c/o Dipartimento di Fisica
Universita' di Napoli "Federico II"
Complesso Universitario M. S. Angelo - Ed. 6
Via Cintia, I-80126, Napoli, Italy
e-mail: giovanni.cant...@spin.cnr.it 
Phone: +39 081 676910
Skype contact: giocan74

ResearcherID: http://www.researcherid.com/rid/A-1951-2009
Web page: https://sites.google.com/view/giovanni-cantele/home


Il giorno mar 25 lug 2023 alle ore 09:40 Bruce Wang 
ha scritto:

> Dear QE users and developers,
>
> I am currently working on calculating the projected band structure of CaC6
> using quantum espresso with GGA+UItrasoft pseudopotential (
> Ca.pbe-spn-rrkjus_psl.1.0.0.UPF). However, I have encountered an issue
> with obtaining the 3d orbital wavefunctions for Calcium.
>
> I followed the QE PP examples, and I have managed to obtain the 4s and 3p
> orbitals. The 3d orbital was missing while it should existed.
>
> I would greatly appreciate any guidance on how to retrieve the 3d orbital
> character of Calcium for my projected band structure calculations.
>
> Thank you for your time and assistance.
>
>
> Regards,
> Bruce
> ___
> The Quantum ESPRESSO community stands by the Ukrainian
> people and expresses its concerns about the devastating
> effects that the Russian military offensive has on their
> country and on the free and peaceful scientific, cultural,
> and economic cooperation amongst peoples
> ___
> Quantum ESPRESSO is supported by MaX (www.max-centre.eu)
> users mailing list users@lists.quantum-espresso.org
> https://lists.quantum-espresso.org/mailman/listinfo/users
>
___
The Quantum ESPRESSO community stands by the Ukrainian
people and expresses its concerns about the devastating
effects that the Russian military offensive has on their
country and on the free and peaceful scientific, cultural,
and economic cooperation amongst peoples
___
Quantum ESPRESSO is supported by MaX (www.max-centre.eu)
users mailing list users@lists.quantum-espresso.org
https://lists.quantum-espresso.org/mailman/listinfo/users

Re: [QE-users] "a" parameter in SYSTEM card

2023-07-09 Thread Giovanni Cantele
Dear David,
the value of a is definitely in Angstrom (
https://www.quantum-espresso.org/Doc/INPUT_PW.html#idm258).
If with 12 A it works and with 6 A it does not work, this means that maybe
your unit cell with 6 A side is not able
to accommodate the atoms (this could be inferred only if you provided the
whole input).

Let me also point out that, as far as the CELL_PARAMETERS card is
concerned, you should (maybe) find in the output
(at the beginning) a message saying that it has been ignored. Indeed, if
you DO specify ibrav >0, the unit cell vectors
are chosen by the code with the parameters given in input (
https://www.quantum-espresso.org/Doc/INPUT_PW.html#idm224),
whereas the input from the CELL_PARAMETERS ard is allowed only with ibrav =
0.

Generally speaking, it is worth visualizing the input (using for example
XCrysDen able to directly open QE input files) before
attempting any calculation.

Giovanni

-- 

Giovanni Cantele, PhD
CNR-SPIN
c/o Dipartimento di Fisica
Universita' di Napoli "Federico II"
Complesso Universitario M. S. Angelo - Ed. 6
Via Cintia, I-80126, Napoli, Italy
e-mail: giovanni.cant...@spin.cnr.it 
Phone: +39 081 676910
Skype contact: giocan74

ResearcherID: http://www.researcherid.com/rid/A-1951-2009
Web page: https://sites.google.com/view/giovanni-cantele/home


Il giorno sab 8 lug 2023 alle ore 13:28  ha scritto:

> Dear all,
>
>   I'm starting to study a perovskite and I am a bit confused about the
> parameter "a" of the SYSTEM card.
>
>   According to the pw.x input page of QE this parameter must be in
> angstroms. I have a calculation in which, apparently, this parameter
> is in bohr:
>
> 
>   ibrav = 2
>   a = 12.1616
>
>   With these numbers, the calculation starts and finshes well.
>
>   I have done the simple check of changing "a" to 6.0808. This last
> value should be angstroms and it should work, according to the webpage:
>
> 
>   ibrav = 2
>   a = 6.0808
>
>   But, this new calculation does not finish and blows up just after
> starting.
>
>   For both of them, I have used:
>
> CELL_PARAMETERS angstrom
>   6.08 -4.10  0.00
>   6.08 -4.10  0.00
> -0.46  0.00 31.37
>
>   ...
>
> ATOMIC_POSITIONS crystal
> Pb 0.45 0.06 0.04
> Pb 0.06 0.45 0.54
>
>   ...
>
>
>   I don't know what is going on. The parameter "a" is in bohr or in
> angstroms?
>
>   Thanks for your help, David.
>
>
>
> --
> David López Durán
> Department of Physics
> University of Córdoba, Spain
> Phone: +34 957 21 20 32
> https://es.linkedin.com/in/davidlopezduran
>
> ___
> The Quantum ESPRESSO community stands by the Ukrainian
> people and expresses its concerns about the devastating
> effects that the Russian military offensive has on their
> country and on the free and peaceful scientific, cultural,
> and economic cooperation amongst peoples
> ___
> Quantum ESPRESSO is supported by MaX (www.max-centre.eu)
> users mailing list users@lists.quantum-espresso.org
> https://lists.quantum-espresso.org/mailman/listinfo/users
>
___
The Quantum ESPRESSO community stands by the Ukrainian
people and expresses its concerns about the devastating
effects that the Russian military offensive has on their
country and on the free and peaceful scientific, cultural,
and economic cooperation amongst peoples
___
Quantum ESPRESSO is supported by MaX (www.max-centre.eu)
users mailing list users@lists.quantum-espresso.org
https://lists.quantum-espresso.org/mailman/listinfo/users

Re: [QE-users] Effect of nbnd on vc-relax

2023-06-05 Thread Giovanni Cantele
Dear Jing,

generally speaking, if you treat the system as a semiconductor (no
occupations keywork, no nbnd specified), the default number of bands will
be the number of electrons divided by two.
For metallic systems or systems that are treated as they were metallic
(occupations = 'smearing') the systems increases this number of bands by a
factor (if I remember well by 20%), because
this is needed in the convergence of the scf cycles when a subset ob bands
may cross the Fermi level.

So, if QE automatically sets the number of bands to 960, it means that you
have 960/1.2 = 800 KS states. It is clear that if your system is metallic
or if it has a very small gap, the scf cycle will never
(or quite hardly) converge if you set nbnd = 800 (nor the code will output
an error, that is instead issued if you set nbnd < nelec/2, for scf/relax).

Concerning the time needed to complete the calculation, this depends on a
number of factors: number of electrons (high in your case), number of
k-points (doubled for spin-polarized calculations
as yours),  starting geometry (far or close to the equilibrium one?),
number of needed plane waves (your cutoff is not that small). Moreover, it
can significantly depend on the number of processes
that you allocate on your supercomputer and on the way you use the parallel
features of QE.

So, the answers to your questions:
- And how should I determine an appropriate value of nbnd before starting
any calculation?
The easiest way (for relax/scf) is to let QE choose the default, bearing in
mind that if your system has small gaps, is metallic or is spin polarized
you need occupations = 'smearing'.

- Is this a normal timescale for a system with >100 atoms?
It can be normal, but there is not sufficient information to establish
whether or not you're efficiently exploring the parallelization features of
QE.

Giovanni

-- 

Giovanni Cantele, PhD
CNR-SPIN
c/o Dipartimento di Fisica
Universita' di Napoli "Federico II"
Complesso Universitario M. S. Angelo - Ed. 6
Via Cintia, I-80126, Napoli, Italy
e-mail: giovanni.cant...@spin.cnr.it 
Phone: +39 081 676910
Skype contact: giocan74

ResearcherID: http://www.researcherid.com/rid/A-1951-2009
Web page: https://sites.google.com/view/giovanni-cantele/home


Il giorno sab 3 giu 2023 alle ore 22:18 Jing Lian Ng 
ha scritto:

> Hello all,
>
> I am trying to do a vc-relax calculation on y-Fe2O3 with 160 atoms.
> Currently I am facing convergence issue with the input script below. With
> nbnd = 800, the calculation usually finishes in ~ 1 hr on a supercomputer,
> but fails to reach convergence even on the first BFGS step. When I removed
> the nbnd parameter, QE automatically set the number of Kohn-Sham states to
> 960 and the calculation was able to proceed. However, the relaxation
> calculation is taking more than 72 hours and hasn't reached completion (It
> is still running now, on 15th BFGS cycle). Is this a normal timescale for a
> system with >100 atoms? And how should I determine an appropriate value of
> nbnd before starting any calculation?
>
>  Input Script 
> 
>ibrav   = 14
>A   = 8.48998
>B   = 8.48998
>C   = 25.46994
>cosAB   = 0
>cosAC   = 0
>cosBC   = 0
>nat = 160
>ntyp= 3
>ecutwfc = 65
>ecutrho = 780
>nbnd= 800
>occupations = 'smearing'
>degauss = 0.001
>smearing= 'mv'
>nspin   = 2
>starting_magnetization(1) = 0.5
>starting_magnetization(2) = -0.5
>starting_magnetization(3) = 0
>  /
>  
>conv_thr= 0.1
>mixing_mode = 'plain'
>mixing_beta = 0.3
>electron_maxstep = 200
>  /
>  
>  /
>  
>!cell_dofree = 'volume'
>  /
> ATOMIC_SPECIES
>   FeO   55.84500  Fe.pbesol-spn-kjpaw_psl.0.2.1.UPF
>   FeT   55.84500  Fe.pbesol-spn-kjpaw_psl.0.2.1.UPF
> O   15.99900  O.pbesol-n-kjpaw_psl.0.1.UPF
> ##
>
> Thanks,
> Jing Lian Ng
> Graduate student at CHE, University of Texas at Austin
>
> ___
> The Quantum ESPRESSO community stands by the Ukrainian
> people and expresses its concerns about the devastating
> effects that the Russian military offensive has on their
> country and on the free and peaceful scientific, cultural,
> and economic cooperation amongst peoples
> ___
> Quantum ESPRESSO is supported by MaX (www.max-centre.eu)
> users mailing list users@lists.quantum-espresso.org
> https://lists.quantum-espresso.org/mailman/listinfo/users
___
The Quan

Re: [QE-users] Regarding Charge density difference

2023-05-31 Thread Giovanni Cantele
All the atoms should be kept frozen and you need to run also for the "top"
part alone.

Giovanni
-- 

Giovanni Cantele, PhD
CNR-SPIN
c/o Dipartimento di Fisica
Universita' di Napoli "Federico II"
Complesso Universitario M. S. Angelo - Ed. 6
Via Cintia, I-80126, Napoli, Italy
e-mail: giovanni.cant...@spin.cnr.it 
Phone: +39 081 676910
Skype contact: giocan74

ResearcherID: http://www.researcherid.com/rid/A-1951-2009
Web page: https://sites.google.com/view/giovanni-cantele/home


Il giorno mer 31 mag 2023 alle ore 06:37 Satyasiban Dash ph19d005 <
ph19d...@smail.iitm.ac.in> ha scritto:

> Thank You for the response. I have already optimised the structure and
> removed top layer, and running for only the bottom part . So all the
> remaining atoms needs to be frozen or only ones near the interface will do
> the job ?
>
> Its a 3d structure having vacuum along z-axix.
>
> Thank You.
>
> Satya
> IIT Madras
>
> On Wed, May 31, 2023 at 12:26 PM Giovanni Cantele <
> giovanni.cant...@spin.cnr.it> wrote:
>
>> If you want to visualize the charge density variations induced by the
>> formation of the interface, once you have the optimized structure of the
>> heterostructure and
>> its charge density rho_tot, you have to make, at frozen atomic positions
>> two more scf calculations, obtained from the former by removing one layer
>> at a time, thus obtaining
>> let's say rho_1 and rho_2. What you're looking for is then easily
>> computed from rho_tot - rho_1 - rho_2.
>>
>> In this way, you just visualize charge density reconstruction induced by
>> the interface, getting rid of contributions related to geometry changes
>> induced by the presence of
>> the interface itself. This is because if you change the structure, the
>> result would be harder visualized.
>>
>> Giovanni
>>
>> --
>>
>> Giovanni Cantele, PhD
>> CNR-SPIN
>> c/o Dipartimento di Fisica
>> Universita' di Napoli "Federico II"
>> Complesso Universitario M. S. Angelo - Ed. 6
>> Via Cintia, I-80126, Napoli, Italy
>> e-mail: giovanni.cant...@spin.cnr.it 
>> Phone: +39 081 676910
>> Skype contact: giocan74
>>
>> ResearcherID: http://www.researcherid.com/rid/A-1951-2009
>> Web page: https://sites.google.com/view/giovanni-cantele/home
>>
>>
>> Il giorno mar 30 mag 2023 alle ore 22:19 Satyasiban Dash ph19d005 <
>> ph19d...@smail.iitm.ac.in> ha scritto:
>>
>>> Dear users,
>>>
>>> I am planning to calculate charge density difference at interface of a
>>> heterostructure. So I underwent relaxation of cell ,then scf of hetero
>>> structure, so here are my small doubts
>>>
>>> 1. Do I need to delete one layer and calculate scf with same lattice
>>> cell and atomic position and do the same for other layer later ?
>>>
>>> 2. Instead of above approach do I need to make a complete different
>>> structure of upper or lower layer and take scf then rho ?
>>>
>>>
>>> Thank You.
>>>
>>>
>>> Satya
>>> PhD Scholar
>>> IIT Madras
>>> ___
>>> The Quantum ESPRESSO community stands by the Ukrainian
>>> people and expresses its concerns about the devastating
>>> effects that the Russian military offensive has on their
>>> country and on the free and peaceful scientific, cultural,
>>> and economic cooperation amongst peoples
>>> ___
>>> Quantum ESPRESSO is supported by MaX (www.max-centre.eu)
>>> users mailing list users@lists.quantum-espresso.org
>>> https://lists.quantum-espresso.org/mailman/listinfo/users
>>
>> ___
>> The Quantum ESPRESSO community stands by the Ukrainian
>> people and expresses its concerns about the devastating
>> effects that the Russian military offensive has on their
>> country and on the free and peaceful scientific, cultural,
>> and economic cooperation amongst peoples
>> ___
>> Quantum ESPRESSO is supported by MaX (www.max-centre.eu)
>> users mailing list users@lists.quantum-espresso.org
>> https://lists.quantum-espresso.org/mailman/listinfo/users
>
> ___
> The Quantum ESPRESSO community stands by the Ukrainian
> people and expresses its concerns about the devastating
> effects that the Russian military offensive has on their
> country and on the free and peaceful scientific, cultural,
> and eco

Re: [QE-users] Regarding Charge density difference

2023-05-31 Thread Giovanni Cantele
If you want to visualize the charge density variations induced by the
formation of the interface, once you have the optimized structure of the
heterostructure and
its charge density rho_tot, you have to make, at frozen atomic positions
two more scf calculations, obtained from the former by removing one layer
at a time, thus obtaining
let's say rho_1 and rho_2. What you're looking for is then easily computed
from rho_tot - rho_1 - rho_2.

In this way, you just visualize charge density reconstruction induced by
the interface, getting rid of contributions related to geometry changes
induced by the presence of
the interface itself. This is because if you change the structure, the
result would be harder visualized.

Giovanni

-- 

Giovanni Cantele, PhD
CNR-SPIN
c/o Dipartimento di Fisica
Universita' di Napoli "Federico II"
Complesso Universitario M. S. Angelo - Ed. 6
Via Cintia, I-80126, Napoli, Italy
e-mail: giovanni.cant...@spin.cnr.it 
Phone: +39 081 676910
Skype contact: giocan74

ResearcherID: http://www.researcherid.com/rid/A-1951-2009
Web page: https://sites.google.com/view/giovanni-cantele/home


Il giorno mar 30 mag 2023 alle ore 22:19 Satyasiban Dash ph19d005 <
ph19d...@smail.iitm.ac.in> ha scritto:

> Dear users,
>
> I am planning to calculate charge density difference at interface of a
> heterostructure. So I underwent relaxation of cell ,then scf of hetero
> structure, so here are my small doubts
>
> 1. Do I need to delete one layer and calculate scf with same lattice cell
> and atomic position and do the same for other layer later ?
>
> 2. Instead of above approach do I need to make a complete different
> structure of upper or lower layer and take scf then rho ?
>
>
> Thank You.
>
>
> Satya
> PhD Scholar
> IIT Madras
> ___
> The Quantum ESPRESSO community stands by the Ukrainian
> people and expresses its concerns about the devastating
> effects that the Russian military offensive has on their
> country and on the free and peaceful scientific, cultural,
> and economic cooperation amongst peoples
> ___
> Quantum ESPRESSO is supported by MaX (www.max-centre.eu)
> users mailing list users@lists.quantum-espresso.org
> https://lists.quantum-espresso.org/mailman/listinfo/users
___
The Quantum ESPRESSO community stands by the Ukrainian
people and expresses its concerns about the devastating
effects that the Russian military offensive has on their
country and on the free and peaceful scientific, cultural,
and economic cooperation amongst peoples
___
Quantum ESPRESSO is supported by MaX (www.max-centre.eu)
users mailing list users@lists.quantum-espresso.org
https://lists.quantum-espresso.org/mailman/listinfo/users

[QE-users] QE 6.5 versus QE 7.2

2023-05-19 Thread Giovanni Cantele
Dear all,

I'm redoing some old calculations that were carried out using QE 6.5,
few years ago.
The new calculations use the latest QE version.

Input files are exactly the same.

By analysing the differences of the output files, for example for an scf
calculation, I noticed what follows:

< !total energy  =-390.19169099 Ry
<  estimated scf accuracy<   0.0008 Ry
<  smearing contrib. (-TS)   =   0. Ry
<  internal energy E=F+TS=-390.19169099 Ry
---
> !total energy  =-390.18965771 Ry
>  estimated scf accuracy<   0.0042 Ry
>  smearing contrib. (-TS)   =   0.00273433 Ry
>  internal energy E=F+TS=-390.19239205 Ry

and following lines concerning contributions to total energy
<  one-electron contribution =-119.48113307 Ry
<  hartree contribution  = 120.72351434 Ry
<  xc contribution   = -99.27362121 Ry
---
>  one-electron contribution =-119.47953593 Ry
>  hartree contribution  = 120.72030987 Ry
>  xc contribution   = -99.27271495 Ry


While I would not expect to exactly the same numbers in the two cases, it
seems to me that the total energy differences are too large to be justified
with numerical oscillations, parallelization over a different number of
cores and so on:
 -390.19169099 - (-390.18965771) ~ -2 mRy ~ 28 meV

Total forces
  Total force = 0.003905 Total SCF correction = 0.000228
---
>  Total force = 0.002307 Total SCF correction = 0.000701



Stress tensor
<   total   stress  (Ry/bohr**3)   (kbar) P=
-2.14
<   -0.1057   0.  -0.0215   -1.550.00
-0.32
<0.  -0.1982   0.0.00   -2.92
 0.00
<   -0.0215   0.  -0.1321   -0.320.00
-1.94
---
>   total   stress  (Ry/bohr**3)   (kbar) P=
-2.30
>   -0.1231   0.  -0.0064   -1.810.00
-0.09
>0.  -0.1774  -0.0.00   -2.61
-0.00
>   -0.0064  -0.  -0.1687   -0.09   -0.00
-2.48



Fermi energy (in a subsequent nscf calculation)
<  the Fermi energy is 7.3474 ev
<  (compare with: 7.3589 eV, computed in scf)
---
>  the Fermi energy is 6.4844 ev
>  (compare with: 6.4868 eV, computed in scf)
(for Fermi energy I'm a little less worried, since both values fall within
the gap of the material).


Do you have any guesses about what could be the reason for these different
results? Do you think that, as it seems to me, these differences are not
that small?

I thank you in advance.
Giovanni

-- 

Giovanni Cantele, PhD
CNR-SPIN
c/o Dipartimento di Fisica
Universita' di Napoli "Federico II"
Complesso Universitario M. S. Angelo - Ed. 6
Via Cintia, I-80126, Napoli, Italy
e-mail: giovanni.cant...@spin.cnr.it 
Phone: +39 081 676910
Skype contact: giocan74

ResearcherID: http://www.researcherid.com/rid/A-1951-2009
Web page: https://sites.google.com/view/giovanni-cantele/home
___
The Quantum ESPRESSO community stands by the Ukrainian
people and expresses its concerns about the devastating
effects that the Russian military offensive has on their
country and on the free and peaceful scientific, cultural,
and economic cooperation amongst peoples
___
Quantum ESPRESSO is supported by MaX (www.max-centre.eu)
users mailing list users@lists.quantum-espresso.org
https://lists.quantum-espresso.org/mailman/listinfo/users

Re: [QE-users] Scf after relax

2023-05-11 Thread Giovanni Cantele
Dear Ricardo,

I think this couldn't/should'n happen unless something has changed from the
relax to the scf run. Maybe, if you could share input/output of relax and
scf, it could be possible to figure out what's going on.

For example: did you keep fixed some atoms or atomic coordinates in the
relax and then you did not specify the variable used to fix atomic
coordinates in scf?

Giovanni

-- 

Giovanni Cantele, PhD
CNR-SPIN
c/o Dipartimento di Fisica
Universita' di Napoli "Federico II"
Complesso Universitario M. S. Angelo - Ed. 6
Via Cintia, I-80126, Napoli, Italy
e-mail: giovanni.cant...@spin.cnr.it 
Phone: +39 081 676910
Skype contact: giocan74

ResearcherID: http://www.researcherid.com/rid/A-1951-2009
Web page: https://sites.google.com/view/giovanni-cantele/home


Il giorno gio 11 mag 2023 alle ore 21:15 Ricardo Cecconello <
rceccone...@ucs.br> ha scritto:

> Dear QE users,
>
> After relaxing the structure of a compound, I realized some SCF
> calculations using the optimized atomic positions (both taking the
> positions as in the output file and in the .xml file). Although
> maintaining the other options the same, I found a total force that is six
> times larger than that in the final relaxation cycle and some components of
> the force are 14 times larger. What could be happening?
>
> PS: the relaxation was NOT vc-relax, which I know gives different results
> at the final cycle.
>
> Enviado via UCSMail.
> ___
> The Quantum ESPRESSO community stands by the Ukrainian
> people and expresses its concerns about the devastating
> effects that the Russian military offensive has on their
> country and on the free and peaceful scientific, cultural,
> and economic cooperation amongst peoples
> ___
> Quantum ESPRESSO is supported by MaX (www.max-centre.eu)
> users mailing list users@lists.quantum-espresso.org
> https://lists.quantum-espresso.org/mailman/listinfo/users
___
The Quantum ESPRESSO community stands by the Ukrainian
people and expresses its concerns about the devastating
effects that the Russian military offensive has on their
country and on the free and peaceful scientific, cultural,
and economic cooperation amongst peoples
___
Quantum ESPRESSO is supported by MaX (www.max-centre.eu)
users mailing list users@lists.quantum-espresso.org
https://lists.quantum-espresso.org/mailman/listinfo/users

[QE-users] Strange behavior of projwfc.x

2023-03-06 Thread Giovanni Cantele
Dear all,

I'm experiencing a strange behavior of projwfc.x.

I ran it after a band calculation (to get band components over atomic wfcs)
and it worked fine.

Afterwards, I ran it after an nscf calculation with a proper grid for
plotting DOS. The output file (filproj) looks properly. However, if I look
into the *atm* files, while some of them seem to be as they should be,
others start with lines like
# E (eV)   ldos(E)   pdos(E)pdos(E)pdos(E)
  0.281E-12  0.223E-13  0.127E-12  0.131E-12
  0.418E-11  0.332E-12  0.189E-11  0.195E-11
  0.385E-10  0.305E-11  0.174E-10  0.180E-10
Those lines with the *'s end at some point, after which I see lines like
-999.991  0.000E+00  0.000E+00  0.000E+00  0.000E+00
-999.981  0.000E+00  0.000E+00  0.000E+00  0.000E+00
-999.971  0.000E+00  0.000E+00  0.000E+00  0.000E+00
-999.961  0.000E+00  0.000E+00  0.000E+00  0.000E+00
-999.951  0.000E+00  0.000E+00  0.000E+00  0.000E+00
showing energies out of range.

It is like the code had calculated the PDOS at exceedingly small energies,
starting from an energy < -1000 eV.

In the *atm* files where the problem does not show up the lowest energy is
-66.596.

The problem does not show up for files related to a particular
atomic species, in that
for a given species some projection files are ok, some others are not.

The qe version I'm using is the qe-gpu 7.1 version, as compiled on the
CINECA Marconi100 supercomputer.

Has anybody ever experienced such a strange behavior? Any hint on possible
causes?

Thank you.

Giovanni

-- 

Giovanni Cantele, PhD
CNR-SPIN
c/o Dipartimento di Fisica
Universita' di Napoli "Federico II"
Complesso Universitario M. S. Angelo - Ed. 6
Via Cintia, I-80126, Napoli, Italy
e-mail: giovanni.cant...@spin.cnr.it 
Phone: +39 081 676910
Skype contact: giocan74

Web page: https://sites.google.com/view/giovanni-cantele/home
___
The Quantum ESPRESSO community stands by the Ukrainian
people and expresses its concerns about the devastating
effects that the Russian military offensive has on their
country and on the free and peaceful scientific, cultural,
and economic cooperation amongst peoples
___
Quantum ESPRESSO is supported by MaX (www.max-centre.eu)
users mailing list users@lists.quantum-espresso.org
https://lists.quantum-espresso.org/mailman/listinfo/users

Re: [QE-users] Why my Hubbard parameter is so much bigger than what I find in the litterature?

2023-02-28 Thread Giovanni Cantele
Dear Salma,

hard to say without knowing which material and atomic species you're
dealing with. Generally speaking, if people expect a 5-6 eV U and you find
more than 3 times more, I would suspect that an error is somwhere in the
simulations.
You should certainly check whether you used appropriate calculation
parameters, e.g. BZ sampling, cutoff(s), and so on.

Giovanni
-- 

Giovanni Cantele, PhD
CNR-SPIN
c/o Dipartimento di Fisica
Universita' di Napoli "Federico II"
Complesso Universitario M. S. Angelo - Ed. 6
Via Cintia, I-80126, Napoli, Italy
e-mail: giovanni.cant...@spin.cnr.it 
Phone: +39 081 676910
Skype contact: giocan74

ResearcherID: http://www.researcherid.com/rid/A-1951-2009
Web page: https://sites.google.com/view/giovanni-cantele/home


Il giorno mar 28 feb 2023 alle ore 12:04 NAIMI SALMA 
ha scritto:

> Dear experts,
> I'm working on inorganic p-type semiconductor using qe-7.0 . I want to use
> GGA+U, so I computed the U value by following the steps from the HP
> directory/example 02 for NiO (scf1 then scf2 then then hp) without
> including the magnetization because my material is not magnetic. My hubbard
> parameter was around 20 eV. While the used parameter for the same material
> in previous papers is around 5-6 eV ( Knowing that in previous papers they
> were extracting the U parameter from the litterature). The difference is
> big between their hubbard parameter and mine is that normal or did I do
> something wrong?
>
>
>
>
>
> ---
> Salma NAIMI
> PhD student - Mohamed 5 University-Rabat
>
> ___
> The Quantum ESPRESSO community stands by the Ukrainian
> people and expresses its concerns about the devastating
> effects that the Russian military offensive has on their
> country and on the free and peaceful scientific, cultural,
> and economic cooperation amongst peoples
> ___
> Quantum ESPRESSO is supported by MaX (www.max-centre.eu)
> users mailing list users@lists.quantum-espresso.org
> https://lists.quantum-espresso.org/mailman/listinfo/users
>
___
The Quantum ESPRESSO community stands by the Ukrainian
people and expresses its concerns about the devastating
effects that the Russian military offensive has on their
country and on the free and peaceful scientific, cultural,
and economic cooperation amongst peoples
___
Quantum ESPRESSO is supported by MaX (www.max-centre.eu)
users mailing list users@lists.quantum-espresso.org
https://lists.quantum-espresso.org/mailman/listinfo/users

Re: [QE-users] Monolayer calculation

2023-02-21 Thread Giovanni Cantele
i) it would be better to sign your questions with name and affiliation
ii) since you give only information on the unit cell, an appropriate answer
is impossible. Indeed, the choice of the k-point grid is made such that the
sums over the BZ are converged within a required accuracy. This depends on
the system you're dealing with and also on the property you're interested
in (in that specific properties, such as magnetism or phonon frequencies,
might require tighter convergence)
iii) if the system is a monolayer, provided the ~8.5 A (or less if it is
distributed over different atomic layers, e.g. MoS2!) vacuum space is
enough, your grid would be in the form 2n x n x 1, considering that the
dimension of the unit cell along y is about twice that along x. The value
of n should be determined with convergence tests according to what
explained in ii)
iv) as far as "basic" questions (such as k-point grid choice) are
concerned, I think there are many tutorials (also on the QE web site) and
presentations, that might help you.

Giovanni

-- 

Giovanni Cantele, PhD
CNR-SPIN
c/o Dipartimento di Fisica
Universita' di Napoli "Federico II"
Complesso Universitario M. S. Angelo - Ed. 6
Via Cintia, I-80126, Napoli, Italy
e-mail: giovanni.cant...@spin.cnr.it 
Phone: +39 081 676910
Skype contact: giocan74

ResearcherID: http://www.researcherid.com/rid/A-1951-2009
Web page: https://sites.google.com/view/giovanni-cantele/home


Il giorno mar 21 feb 2023 alle ore 01:21 imane BEZZAOUI <
imane.bezza...@ump.ac.ma> ha scritto:

> Dear QE
>
> how I choose K-points in scf  calculation of monolayer with
> CELL_PARAMETERS angstrom
>   6.7882251740   0.00   0.00
>   0.00  11.7575511932   0.00
>   0.00   0.00   8.5425634384
> ___
> The Quantum ESPRESSO community stands by the Ukrainian
> people and expresses its concerns about the devastating
> effects that the Russian military offensive has on their
> country and on the free and peaceful scientific, cultural,
> and economic cooperation amongst peoples
> ___
> Quantum ESPRESSO is supported by MaX (www.max-centre.eu)
> users mailing list users@lists.quantum-espresso.org
> https://lists.quantum-espresso.org/mailman/listinfo/users
___
The Quantum ESPRESSO community stands by the Ukrainian
people and expresses its concerns about the devastating
effects that the Russian military offensive has on their
country and on the free and peaceful scientific, cultural,
and economic cooperation amongst peoples
___
Quantum ESPRESSO is supported by MaX (www.max-centre.eu)
users mailing list users@lists.quantum-espresso.org
https://lists.quantum-espresso.org/mailman/listinfo/users

Re: [QE-users] Can an increase in the basis-set increase the total energy of a system?

2023-02-21 Thread Giovanni Cantele
Hi,

users writing on this forum are usually requested to sign their post with
name and affiliation.

This being said, what you find can be perfectly logic.

You have to distinguish between ecutrho and ecutwfc.

The total energy is expected to have a "variational" behavior  with respect
to ecutwfc, because ecutwfc controls the size of the plane wave basis set:
the larger ecutwfc, the larger the basis set, the smaller the total energy.
At which values of ecutwfc should one stop? This can be controlled by
looking at the convergence of calculated properties (e.g. forces of energy
differences or phonon frequencies, and so on, depending on what you're
interested in).

A different logic must be applied to ecutrho. Indeed, ecutrho controls the
representation of the part of the charge density that is not expected to
change significantly during the scf cycle and that is added to the total
charge density at the end of each scf cycle. For norm-conserving
pseudopotential ecuttho=4*ecutwfc exactly reproduces the charge density,
whereas for pseudopotentials that are not norm conserving you need more
plane waves to represent the charge density localized on the atomic cores.
As such, you fix ecutrho = 6, 8, 12,  times ecutwfc (see also here:
https://www.quantum-espresso.org/Doc/INPUT_PW.html#idm301). So, saying that
you find a minimum total energy with respect to ecutrho is strictly
speaking "wrong", because this parameter does not control the size of the
basis set but only the accuracy with which you represent the charge
density. So, the purpose is that of finding a value of ecutrho at which you
reach a satisfactory convergence, not the minimum energy. The total energy,
as a function of ecutrho, can either increase or decrease, because it is a
matter of numerical accuracy (good versus band representation of the charge
density), not of basis set size.

Giovanni

-- 

Giovanni Cantele, PhD
CNR-SPIN
c/o Dipartimento di Fisica
Universita' di Napoli "Federico II"
Complesso Universitario M. S. Angelo - Ed. 6
Via Cintia, I-80126, Napoli, Italy
e-mail: giovanni.cant...@spin.cnr.it 
Phone: +39 081 676910
Skype contact: giocan74

ResearcherID: http://www.researcherid.com/rid/A-1951-2009
Web page: https://sites.google.com/view/giovanni-cantele/home


Il giorno mar 21 feb 2023 alle ore 10:56 NM SALMA <
rose-marron2...@hotmail.fr> ha scritto:

> Hello,
>
> I was doing the convergence testing of CuI material.
> For a fixed Ecutwfc value, when I increase the Ecutrho the total energy of
> the system increase. I was expecting the inverse.
> For example for Ecutwfc=90 Ry and Ecutrho= 180 Ry the total energy=
> -2368.11704109 Ry
> and for Ecutwfc=90 Ry and Ecutrho=450 Ry the total energy= -2368.11613851
> Ry.
> Is that logic? I thought that an increase in the basis-set minimize the
> system energy.
>
> I also remarked in the pseudopotential file  that :
>   For Cu : - Suggested minimum cutoff for wavefunctions:  45. Ry
> - Suggested minimum cutoff for charge density: 236. Ry
>
>   For I  : - Suggested minimum cutoff for wavefunctions:  24. Ry
>  - Suggested minimum cutoff for charge density: 109. Ry
>
> So how it can be that the suggested minimum cutoff for charge density of
> Cu is 236 Ry, while I found that the system have a minimum total energy for
> Ecutrho=180 Ry
> ___
> The Quantum ESPRESSO community stands by the Ukrainian
> people and expresses its concerns about the devastating
> effects that the Russian military offensive has on their
> country and on the free and peaceful scientific, cultural,
> and economic cooperation amongst peoples
> ___
> Quantum ESPRESSO is supported by MaX (www.max-centre.eu)
> users mailing list users@lists.quantum-espresso.org
> https://lists.quantum-espresso.org/mailman/listinfo/users
>
___
The Quantum ESPRESSO community stands by the Ukrainian
people and expresses its concerns about the devastating
effects that the Russian military offensive has on their
country and on the free and peaceful scientific, cultural,
and economic cooperation amongst peoples
___
Quantum ESPRESSO is supported by MaX (www.max-centre.eu)
users mailing list users@lists.quantum-espresso.org
https://lists.quantum-espresso.org/mailman/listinfo/users

Re: [QE-users] Band structure calculation

2022-12-28 Thread Giovanni Cantele
Hi,
it is always a good practice to look at the documentation first.

Here, you can find some answers to your questions:
https://www.quantum-espresso.org/Doc/INPUT_PW.html#idm1487

nks:  Number of supplied special k-points.
So, you decide how many points you want in your path (e.g., 50) and fix nks
to that value.

How to write the k-point path: given that your crystal is 1D, this is
an easy task. As you might read from the above link,
you specify the units of supplied k-points after K_POINTS. Let's use
'crystal' units and let us suppose that the direction of periodicity
of your crystal is the x one.
Your path (in crystal units) begins from -0.5 0.0 0.0 and ends to 0.5 0.0
0.0. As an example you can specify
K_POINTS { crystal }
11
-0.5  0.0  0.0.   1.0
-0.4  0.0  0.0.   1.0
-0.3  0.0  0.0.   1.0
-0.2  0.0  0.0.   1.0
-0.1  0.0  0.0.   1.0
 0.0  0.0  0.0.   1.0
 0.1  0.0  0.0.   1.0
 0.2  0.0  0.0.   1.0
 0.3  0.0  0.0.   1.0
 0.4  0.0  0.0.   1.0
 0.5  0.0  0.0.   1.0

A smooth band structure plot would require a finer division of the [-0.5
0.0 0.0] - [0.5 0.0 0.0] segment.
The fourth column (that with all the "1.0") contains k-points weights that
would be suitably set in the self-consistent calculation, but they do not
affect the results in a band structure not self consistent calculation (and
can be set to any value).

Giovanni
-- 

Giovanni Cantele, PhD
CNR-SPIN
c/o Dipartimento di Fisica
Universita' di Napoli "Federico II"
Complesso Universitario M. S. Angelo - Ed. 6
Via Cintia, I-80126, Napoli, Italy
e-mail: giovanni.cant...@spin.cnr.it 
Phone: +39 081 676910
Skype contact: giocan74

ResearcherID: http://www.researcherid.com/rid/A-1951-2009
Web page: https://sites.google.com/view/giovanni-cantele/home


Il giorno dom 25 dic 2022 alle ore 17:23 Rameswar Bhattacharjee <
rb1...@georgetown.edu> ha scritto:

> Hi Everyone,
> I am trying to do a band structure calculation of a 1D organic crystal.
> The path of the band should be   * -pi/a-gamma-pi/a*  as used in the
> previous literature. Can anyone tell me how to write the k-path from -pi/a
> to +pi/a via gamma point in the QE input? Also, how to choose the "nks"
> value for each k-points coordinate.
>
> Any suggestion would be highly appreciated. I am attaching an image from a
> previous literature for a sample reference. Thank you in advance and Merry
> Christmas to all
>
> Rameswar Bhattacharjee
> Georgetown University
> ___
> The Quantum ESPRESSO community stands by the Ukrainian
> people and expresses its concerns about the devastating
> effects that the Russian military offensive has on their
> country and on the free and peaceful scientific, cultural,
> and economic cooperation amongst peoples
> ___
> Quantum ESPRESSO is supported by MaX (www.max-centre.eu)
> users mailing list users@lists.quantum-espresso.org
> https://lists.quantum-espresso.org/mailman/listinfo/users
___
The Quantum ESPRESSO community stands by the Ukrainian
people and expresses its concerns about the devastating
effects that the Russian military offensive has on their
country and on the free and peaceful scientific, cultural,
and economic cooperation amongst peoples
___
Quantum ESPRESSO is supported by MaX (www.max-centre.eu)
users mailing list users@lists.quantum-espresso.org
https://lists.quantum-espresso.org/mailman/listinfo/users

Re: [QE-users] Convergence tests for slabs

2022-12-19 Thread Giovanni Cantele
Strictly speaking, you should get convergence of the clean slab first,
concerning both the slab thickness and the vacuum. Consider that thin
slabs might suffer from "quantum confinement" effects, so it is important
to look at the changes of the electronic properties if you are interested
in accurately reproducing them, especialy as far as 'bulk' states are
concerned.

Once you place a molecule on the slab, you should take care of the fact
that the molecule "thickness" in the direction perpendicular to the slab
actually corresponds to decreasing the vacuum width. As such, you should
add to the vacuum this thickness, even approximately, to avoid what you
mention.

Also consider that, if you want to get rid of effects of the slab asymmetry
once the molecule has been placed on the slab, you should include dipole
corrections. The alternative is to build a symmetric slab+molecule system,
where the molecule is on both sides of the slab, which however is much more
computationally expensive, because it requires a large unit cell size in
the direction perpendicular to the slab.

Giovanni
-- 

Giovanni Cantele, PhD
CNR-SPIN
c/o Dipartimento di Fisica
Universita' di Napoli "Federico II"
Complesso Universitario M. S. Angelo - Ed. 6
Via Cintia, I-80126, Napoli, Italy
e-mail: giovanni.cant...@spin.cnr.it 
Phone: +39 081 676910
Skype contact: giocan74

ResearcherID: http://www.researcherid.com/rid/A-1951-2009
Web page: https://sites.google.com/view/giovanni-cantele/home


Il giorno lun 19 dic 2022 alle ore 10:32 Léon Luntadila Lufungula <
luntadilat...@hotmail.com> ha scritto:

> Dear QE users,
>
>
>
> I’m creating a slab for studying adsorption of small molecules (around 6-8
> atoms in length) and want to converge both the slab thickness and vacuum
> width. However, I don’t fully understand how to do this properly… For the
> convergence of the slab thickness, I was thinking about taking slabs of
> different thickness and calculating the surface energy and look at when the
> difference between two consecutive surface energies is smaller than 1
> kcal/mol (i.e. chemical accuracy), but I’ve also heard that you can look at
> the difference in energy when adding a layer and converging this energy
> difference to chemical accuracy. Any suggestions on which is the better
> option? For the vacuum width, I’m confused if you should converge this
> value on a clean slab or on a slab with a model compound adsorbed onto it.
> I’ve been told you should do this on a clean slab, but I would expect that
> once an adsorbate is bonded onto the surface, the end of the adsorbate
> might be close enough to the next layer to interact with it, which would
> make the convergence test obsolete…
>
>
>
> I would be most thankful if someone could tell me the correct procedure in
> this situation!
>
>
>
> Thanks in advance and kind regards,
>
> Léon Luntadila Lufungula
>
> Structural Chemistry group
>
> University of Antwerp
>
>
>
>
> ___
> The Quantum ESPRESSO community stands by the Ukrainian
> people and expresses its concerns about the devastating
> effects that the Russian military offensive has on their
> country and on the free and peaceful scientific, cultural,
> and economic cooperation amongst peoples
> ___
> Quantum ESPRESSO is supported by MaX (www.max-centre.eu)
> users mailing list users@lists.quantum-espresso.org
> https://lists.quantum-espresso.org/mailman/listinfo/users
>
___
The Quantum ESPRESSO community stands by the Ukrainian
people and expresses its concerns about the devastating
effects that the Russian military offensive has on their
country and on the free and peaceful scientific, cultural,
and economic cooperation amongst peoples
___
Quantum ESPRESSO is supported by MaX (www.max-centre.eu)
users mailing list users@lists.quantum-espresso.org
https://lists.quantum-espresso.org/mailman/listinfo/users

[QE-users] No or band convergence for WS2 bilayer

2022-11-04 Thread Giovanni Cantele
Dear all,

I'm facing a bad convergence issue for a very simple system consisting of a
WS2 bilayer.

While with NO vdW interaction turned on everything works fine, as I use the
input_dft variable the optimization task fails. In practice, the pw.x tries
to optimize the bilayer for some steps, when the Total force decreases,
then the Total force starts increasing, until at some point (=geometry),
the electron minimization does not converge within electron_maxstep = 300
steps.

Here, a typical input file I use:


calculation   = 'relax'
title = 'WS1-2L'
restart_mode  = 'from_scratch'
pseudo_dir   = '/m100_work/IscrB_TWITTER/pseudo/'
outdir= './tmp/'
prefix= 'WS2-2L'
verbosity = 'high'
tprnfor   = .true.
tstress   = .true.
!etot_conv_thr = 1.D-5
forc_conv_thr = 1.D-6
/

ibrav = 4
a = __A
c = 24.450
nat   = 6
ntyp  = 2
ecutwfc   = 50
ecutrho   = 500
occupations   = 'smearing'
smearing  = 'mv'
degauss   = 0.00735
input_dft = 'vdw-df2-b86r'
/

conv_thr = 1.d-8
electron_maxstep = 300
mixing_beta  = 0.15
/

/
ATOMIC_SPECIES
W183.84   W.pbe-spn-kjpaw_psl.1.0.0.UPF
S 32.065  S.pbe-n-kjpaw_psl.1.0.0.UPF
ATOMIC_POSITIONS { crystal }
S0.7   0.3   0.080882686
W0.3   0.7   0.145220672
S0.7   0.3   0.209558657
S0.3   0.7   0.371324029
W0.7   0.3   0.435662015
S0.3   0.7   0.5
K_POINTS { automatic }
12  12  10  0  0


I tried to decrease the beta (down to 0.01), to increase degauss (even
though the system is semiconducting and this should not affect the
convergence, since the energy gap is always much larger than degauss), to
increase the k-point grid, to increase or decrease the vacuum, to increase
ecutrho.

I suspect some stupid error I'm doing and that I cannot see, since most of
the above listed recipies has worked for much more difficult systems that a
simple WS2 bilayer, unless WS2 requires some special care that I ignore, or
unless there is some issue with the W and/or S pseudopotential.
I also tried to run the run with no vdw and then that with vdw as a restart
of the previous one.

WS2 bilayer seems for example to have been successfully studied with
several (and even more recent) vdW functionals for example in this paper:
https://pubs.acs.org/doi/pdf/10.1021/acs.jctc.0c00471

I would appreciate any hint/help.

Thank you.
Giovanni

-- 

Giovanni Cantele, PhD
CNR-SPIN
c/o Dipartimento di Fisica
Universita' di Napoli "Federico II"
Complesso Universitario M. S. Angelo - Ed. 6
Via Cintia, I-80126, Napoli, Italy
e-mail: giovanni.cant...@spin.cnr.it 
Phone: +39 081 676910
Skype contact: giocan74

ResearcherID: http://www.researcherid.com/rid/A-1951-2009
Web page: https://sites.google.com/view/giovanni-cantele/home
___
The Quantum ESPRESSO community stands by the Ukrainian
people and expresses its concerns about the devastating
effects that the Russian military offensive has on their
country and on the free and peaceful scientific, cultural,
and economic cooperation amongst peoples
___
Quantum ESPRESSO is supported by MaX (www.max-centre.eu)
users mailing list users@lists.quantum-espresso.org
https://lists.quantum-espresso.org/mailman/listinfo/users

Re: [QE-users] Difference in k-path for electron and phonon bands

2022-10-19 Thread Giovanni Cantele
Dear Arthur,

maybe it would be easier to help you looking at at least part of the input
files. However, one possible explanation could be that you are using the
same coordinates of the k or q points in both calculations, but by default
one expects some units, the other, different units. For example, as far as
I remember (to be checked), matdyn.x requires q-point coordinates in 2 pi /
a units, in the case, is that you have provided in the input?

Giovanni

-- 

Giovanni Cantele, PhD
CNR-SPIN
c/o Dipartimento di Fisica
Universita' di Napoli "Federico II"
Complesso Universitario M. S. Angelo - Ed. 6
Via Cintia, I-80126, Napoli, Italy
e-mail: giovanni.cant...@spin.cnr.it 
Phone: +39 081 676910
Skype contact: giocan74

ResearcherID: http://www.researcherid.com/rid/A-1951-2009
Web page: https://sites.google.com/view/giovanni-cantele/home


Il giorno mer 19 ott 2022 alle ore 08:58 Arthur Arthur via users <
users@lists.quantum-espresso.org> ha scritto:

> Dear Users and Developers,
> I would like to plot phonons band structure calculated in the following
> way:
> scf (pw.x) -> ph (ph.x) -> q2r (q2r.x) -> matdyn (matdyn.x) -> plotband
> (plotband.x)
>
> and electron band structure calculated in the following way:
> scf (pw.x) -> nscf (pw.x) -> bands (pw.x) -> bands (bands.x)
>
> Of course, both calculations for the same parameters and k-path generated
> using the xcrysden.
> Unfortunately, I found the difference in the high-symmetry line (please
> see attachment).
> For phonons, the line is longer and the positions of K and M are
> inconsistent with these obtained for electrons.
>
> Something is wrong because the high-symmetry line and G-K-M-G positions
> should be the same. Where could be a mistake?
>
> Thank you for your response.
>
> Arthur
> University of Birmingham
>
> ___
> The Quantum ESPRESSO community stands by the Ukrainian
> people and expresses its concerns about the devastating
> effects that the Russian military offensive has on their
> country and on the free and peaceful scientific, cultural,
> and economic cooperation amongst peoples
> ___
> Quantum ESPRESSO is supported by MaX (www.max-centre.eu)
> users mailing list users@lists.quantum-espresso.org
> https://lists.quantum-espresso.org/mailman/listinfo/users
___
The Quantum ESPRESSO community stands by the Ukrainian
people and expresses its concerns about the devastating
effects that the Russian military offensive has on their
country and on the free and peaceful scientific, cultural,
and economic cooperation amongst peoples
___
Quantum ESPRESSO is supported by MaX (www.max-centre.eu)
users mailing list users@lists.quantum-espresso.org
https://lists.quantum-espresso.org/mailman/listinfo/users

Re: [QE-users] the band structure of Au(111)

2022-10-14 Thread Giovanni Cantele
1) the attached image is not publicly available
2) usually users of this forum are asked to sign posts with their name and
affiliation

If you could provide more information on the problem, that does not look
that clear to me, I'll be happy to try to help you.
Giovanni

-- 

Giovanni Cantele, PhD
CNR-SPIN
c/o Dipartimento di Fisica
Universita' di Napoli "Federico II"
Complesso Universitario M. S. Angelo - Ed. 6
Via Cintia, I-80126, Napoli, Italy
e-mail: giovanni.cant...@spin.cnr.it 
Phone: +39 081 676910
Skype contact: giocan74

ResearcherID: http://www.researcherid.com/rid/A-1951-2009
Web page: https://sites.google.com/view/giovanni-cantele/home

Il giorno ven 14 ott 2022 alle ore 14:38 imane BEZZAOUI <
imane.bezza...@ump.ac.ma> ha scritto:

> Dear all
>
> I would ask you when I plot the band structure of Au(111) with 7 layers to
> get the Shockley states and I want to add more layers But as in the graph
> below, I obtained the band as two separate curves instead of obtaining only
> one.
>  au7layers.png
> <https://drive.google.com/file/d/1HsGgX5gAekaiLSevHPb1hi-8X3g5S9N2/view?usp=drive_web>
>
> ___
> The Quantum ESPRESSO community stands by the Ukrainian
> people and expresses its concerns about the devastating
> effects that the Russian military offensive has on their
> country and on the free and peaceful scientific, cultural,
> and economic cooperation amongst peoples
> ___
> Quantum ESPRESSO is supported by MaX (www.max-centre.eu)
> users mailing list users@lists.quantum-espresso.org
> https://lists.quantum-espresso.org/mailman/listinfo/users
___
The Quantum ESPRESSO community stands by the Ukrainian
people and expresses its concerns about the devastating
effects that the Russian military offensive has on their
country and on the free and peaceful scientific, cultural,
and economic cooperation amongst peoples
___
Quantum ESPRESSO is supported by MaX (www.max-centre.eu)
users mailing list users@lists.quantum-espresso.org
https://lists.quantum-espresso.org/mailman/listinfo/users

[QE-users] A question regarding tot_magnetization

2022-09-27 Thread Giovanni Cantele
Dear users/developers,

I've a simple question regarding the tot_magnetization keyword. I have a
system composed by the repetition of n units, each one that should carry a
1.0 magnetization. The total magnetization should be n x 1.0.
In the isolated unit, if I use, let's say, starting_magnetization(1) = 0.1,
it correctly converges towards 1.0 magnetization. However, in the system
where this unit cell is repeated n times along a given direction, the
system wants for some values of n to converge to a much higher energy with
a 5.0 magnetization per unit (so 5xn in total). To restore the correct
ground state, I set tot_magnetization = n.

The question is: the result I get "constraining" the magnetization with the
tot_magnetization keyword is necessarily the same (in terms, for example,
of electronic levels) as that I would obtain using starting_magnetization
if the system spontaneously converged to a magnetization equal to n, or the
constraint could force, in some cases, unusual occupations and so I get the
correct magnetization but this would not correspond to the ground state
with that magnetization I'm looking for?
The question arises because in some cases I get the warning
BEWARE: non-integer number of up and down electrons!
As such, I want to be sure that the "BEWARE" concerns only the "starting"
occupations, but then the convergence is to the correct ground state,
whereas it is not the case that "unusual" (or, even worse, "unphysical")
occupations are forced even though the final magnetization is the same as
that I expect.

I thank you in advance for your attention.

Giovanni

-- 

Giovanni Cantele, PhD
CNR-SPIN
c/o Dipartimento di Fisica
Universita' di Napoli "Federico II"
Complesso Universitario M. S. Angelo - Ed. 6
Via Cintia, I-80126, Napoli, Italy
e-mail: giovanni.cant...@spin.cnr.it 
Phone: +39 081 676910
Skype contact: giocan74

ResearcherID: http://www.researcherid.com/rid/A-1951-2009
Web page: https://sites.google.com/view/giovanni-cantele/home
___
The Quantum ESPRESSO community stands by the Ukrainian
people and expresses its concerns about the devastating
effects that the Russian military offensive has on their
country and on the free and peaceful scientific, cultural,
and economic cooperation amongst peoples
___
Quantum ESPRESSO is supported by MaX (www.max-centre.eu)
users mailing list users@lists.quantum-espresso.org
https://lists.quantum-espresso.org/mailman/listinfo/users

Re: [QE-users] error of band-structure calculations with tpiba_c

2022-09-08 Thread Giovanni Cantele
The partial outcome in the first case is not enough, because one wants to
check what the executable is reading from input. What looks weird is:
i) from a previous input it seems you specify three k-points, but I can see
only two instead
2) in the two k-points I see there are NaN, this is not possible
3) even more impossible,
 Smooth grid:0 G-vectors FFT dimensions: (   1,   1,   1)

So either the input contains something really wrong or I can only guess
your executable is badly compiled or has for no other reason I can guess
serious problems.
Giovanni

-- 

Giovanni Cantele, PhD
CNR-SPIN
c/o Dipartimento di Fisica
Universita' di Napoli "Federico II"
Complesso Universitario M. S. Angelo - Ed. 6
Via Cintia, I-80126, Napoli, Italy
e-mail: giovanni.cant...@spin.cnr.it 
Phone: +39 081 676910
Skype contact: giocan74

ResearcherID: http://www.researcherid.com/rid/A-1951-2009
Web page: https://sites.google.com/view/giovanni-cantele/home

Il giorno gio 8 set 2022 alle ore 11:13 Jibiao Li  ha
scritto:

>
> Hi, Giovanni
>
> I used command:  pw.x  top.nscf.out&
>
> The partial outcome of the out file is shown below,
>
> ...
>  number of k points= 2  Methfessel-Paxton smearing, width (Ry)=
> 0.0200
>cart. coord. in units 2pi/alat
> k(1) = ( NaN NaN NaN), wk =   0.500
> k(2) = ( NaN NaN NaN), wk =   0.500
>
>  Dense  grid:   925237 G-vectors FFT dimensions: (  75,  75, 360)
>
>  Smooth grid:0 G-vectors FFT dimensions: (   1,   1,   1)
>
>  Estimated max dynamical RAM per process >   3.58 GB
>
>
>  
> %%
>  Error in routine allocate_fft (1):
>  wrong ngms
>
>  
> %%
>
>  stopping ...
>
>
> When I used command: mpirun -np 2 pw.x  top.nscf.out&
>
> The terminal gave me the error below
>
>  
> %%
>  Error in routine  fft_type_set (6):
>   there are processes with no planes. Use pencil decomposition (-pd .true.)
>
>  
> %%
>
>  stopping ...
> Abort(6) on node 1 (rank 1 in comm 0): application called
> MPI_Abort(MPI_COMM_WORLD, 6) - process 1
>
>
> The compete out file is shown below
>  Program PWSCF v.6.8 starts on  8Sep2022 at 17:10:37
>
>  This program is part of the open-source Quantum ESPRESSO suite
>  for quantum simulation of materials; please cite
>  "P. Giannozzi et al., J. Phys.:Condens. Matter 21 395502 (2009);
>  "P. Giannozzi et al., J. Phys.:Condens. Matter 29 465901 (2017);
>  "P. Giannozzi et al., J. Chem. Phys. 152 154105 (2020);
>   URL http://www.quantum-espresso.org;,
>  in publications or presentations arising from this work. More details
> at
>  http://www.quantum-espresso.org/quote
>
>  Parallel version (MPI), running on 2 processors
>
>  MPI processes distributed on 1 nodes
>  R & G space division:  proc/nbgrp/npool/nimage =   2
>  61852 MiB available memory on the printing compute node when the
> environment starts
>
>  Waiting for input...
>  Reading input from standard input
>
>  Current dimensions of program PWSCF are:
>  Max number of different atomic species (ntypx) = 10
>  Max number of k-points (npk) =  4
>  Max angular momentum in pseudopotentials (lmaxx) =  4
>
>  Atomic positions and unit cell read from directory:
>  ./top.save/
>  Atomic positions from file used, from input discarded
>
>  file O.pbe-n-kjpaw_psl.1.0.0.UPF: wavefunction(s)  2S 2P renormalized
>  file C.pbe-n-kjpaw_psl.1.0.0.UPF: wavefunction(s)  2S 2P renormalized
>  file Fe.pbe-spn-kjpaw_psl.1.0.0.UPF: wavefunction(s)  3P 3D
> renormalized
>
>  IMPORTANT: XC functional enforced from input :
>  Exchange-correlation= VDW-DF
>(   1   4   4   0   1   0   0)
>  Any further DFT definition will be discarded
>  Please, verify this is what you really want
>
>
>  Subspace diagonalization in iterative solution of the eigenvalue
> problem:
>  a serial algorithm will be used
>
> (nothing else)
>
> -- Original --
> *From:* "Quantum ESPRESSO users Forum" ;
> *Date:* Thu, Sep 8, 2022 04:49 PM
> *To:* "Quantum ESPRESSO users Forum";
> *Subject:* Re: [QE-users] error of band-structure 

Re: [QE-users] error of band-structure calculations with tpiba_c

2022-09-08 Thread Giovanni Cantele
Ok, but does top.nscf.out contains only the error message or also the other
information that is usually printed before this kind of error messages are
given
Giovanni

-- 

Giovanni Cantele, PhD
CNR-SPIN
c/o Dipartimento di Fisica
Universita' di Napoli "Federico II"
Complesso Universitario M. S. Angelo - Ed. 6
Via Cintia, I-80126, Napoli, Italy
e-mail: giovanni.cant...@spin.cnr.it 
Phone: +39 081 676910
Skype contact: giocan74

ResearcherID: http://www.researcherid.com/rid/A-1951-2009
Web page: https://sites.google.com/view/giovanni-cantele/home

Il giorno gio 8 set 2022 alle ore 10:53 Jibiao Li  ha
scritto:

> Hi, Giovanni
> I used command: mpirun -np 2 pw.x  top.nscf.out&
> and command simply pw.x  top.nscf.out&
>
> They gave me the same error
>
> -- Original --
> *From:* "Quantum ESPRESSO users Forum" ;
> *Date:* Thu, Sep 8, 2022 04:49 PM
> *To:* "Quantum ESPRESSO users Forum";
> *Subject:* Re: [QE-users] error of band-structure calculations with
> tpiba_c
>
> May I see the command and/or the submission script you use to launch pw.x?
> Also, please provide the full pw.x output header (printed before the error
> message), if any.
> Giovanni
>
> --
>
> Giovanni Cantele, PhD
> CNR-SPIN
> c/o Dipartimento di Fisica
> Universita' di Napoli "Federico II"
> Complesso Universitario M. S. Angelo - Ed. 6
> Via Cintia, I-80126, Napoli, Italy
> e-mail: giovanni.cant...@spin.cnr.it 
> Phone: +39 081 676910
> Skype contact: giocan74
>
> ResearcherID: http://www.researcherid.com/rid/A-1951-2009
> Web page: https://sites.google.com/view/giovanni-cantele/home
>
> Il giorno gio 8 set 2022 alle ore 10:21 Jibiao Li 
> ha scritto:
>
>>  Dear Giovanni,
>>
>> >That error should mean that you're using more cpus than the dimension of
>> >the FFT grid along the z direction. Reducing the number of cpus should fix
>> >the error.
>> The error below remains even if I used only one cpu. Any idea?
>>
>>  
>> %%
>>  Error in routine  fft_type_set (6):
>>   there are processes with no planes. Use pencil decomposition (-pd .true.)
>>  
>> %%
>>
>> --
>>
>> Jibiao Li
>>
>> Department of Materials Science and Engineering
>>
>> Yangtze Normal University
>>
>> Juxian Avenue 16, Fuling, Chongqing, China 408100
>>
>> Scopus Research ID: 54944118000
>> <https://www.scopus.com/authid/detail.uri?authorId=54944118000>
>>
>> Web of Science Research ID: F-1905-2016
>> <https://publons.com/researcher/2283103/jibiao-li/>
>>
>> >Giovanni Cantele, PhD
>> >CNR-SPIN
>> >c/o Dipartimento di Fisica
>> >Universita' di Napoli "Federico II"
>> >Complesso Universitario M. S. Angelo - Ed. 6
>> >Via Cintia, I-80126, Napoli, Italy
>> >e-mail: giovanni.cant...@spin.cnr.it 
>> >Phone: +39 081 676910
>> >Skype contact: giocan74
>>
>> >ResearcherID: http://www.researcherid.com/rid/A-1951-2009
>> >Web page: https://sites.google.com/view/giovanni-cantele/home
>>
>> >Il giorno mer 7 set 2022 alle ore 10:32 Jibiao Li  ha
>> scritto:
>>
>> >
>> > Dear Giovanni,
>> >
>> > When I set all weights to 1.0, the calculation gave the error below
>> >
>> > K_POINTS tpiba_c
>> > 3
>> > 0.0  0.0  0.0 1.0
>> > 0.5  0.0  0.0 1.0
>> > 0.0  0.5  0.0 1.0
>> >
>> >
>> >
>> > %%
>> >  Error in routine  fft_type_set (6):
>> >   there are processes with no planes. Use pencil decomposition (-pd .true.)
>> >
>> >
>> > %%
>> >
>> > Any idea to remove this error?
>> > --
>> >
>> > Jibiao Li
>> >
>> > Department of Materials Science and Engineering
>> >
>> > Yangtze Normal University
>> >
>> > Juxian Avenue 16, Fuling, Chongqing, China 408100
>> >
>> > -- Original --
>> > *From:* "Quantum ESPRESSO users Forum" ;
>> > *Date:* Wed, Sep 7, 2022 04:19 PM
>> > *To:* "Quantum ESPRESSO users Forum";
>> > *Subject:* Re: [QE-users] error of band-structu

Re: [QE-users] error of band-structure calculations with tpiba_c

2022-09-08 Thread Giovanni Cantele
May I see the command and/or the submission script you use to launch pw.x?
Also, please provide the full pw.x output header (printed before the error
message), if any.
Giovanni

-- 

Giovanni Cantele, PhD
CNR-SPIN
c/o Dipartimento di Fisica
Universita' di Napoli "Federico II"
Complesso Universitario M. S. Angelo - Ed. 6
Via Cintia, I-80126, Napoli, Italy
e-mail: giovanni.cant...@spin.cnr.it 
Phone: +39 081 676910
Skype contact: giocan74

ResearcherID: http://www.researcherid.com/rid/A-1951-2009
Web page: https://sites.google.com/view/giovanni-cantele/home

Il giorno gio 8 set 2022 alle ore 10:21 Jibiao Li  ha
scritto:

>  Dear Giovanni,
>
> >That error should mean that you're using more cpus than the dimension of
> >the FFT grid along the z direction. Reducing the number of cpus should fix
> >the error.
> The error below remains even if I used only one cpu. Any idea?
>
>  
> %%
>  Error in routine  fft_type_set (6):
>   there are processes with no planes. Use pencil decomposition (-pd .true.)
>  
> %%
>
> --
>
> Jibiao Li
>
> Department of Materials Science and Engineering
>
> Yangtze Normal University
>
> Juxian Avenue 16, Fuling, Chongqing, China 408100
>
> Scopus Research ID: 54944118000
> <https://www.scopus.com/authid/detail.uri?authorId=54944118000>
>
> Web of Science Research ID: F-1905-2016
> <https://publons.com/researcher/2283103/jibiao-li/>
>
> >Giovanni Cantele, PhD
> >CNR-SPIN
> >c/o Dipartimento di Fisica
> >Universita' di Napoli "Federico II"
> >Complesso Universitario M. S. Angelo - Ed. 6
> >Via Cintia, I-80126, Napoli, Italy
> >e-mail: giovanni.cant...@spin.cnr.it 
> >Phone: +39 081 676910
> >Skype contact: giocan74
>
> >ResearcherID: http://www.researcherid.com/rid/A-1951-2009
> >Web page: https://sites.google.com/view/giovanni-cantele/home
>
> >Il giorno mer 7 set 2022 alle ore 10:32 Jibiao Li  ha
> scritto:
>
> >
> > Dear Giovanni,
> >
> > When I set all weights to 1.0, the calculation gave the error below
> >
> > K_POINTS tpiba_c
> > 3
> > 0.0  0.0  0.0 1.0
> > 0.5  0.0  0.0 1.0
> > 0.0  0.5  0.0 1.0
> >
> >
> >
> > %%
> >  Error in routine  fft_type_set (6):
> >   there are processes with no planes. Use pencil decomposition (-pd .true.)
> >
> >
> > %%
> >
> > Any idea to remove this error?
> > --
> >
> > Jibiao Li
> >
> > Department of Materials Science and Engineering
> >
> > Yangtze Normal University
> >
> > Juxian Avenue 16, Fuling, Chongqing, China 408100
> >
> > -- Original --
> > *From:* "Quantum ESPRESSO users Forum" ;
> > *Date:* Wed, Sep 7, 2022 04:19 PM
> > *To:* "Quantum ESPRESSO users Forum";
> > *Subject:* Re: [QE-users] error of band-structure calculations with
> > tpiba_c
> >
> > Dear Jibiao Li,
> >
> > I'm not very sure but I think that in specifying the k points in any
> > format weights are mandatory even if not used.
> >
> > As specified in the documentation
> > https://www.quantum-espresso.org/Doc/INPUT_PW.html#idm1514
> > In a non-scf calculation, weights do not affect the results.
> > If you just need eigenvalues and eigenvectors (for instance,
> > for a band-structure plot), weights can be set to any value
> > (for instance all equal to 1).
> >
> > So, it is understood that you can set all weights to 1.0 or any other
> > value for band structure calculations, but you must
> > specify some value.
> >
> > Giovanni
> >
> > --
> >
> > Giovanni Cantele, PhD
> > CNR-SPIN
> > c/o Dipartimento di Fisica
> > Universita' di Napoli "Federico II"
> > Complesso Universitario M. S. Angelo - Ed. 6
> > Via Cintia, I-80126, Napoli, Italy
> > e-mail: giovanni.cant...@spin.cnr.it 
> > Phone: +39 081 676910
> > Skype contact: giocan74
> >
> > ResearcherID: http://www.researcherid.com/rid/A-1951-2009
> > Web page: https://sites.google.com/view/giovanni-cantele/home
> >
> > Il giorno mer 7 set 2022 alle ore 09:03 Jibiao Li 
> > ha scritto:
> >
> >> Dear All,
> >>
> >> I am tryi

Re: [QE-users] error of band-structure calculations with tpiba_c

2022-09-07 Thread Giovanni Cantele
That error should mean that you're using more cpus than the dimension of
the FFT grid along the z direction. Reducing the number of cpus should fix
the error.
Giovanni

-- 

Giovanni Cantele, PhD
CNR-SPIN
c/o Dipartimento di Fisica
Universita' di Napoli "Federico II"
Complesso Universitario M. S. Angelo - Ed. 6
Via Cintia, I-80126, Napoli, Italy
e-mail: giovanni.cant...@spin.cnr.it 
Phone: +39 081 676910
Skype contact: giocan74

ResearcherID: http://www.researcherid.com/rid/A-1951-2009
Web page: https://sites.google.com/view/giovanni-cantele/home

Il giorno mer 7 set 2022 alle ore 10:32 Jibiao Li  ha
scritto:

>
> Dear Giovanni,
>
> When I set all weights to 1.0, the calculation gave the error below
>
> K_POINTS tpiba_c
> 3
> 0.0  0.0  0.0 1.0
> 0.5  0.0  0.0 1.0
> 0.0  0.5  0.0 1.0
>
>
>  
> %%
>  Error in routine  fft_type_set (6):
>   there are processes with no planes. Use pencil decomposition (-pd .true.)
>
>  
> %%
>
> Any idea to remove this error?
> --
>
> Jibiao Li
>
> Department of Materials Science and Engineering
>
> Yangtze Normal University
>
> Juxian Avenue 16, Fuling, Chongqing, China 408100
>
> -- Original --
> *From:* "Quantum ESPRESSO users Forum" ;
> *Date:* Wed, Sep 7, 2022 04:19 PM
> *To:* "Quantum ESPRESSO users Forum";
> *Subject:* Re: [QE-users] error of band-structure calculations with
> tpiba_c
>
> Dear Jibiao Li,
>
> I'm not very sure but I think that in specifying the k points in any
> format weights are mandatory even if not used.
>
> As specified in the documentation
> https://www.quantum-espresso.org/Doc/INPUT_PW.html#idm1514
> In a non-scf calculation, weights do not affect the results.
> If you just need eigenvalues and eigenvectors (for instance,
> for a band-structure plot), weights can be set to any value
> (for instance all equal to 1).
>
> So, it is understood that you can set all weights to 1.0 or any other
> value for band structure calculations, but you must
> specify some value.
>
> Giovanni
>
> --
>
> Giovanni Cantele, PhD
> CNR-SPIN
> c/o Dipartimento di Fisica
> Universita' di Napoli "Federico II"
> Complesso Universitario M. S. Angelo - Ed. 6
> Via Cintia, I-80126, Napoli, Italy
> e-mail: giovanni.cant...@spin.cnr.it 
> Phone: +39 081 676910
> Skype contact: giocan74
>
> ResearcherID: http://www.researcherid.com/rid/A-1951-2009
> Web page: https://sites.google.com/view/giovanni-cantele/home
>
> Il giorno mer 7 set 2022 alle ore 09:03 Jibiao Li 
> ha scritto:
>
>> Dear All,
>>
>> I am trying to do calculations for band-structure contour plots by using 
>> tpiba_c,
>> but the calculation stopped and give an error below:
>>
>>
>>
>> %%
>>   t
>> ask # 9 from card_kpoints : error # 1
>>   end of file while reading tpiba k points
>>
>>  
>> %%
>>
>> Something wrong with my input format below?
>>
>> K_POINTS tpiba_c
>> 3
>> 0.0  0.0  0.0
>> 0.5  0.0  0.0
>> 0.0  0.5  0.0
>>
>> I look forward to receiving your solution.
>>
>>
>> --
>>
>> Jibiao Li
>>
>> Department of Materials Science and Engineering
>>
>> Yangtze Normal University
>>
>> Juxian Avenue 16, Fuling, Chongqing, China 408100
>>
>> Scopus Research ID: 54944118000
>> <https://www.scopus.com/authid/detail.uri?authorId=54944118000>
>>
>> Web of Science Research ID: F-1905-2016
>> <https://publons.com/researcher/2283103/jibiao-li/>
>>
>> 
>> calculation = 'nscf' ,
>> restart_mode = 'from_scratch' ,
>>   outdir = './' ,
>>   pseudo_dir = '/home/jibiaoli/pseudo/PAW' ,
>>   prefix = 'top' ,
>>  tstress = .true. ,
>>  tprnfor = .true. ,
>>  /
>>  
>>ibrav = 6,
>>celldm(1) = 10.833426245,
>>celldm(3) = 4.5,
>>  nat = 26,
>> ntyp = 3,
>>  ecutwfc = 49 ,
>>  ecutrho = 451 ,
>>   

Re: [QE-users] error of band-structure calculations with tpiba_c

2022-09-07 Thread Giovanni Cantele
Dear Jibiao Li,

I'm not very sure but I think that in specifying the k points in any format
weights are mandatory even if not used.

As specified in the documentation
https://www.quantum-espresso.org/Doc/INPUT_PW.html#idm1514
In a non-scf calculation, weights do not affect the results.
If you just need eigenvalues and eigenvectors (for instance,
for a band-structure plot), weights can be set to any value
(for instance all equal to 1).

So, it is understood that you can set all weights to 1.0 or any other value
for band structure calculations, but you must
specify some value.

Giovanni

-- 

Giovanni Cantele, PhD
CNR-SPIN
c/o Dipartimento di Fisica
Universita' di Napoli "Federico II"
Complesso Universitario M. S. Angelo - Ed. 6
Via Cintia, I-80126, Napoli, Italy
e-mail: giovanni.cant...@spin.cnr.it 
Phone: +39 081 676910
Skype contact: giocan74

ResearcherID: http://www.researcherid.com/rid/A-1951-2009
Web page: https://sites.google.com/view/giovanni-cantele/home

Il giorno mer 7 set 2022 alle ore 09:03 Jibiao Li  ha
scritto:

> Dear All,
>
> I am trying to do calculations for band-structure contour plots by using 
> tpiba_c,
> but the calculation stopped and give an error below:
>
>
>
> %%
>   t
> ask # 9 from card_kpoints : error # 1
>   end of file while reading tpiba k points
>
>  
> %%
>
> Something wrong with my input format below?
>
> K_POINTS tpiba_c
> 3
> 0.0  0.0  0.0
> 0.5  0.0  0.0
> 0.0  0.5  0.0
>
> I look forward to receiving your solution.
>
>
> --
>
> Jibiao Li
>
> Department of Materials Science and Engineering
>
> Yangtze Normal University
>
> Juxian Avenue 16, Fuling, Chongqing, China 408100
>
> Scopus Research ID: 54944118000
> <https://www.scopus.com/authid/detail.uri?authorId=54944118000>
>
> Web of Science Research ID: F-1905-2016
> <https://publons.com/researcher/2283103/jibiao-li/>
>
> 
> calculation = 'nscf' ,
> restart_mode = 'from_scratch' ,
>   outdir = './' ,
>   pseudo_dir = '/home/jibiaoli/pseudo/PAW' ,
>   prefix = 'top' ,
>  tstress = .true. ,
>  tprnfor = .true. ,
>  /
>  
>ibrav = 6,
>celldm(1) = 10.833426245,
>celldm(3) = 4.5,
>  nat = 26,
> ntyp = 3,
>  ecutwfc = 49 ,
>  ecutrho = 451 ,
>nspin = 2,
>starting_magnetization(1) = -0.1,
>starting_magnetization(2) = -0.1,
>starting_magnetization(3) = 2.5,
>input_dft = 'vdw-df' ,
>  occupations = 'smearing' ,
>  degauss = 0.02D0 ,
> smearing = 'methfessel-paxton' ,
>  /
>  
> electron_maxstep = 299,
>  mixing_beta = 0.2D0 ,
>  diagonalization = 'david' ,
>  /
> ATOMIC_SPECIES
> O   15.999  O.pbe-n-kjpaw_psl.1.0.0.UPF
> C   12.0107 C.pbe-n-kjpaw_psl.1.0.0.UPF
>Fe   55.85  Fe.pbe-spn-kjpaw_psl.1.0.0.UPF
> ATOMIC_POSITIONS angstrom
> O 2.8664002.8664009.4012394775
> C 2.8664002.8664008.2401634402
> C 1.43176793951.43176793956.1720891643
> C 1.43176793954.30103206056.1720891643
> C 4.30103206051.43176793956.1720891643
> C 4.30103206054.30103206056.1720891643
> Fe0.000.006.0340426424
> Fe0.002.8664006.0179725127
> Fe2.8664000.006.0179725127
> Fe2.8664002.8664006.4391387456
> Fe1.45406686431.45406686434.3300831677
> Fe1.45406686434.27873313574.3300831677
> Fe4.27873313571.45406686434.3300831677
> Fe4.27873313574.27873313574.3300831677
> Fe0.000.002.8664000
> 0   0
> Fe0.002.8664002.8664000
> 0   0
> Fe2.8664000.002.8664000
> 0   0
> Fe2.8664002.8664002.8664000
> 0   0
> Fe1.4332001.4332001.433200

Re: [QE-users] Regarding bands

2022-04-28 Thread Giovanni Cantele
Dipole correction might certainly being a point, also given that you are 
considering a polar system. 

So you could try to share in/out files including dipole correction.

Moreover, you should also try to understand whether or not the metallicity you 
find is a spurious effect or not. 
Try, for example, to make a plot of the averaged potential along direction 
orthogonal to the heterostructure, to see if it looks reasonable. 
Also you could make charge transfer plots, again to guess if they provide at 
least reasonable results or unphysical patterns.
Another point: are you SURE that you have correctly matched the Si surface to 
the ZnO? Indeed, you said that after removing the ZnO, you find a gap. Could it 
be that metallicity is induced by “surface” (in this case “interface”) states 
of Si(100)? Depending on its reconstruction, Si(100) can show surface states 
with different or no gap between occupied and unoccupied states, as far as I 
remember.

Giovanni

> On 28 Apr 2022, at 13:38, Satyasiban Dash ph19d005 
>  wrote:
> 
> Yes . Thank you for the clarification. I am trying to match (0001) surface. 
> But I have observed upon relaxing ZnO loosing its symmetry and this may be 
> due to the polar surfaces but experimentally we do observe a band gap. I have 
> also tried dipole correction but results are same. But if I remove Si its 
> giving a gap. Is there anyway to reproduce it with Si (001).
> 
> On Thu, 28 Apr, 2022, 4:55 pm Giovanni Cantele,  <mailto:giovanni.cant...@spin.cnr.it>> wrote:
> Dear Satya,
> 
> it is not that easy (at least for me) to provide an answer to your question 
> using the data you have provided. Maybe the output files (of scf and bands) 
> and a plot of your band structure might help to infer if there is any issue 
> in your calculation.
> 
> However, before wondering whether you get a metal whereas experiments measure 
> a gap, you should also check that you are simulating a system that is as 
> close as possible to the experimental one (in terms of thickness and/or 
> surface orientation of both ZnO and Si, and so on). As far as the atomic 
> positions are concerned, they look reasonable. It seems (if I’m not wrong) 
> that you are trying to match a Si(100) surface with a ZnO(0001) surface, 
> aren’t you? Is this what you expect?
> 
> Giovanni
> 
> -- 
> 
> Giovanni Cantele, PhD
> CNR-SPIN
> c/o Dipartimento di Fisica
> Universita' di Napoli "Federico II"
> Complesso Universitario M. S. Angelo - Ed. 6
> Via Cintia, I-80126, Napoli, Italy
> e-mail: giovanni.cant...@spin.cnr.it <mailto:giovanni.cant...@spin.cnr.it>
> Phone: +39 081 676910
> Skype contact: giocan74
> 
> ResearcherID: http://www.researcherid.com/rid/A-1951-2009 
> <http://www.researcherid.com/rid/A-1951-2009>
> Web page: https://sites.google.com/view/giovanni-cantele/home 
> <https://sites.google.com/view/giovanni-cantele/home>
> 
> 
>> On 28 Apr 2022, at 07:09, Satyasiban Dash ph19d005 
>> mailto:ph19d...@smail.iitm.ac.in>> wrote:
>> 
>> Dear Users
>>  I have been trying to calculate a heterostructure with a slab model. I have 
>> followed most of the examples given in the forum and tried it still I am 
>> getting a metallic band structure. I have used the K-path generation tool 
>> provided in "material project" website  to generate k-path. I have given the 
>> scf and bands.in <http://bands.in/> file as attached files. Please have a 
>> look. Experimentally this system has a wide band gap.
>> 
>> 
>> Thanks.
>> Satya
>> Research Scholar
>> IIT Madras.
>> http://scf.in/>>> <http://job576185.pw.in/>>___
>> The Quantum ESPRESSO community stands by the Ukrainian
>> people and expresses its concerns about the devastating
>> effects that the Russian military offensive has on their
>> country and on the free and peaceful scientific, cultural,
>> and economic cooperation amongst peoples
>> ___
>> Quantum ESPRESSO is supported by MaX (www.max-centre.eu 
>> <http://www.max-centre.eu/>)
>> users mailing list users@lists.quantum-espresso.org 
>> <mailto:users@lists.quantum-espresso.org>
>> https://lists.quantum-espresso.org/mailman/listinfo/users 
>> <https://lists.quantum-espresso.org/mailman/listinfo/users>
> ___
> The Quantum ESPRESSO community stands by the Ukrainian
> people and expresses its concerns about the devastating
> effects that the Russian military offensive has on their
> country and on the free and peaceful scient

Re: [QE-users] Regarding bands

2022-04-28 Thread Giovanni Cantele
Dear Satya,

it is not that easy (at least for me) to provide an answer to your question 
using the data you have provided. Maybe the output files (of scf and bands) and 
a plot of your band structure might help to infer if there is any issue in your 
calculation.

However, before wondering whether you get a metal whereas experiments measure a 
gap, you should also check that you are simulating a system that is as close as 
possible to the experimental one (in terms of thickness and/or surface 
orientation of both ZnO and Si, and so on). As far as the atomic positions are 
concerned, they look reasonable. It seems (if I’m not wrong) that you are 
trying to match a Si(100) surface with a ZnO(0001) surface, aren’t you? Is this 
what you expect?

Giovanni

-- 

Giovanni Cantele, PhD
CNR-SPIN
c/o Dipartimento di Fisica
Universita' di Napoli "Federico II"
Complesso Universitario M. S. Angelo - Ed. 6
Via Cintia, I-80126, Napoli, Italy
e-mail: giovanni.cant...@spin.cnr.it
Phone: +39 081 676910
Skype contact: giocan74

ResearcherID: http://www.researcherid.com/rid/A-1951-2009
Web page: https://sites.google.com/view/giovanni-cantele/home


> On 28 Apr 2022, at 07:09, Satyasiban Dash ph19d005 
>  wrote:
> 
> Dear Users
>  I have been trying to calculate a heterostructure with a slab model. I have 
> followed most of the examples given in the forum and tried it still I am 
> getting a metallic band structure. I have used the K-path generation tool 
> provided in "material project" website  to generate k-path. I have given the 
> scf and bands.in <http://bands.in/> file as attached files. Please have a 
> look. Experimentally this system has a wide band gap.
> 
> 
> Thanks.
> Satya
> Research Scholar
> IIT Madras.
> ___
> The Quantum ESPRESSO community stands by the Ukrainian
> people and expresses its concerns about the devastating
> effects that the Russian military offensive has on their
> country and on the free and peaceful scientific, cultural,
> and economic cooperation amongst peoples
> ___
> Quantum ESPRESSO is supported by MaX (www.max-centre.eu)
> users mailing list users@lists.quantum-espresso.org
> https://lists.quantum-espresso.org/mailman/listinfo/users

___
The Quantum ESPRESSO community stands by the Ukrainian
people and expresses its concerns about the devastating
effects that the Russian military offensive has on their
country and on the free and peaceful scientific, cultural,
and economic cooperation amongst peoples
___
Quantum ESPRESSO is supported by MaX (www.max-centre.eu)
users mailing list users@lists.quantum-espresso.org
https://lists.quantum-espresso.org/mailman/listinfo/users

Re: [QE-users] qe-gpu 7.0 - compilation issue (MKL not detected)

2022-03-08 Thread Giovanni Cantele
Thank you all for so many hints!

Paolo’s suggestion seems to work, but besides BLAS_LIBS it is necessary to set 
LAPACK_LIBS as well, the internal $(TOPDI$)…. lapack
lib is selected otherwise.

Now things seem to work (with exception of unresolved do to some link mismatch 
of Intel MKL and PGI OpenMPI, that I can hopefully solve by myself).

The remaining issue is that if I try to set up FCFLAGS, again they are checked 
against gfortran, so the configure fails if those flags are not recognised by 
gfortran.
In other words: with NO FCFLAGS set up, nvfortran is selected correctly. But if 
I set up FCFLAGS, then the selected compiler is gfortran. Any suggestion on how 
to override
this choice?

Thanks again to all.

Giovanni

> On 8 Mar 2022, at 16:39, Ye Luo  wrote:
> 
> The error message indicates -lmkl_intel_lp64 failed to find 
> libmkl_intel_lp64.so file on the default library search path known by the 
> compiler.
> How to fix that? You can either add the directory to the default search path 
> or specify the search path with -L${DIR}.
> 1. LDFLAGS=“-L${MKLROOT}/lib/intel64" whether it fixes the issue or not 
> depends on if this flag is propagated to the configure internal test line. In 
> addition its location on the linking line may also matter.
> 2. Add  "${MKLROOT}/lib/intel64" to the environment variable LIBRARY_PATH 
> which may be honored by some compilers.
> Remember don't try to set LIBRARY_PATH persistent in your environment. It may 
> cause surprise another day when you try to use a different MKL installation 
> without deleting this line in your bashrc.
> 
> I would say both methods are not friendly to users. You could try CMake. 
> https://gitlab.com/QEF/q-e/-/wikis/Developers/CMake-build-system 
> <https://gitlab.com/QEF/q-e/-/wikis/Developers/CMake-build-system>
> What we do with CMake is first locate the exact library file and specify the 
> full path on the link line.
> Hopefully it gives you a chance of successful compilation.
> 
> Best,
> Ye
> 
> ===
> Ye Luo, Ph.D.
> Computational Science Division & Leadership Computing Facility
> Argonne National Laboratory
> 
> 
> On Tue, Mar 8, 2022 at 6:15 AM Giovanni Cantele  <mailto:giovanni.cant...@spin.cnr.it>> wrote:
> Dear Pietro, I tried both, but it does not work.
> 
> My LD_LIBRARY_PATH after sourcing vars.sh is
> /nfsexports/intel/oneapi/mkl/2021.1.1/env/../lib/intel64:/nfsexports/nvidia/hpc_sdk/Linux_x86_64/20.11/cuda/lib64:/nfsexports/nvidia/hpc_sdk/Linux_x86_64/20.11/compilers/lib:/nfsexports/nvidia/hpc_sdk/Linux_x86_64/20.11/math_libs/lib64:/nfsexports/nvidia/hpc_sdk/Linux_x86_64/20.11/comm_libs/mpi/lib:/nfsexports/nvidia/hpc_sdk/Linux_x86_64/20.11/comm_libs/nccl/lib:/nfsexports/nvidia/hpc_sdk/Linux_x86_64/20.11/comm_libs/nvshmem/lib:/usr/mpi/gcc/openmpi-4.1.0rc5/lib64:
> that indeed contains the bat to MKL library.
> 
> In install/config.log I find, among the others, this message:
> configure:5252: nvfortran -o conftest -fast -Mcache_align -Mpreprocess 
> -Mlarge_arrays -mp  -mp   conftest.f -lmkl_intel_lp64 -lmkl_intel_thread 
> -lmkl_core   >&5
> /usr/bin/ld: cannot find -lmkl_intel_lp64
> /usr/bin/ld: cannot find -lmkl_intel_thread
> /usr/bin/ld: cannot find -lmkl_core
> 
> It is like although LD_LIBRARY_PATH contains the correct path to the library, 
> ld cannot find it. On the other hand, if I pass the whole link command using 
> LDFLAGS, then configure for some reason finds
> gfrotran in place of nvfortran and then exits with errors.
> 
> Giovanni
> 
>> On 8 Mar 2022, at 13:00, Pietro Davide Delugas > <mailto:pdelu...@sissa.it>> wrote:
>> 
>> dear Giovanni 
>> you could  try just adding the /.../mkl/lib/intel64/ to the LD_LIBRARY_PATH 
>> or 
>> source of the /mkl//env/vars.sh script and then running the 
>> configure script 
>> 
>> Hope this solves your problem 
>> greetings 
>> Pietro 
>> Da: users > <mailto:users-boun...@lists.quantum-espresso.org>> per conto di Giovanni 
>> Cantele mailto:giovanni.cant...@spin.cnr.it>>
>> Inviato: martedì 8 marzo 2022 12:46
>> A: Quantum ESPRESSO users Forum > <mailto:users@lists.quantum-espresso.org>>
>> Oggetto: [QE-users] qe-gpu 7.0 - compilation issue (MKL not detected)
>>  
>> Dear all,
>> 
>> I’m trying to configure the compilation of qe-7.0 on GPUs. I’m using the 
>> following script to launch configure:
>> ---
>> export VERSION=21.3
>> export VERSION=20.11
>> export CUDA_VERSION=11.1
>> 
>> export NVHPC=/nfsexports/nvidia/hpc_sdk
>> 
>> 

Re: [QE-users] qe-gpu 7.0 - compilation issue (MKL not detected)

2022-03-08 Thread Giovanni Cantele
Dear Pietro, I tried both, but it does not work.

My LD_LIBRARY_PATH after sourcing vars.sh is
/nfsexports/intel/oneapi/mkl/2021.1.1/env/../lib/intel64:/nfsexports/nvidia/hpc_sdk/Linux_x86_64/20.11/cuda/lib64:/nfsexports/nvidia/hpc_sdk/Linux_x86_64/20.11/compilers/lib:/nfsexports/nvidia/hpc_sdk/Linux_x86_64/20.11/math_libs/lib64:/nfsexports/nvidia/hpc_sdk/Linux_x86_64/20.11/comm_libs/mpi/lib:/nfsexports/nvidia/hpc_sdk/Linux_x86_64/20.11/comm_libs/nccl/lib:/nfsexports/nvidia/hpc_sdk/Linux_x86_64/20.11/comm_libs/nvshmem/lib:/usr/mpi/gcc/openmpi-4.1.0rc5/lib64:
that indeed contains the bat to MKL library.

In install/config.log I find, among the others, this message:
configure:5252: nvfortran -o conftest -fast -Mcache_align -Mpreprocess 
-Mlarge_arrays -mp  -mp   conftest.f -lmkl_intel_lp64 -lmkl_intel_thread 
-lmkl_core   >&5
/usr/bin/ld: cannot find -lmkl_intel_lp64
/usr/bin/ld: cannot find -lmkl_intel_thread
/usr/bin/ld: cannot find -lmkl_core

It is like although LD_LIBRARY_PATH contains the correct path to the library, 
ld cannot find it. On the other hand, if I pass the whole link command using 
LDFLAGS, then configure for some reason finds
gfrotran in place of nvfortran and then exits with errors.

Giovanni

> On 8 Mar 2022, at 13:00, Pietro Davide Delugas  wrote:
> 
> dear Giovanni 
> you could  try just adding the /.../mkl/lib/intel64/ to the LD_LIBRARY_PATH 
> or 
> source of the /mkl//env/vars.sh script and then running the 
> configure script 
> 
> Hope this solves your problem 
> greetings 
> Pietro 
> Da: users  per conto di Giovanni 
> Cantele 
> Inviato: martedì 8 marzo 2022 12:46
> A: Quantum ESPRESSO users Forum 
> Oggetto: [QE-users] qe-gpu 7.0 - compilation issue (MKL not detected)
>  
> Dear all,
> 
> I’m trying to configure the compilation of qe-7.0 on GPUs. I’m using the 
> following script to launch configure:
> ---
> export VERSION=21.3
> export VERSION=20.11
> export CUDA_VERSION=11.1
> 
> export NVHPC=/nfsexports/nvidia/hpc_sdk
> 
> nvcudadir=$NVHPC/Linux_x86_64/$VERSION/cuda
> nvcompdir=$NVHPC/Linux_x86_64/$VERSION/compilers
> nvmathdir=$NVHPC/Linux_x86_64/$VERSION/math_libs
> nvcommdir=$NVHPC/Linux_x86_64/$VERSION/comm_libs
> 
> export OPAL_PREFIX=$nvcommdir/mpi
> 
> export MANPATH=$MANPATH:$nvcompdir/man
> 
> PATH=$nvcudadir/bin:$nvcompdir/bin:$nvcommdir/mpi/bin:$PATH
> export PATH
> 
> CPATH=$nvcudadir/include:$nvmathdir/include:$nvcommdir/mpi/include:$nvcommdir/nccl/include:$nvcommdir/nvshmem/include:$CPATH
> export CPATH
> 
> LD_LIBRARY_PATH=$nvcudadir/lib64:$nvcompdir/lib:$nvmathdir/lib64:$nvcommdir/mpi/lib:$nvcommdir/nccl/lib:$nvcommdir/nvshmem/lib:$LD_LIBRARY_PATH
> export LD_LIBRARY_PATH
> . /nfsexports/intel/oneapi/mkl/latest/env/vars.sh
> 
> export OMPI_FC=nvfortran
> export CUDA_HOME=$nvcudadir/$CUDA_VERSION
> 
> ./configure --enable-openmp  --with-cuda=$CUDA_HOME --with-cuda-cc=70 
> --with-cuda-runtime=11.1 --without-scalapack -with-cuda=yes 
> --enable-cuda-env-check=no 
> ---
> 
> 
> Now, the problem is that I’m not able to make configure to recognise Intel 
> MKL (2021.1.1 version).
> 
> The correct line to link to them is
>  -L${MKLROOT}/lib/intel64 -lmkl_intel_ilp64 -lmkl_pgi_thread -lmkl_core 
> -pgf90libs -mp -lpthread -lm -ldl
> and adding 
>  -i8  -I"${MKLROOT}/include”
> to the compiler options.
> 
> However, if I try to run the above configure command adding
> LDFLAGS=“-L${MKLROOT}/lib/intel64 -lmkl_intel_ilp64 -lmkl_pgi_thread 
> -lmkl_core -pgf90libs -mp -lpthread -lm -ldl”
> the configure stops finding the cuda enviroment, selects gfortran as compiler 
> and exits with error. On the other hand, if I skip LDFLAGS (but
> with MKLROOT correctly set up), the cuda environment is correctly detected 
> but MKL are not automatically detected.
> 
> Could you please check the above script and, possibly, give any hint useful 
> to solve the issue?
> 
> I thank you all in advance.
> 
> Giovanni
> 
> 
> 
> -- 
> 
> Giovanni Cantele, PhD
> CNR-SPIN
> c/o Dipartimento di Fisica
> Universita' di Napoli "Federico II"
> Complesso Universitario M. S. Angelo - Ed. 6
> Via Cintia, I-80126, Napoli, Italy
> e-mail: giovanni.cant...@spin.cnr.it <mailto:giovanni.cant...@spin.cnr.it>
> Phone: +39 081 676910
> Skype contact: giocan74
> 
> ResearcherID: http://www.researcherid.com/rid/A-1951-2009
> Web page: https://sites.google.com/view/giovanni-cantele/home 
> <https://sites.google.com/view/giovanni-cantele/home>

[QE-users] qe-gpu 7.0 - compilation issue (MKL not detected)

2022-03-08 Thread Giovanni Cantele
Dear all,

I’m trying to configure the compilation of qe-7.0 on GPUs. I’m using the 
following script to launch configure:
---
export VERSION=21.3
export VERSION=20.11
export CUDA_VERSION=11.1

export NVHPC=/nfsexports/nvidia/hpc_sdk

nvcudadir=$NVHPC/Linux_x86_64/$VERSION/cuda
nvcompdir=$NVHPC/Linux_x86_64/$VERSION/compilers
nvmathdir=$NVHPC/Linux_x86_64/$VERSION/math_libs
nvcommdir=$NVHPC/Linux_x86_64/$VERSION/comm_libs

export OPAL_PREFIX=$nvcommdir/mpi

export MANPATH=$MANPATH:$nvcompdir/man

PATH=$nvcudadir/bin:$nvcompdir/bin:$nvcommdir/mpi/bin:$PATH
export PATH

CPATH=$nvcudadir/include:$nvmathdir/include:$nvcommdir/mpi/include:$nvcommdir/nccl/include:$nvcommdir/nvshmem/include:$CPATH
export CPATH

LD_LIBRARY_PATH=$nvcudadir/lib64:$nvcompdir/lib:$nvmathdir/lib64:$nvcommdir/mpi/lib:$nvcommdir/nccl/lib:$nvcommdir/nvshmem/lib:$LD_LIBRARY_PATH
export LD_LIBRARY_PATH
. /nfsexports/intel/oneapi/mkl/latest/env/vars.sh

export OMPI_FC=nvfortran
export CUDA_HOME=$nvcudadir/$CUDA_VERSION

./configure --enable-openmp  --with-cuda=$CUDA_HOME --with-cuda-cc=70 
--with-cuda-runtime=11.1 --without-scalapack -with-cuda=yes 
--enable-cuda-env-check=no 
---


Now, the problem is that I’m not able to make configure to recognise Intel MKL 
(2021.1.1 version).

The correct line to link to them is
 -L${MKLROOT}/lib/intel64 -lmkl_intel_ilp64 -lmkl_pgi_thread -lmkl_core 
-pgf90libs -mp -lpthread -lm -ldl
and adding 
 -i8  -I"${MKLROOT}/include”
to the compiler options.

However, if I try to run the above configure command adding
LDFLAGS=“-L${MKLROOT}/lib/intel64 -lmkl_intel_ilp64 -lmkl_pgi_thread -lmkl_core 
-pgf90libs -mp -lpthread -lm -ldl”
the configure stops finding the cuda enviroment, selects gfortran as compiler 
and exits with error. On the other hand, if I skip LDFLAGS (but
with MKLROOT correctly set up), the cuda environment is correctly detected but 
MKL are not automatically detected.

Could you please check the above script and, possibly, give any hint useful to 
solve the issue?

I thank you all in advance.

Giovanni



-- 

Giovanni Cantele, PhD
CNR-SPIN
c/o Dipartimento di Fisica
Universita' di Napoli "Federico II"
Complesso Universitario M. S. Angelo - Ed. 6
Via Cintia, I-80126, Napoli, Italy
e-mail: giovanni.cant...@spin.cnr.it
Phone: +39 081 676910
Skype contact: giocan74

ResearcherID: http://www.researcherid.com/rid/A-1951-2009
Web page: https://sites.google.com/view/giovanni-cantele/home

___
The Quantum ESPRESSO community stands by the Ukrainian
people and expresses its concerns about the devastating
effects that the Russian military offensive has on their
country and on the free and peaceful scientific, cultural,
and economic cooperation amongst peoples
___
Quantum ESPRESSO is supported by MaX (www.max-centre.eu)
users mailing list users@lists.quantum-espresso.org
https://lists.quantum-espresso.org/mailman/listinfo/users

Re: [QE-users] convergence testing very high dual parameter ecutrho for ultrasoft pseudopotential

2022-03-01 Thread Giovanni Cantele
Dear Johannes,

I don’t know if I’ve well understood your point or if I’m missing something. 
However, after looking yo the plots you posted on Google Drive, it seems to me 
that actually your forces are fully converged even at the smallest values of 
ecutrho.
For example, in S.tif, I see the scale ranging from 0.130328 to 0.130340 
Ry/a.u., about 1.e-5 wide. Similarly, just to consider another case, in O5.tif 
forces range from 0.45811 to 0.45815, about 4.e-5 wide (and if you discard the 
first point the range is reduced by a further factor of 10).

Now, the default value for convergence of forces is 1.e-3 a.u., see 
https://www.quantum-espresso.org/Doc/INPUT_PW.html#idm119 


So, this means that your forces are much more converged than the threshold used 
by QE, unless you otherwise specified in your input.

You can also see some sample convergence plots on forces here:
http://theossrv1.epfl.ch/Main/ElectronicTemperature 

The vertical axis spans a range of about 0.2 eV / A ~ 0.2 / 25.7 Ry / Bohr ~ 
0.008 Ry / Bohr, again ~2-3 orders of magnitude larger that the same range in 
your plots.

Giovanni

> On 1 Mar 2022, at 19:00, Johannes Meusburger - STFC UKRI via users 
>  wrote:
> 
> Dear all, 
> I have observed a rather curious behaviour when performing convergence 
> testing for CuSO4 x 5H2O. For the convergence testing I have displaced the 
> oxygens surrounding the Cu and S atom by 10% of the equilibrium Cu-O and S-O 
> bond lengths and computed the forces on each of the atoms for various k-point 
> grids, smearing widths (marzari-vanderbilt type), ecutrho and ecutwfc. To my 
> surprise I found that the ecutrho value did not converge up to a dual 
> parameter of 5 (i.e., wfc cut = 50 Ry and ecutrho = 1000 Ry; forces vs dual 
> parameter plots available on google 
> drive:https://drive.google.com/drive/folders/1YaotezLaEHiSKouW19azGMdC6T8J-Jg8?usp=sharing
>  
> ).
>  
>  I am using ultrasoft PP from the GBRV library, usually I find that a dual 
> parameter of 2 or 3 (i.e., ecutrho 8 or 12 times larger than ecutwfc) yields 
> well converged forces. My question now is if there is any reason, why I 
> should NOT use such a high dual parameter for the geometry optimization? 
>  
> All the best and many thanks, 
>  
> Johannes 
>  
> Johannes Meusburger  
> ISIS Neutron and Muon Source, UK 
>  
> 
> This email and any attachments are intended solely for the use of the named 
> recipients. If you are not the intended recipient you must not use, disclose, 
> copy or distribute this email or any of its attachments and should notify the 
> sender immediately and delete this email from your system. UK Research and 
> Innovation (UKRI) has taken every reasonable precaution to minimise risk of 
> this email or any attachments containing viruses or malware but the recipient 
> should carry out its own virus and malware checks before opening the 
> attachments. UKRI does not accept any liability for any losses or damages 
> which the recipient may sustain due to presence of any viruses. 
> ___
> Quantum ESPRESSO is supported by MaX (www.max-centre.eu 
> )
> users mailing list users@lists.quantum-espresso.org 
> 
> https://lists.quantum-espresso.org/mailman/listinfo/users 
> 
___
Quantum ESPRESSO is supported by MaX (www.max-centre.eu)
users mailing list users@lists.quantum-espresso.org
https://lists.quantum-espresso.org/mailman/listinfo/users

Re: [QE-users] (no subject)

2022-02-23 Thread Giovanni Cantele
Dear Nirmal,

there can be several reasons why you find such a (possibly strange) behaviour. 
I give you two possible reasons, that I can easily explain.
One is that one of the two calculations (or both) is (are) wrong, but this 
point might be better assessed if you provide the inputs of both calculation. 
The second is that the two calculations are NOT carried out within the same 
framework. The point is that in plane-waves calculations the total energy is 
meaningless. It is enough that you change pseudopotentials, and the total 
energy might change by many Ry! Again, the inputs could be helpful in 
understanding if this is the case.
Much better, to compare two different calculations, are the energy differences. 
For example, in you MD simulation, you could have an initial and a final 
geometry, and you can compute the energy difference between the two.
You could then input both geometries in a scf calculation, and compute again 
their energy difference.
The two differences, should at least look similar, otherwise you can conclude 
that something wrong is affecting your results.

Giovanni

> On 23 Feb 2022, at 15:55, Nirmal Barman  wrote:
> 
> Dear Quantum Espresso Users and Developers,
> I have calculated scf energy for a graphene sheet and obtained the value 
> -205.39676576 Ry. But this scf energy is not matching or close to the energy 
> obtained at different steps of MD simulation. There is a significant 
> deviation in magnitude as well as energy is positive in case of MD 
> simulation. Could you please tell me the reason? Ideally this scf energy 
> should mimic the MD simulated result.
> 
> Thanking you
> Best regards 
> Nirmal
> ___
> Quantum ESPRESSO is supported by MaX (www.max-centre.eu)
> users mailing list users@lists.quantum-espresso.org
> https://lists.quantum-espresso.org/mailman/listinfo/users

___
Quantum ESPRESSO is supported by MaX (www.max-centre.eu)
users mailing list users@lists.quantum-espresso.org
https://lists.quantum-espresso.org/mailman/listinfo/users


Re: [QE-users] Error in band structure calculation

2022-02-17 Thread Giovanni Cantele
Dear DHILSHADA V.N.,

1) users writing to this forum are usually kindly requested to sign messages 
with their name and affiliation

2) for getting help one should provide as many details as possible, to allow 
others to figure out the kind of error and suggest possible solutions, if any.

"Bad termination error” could be related (generally speaking) to lot of 
problems, from code bugs (unlikely in your case) to operating system issues to 
bad compilation or libraries and so on.

If you just google “bad termination error quantum espresso" you end up, among 
the many, for example with
https://lists.quantum-espresso.org/pipermail/users/2015-May/032338.html 
<https://lists.quantum-espresso.org/pipermail/users/2015-May/032338.html>
where it is suggested
Error 9 = SIGKILL = some serious error. See the FAQ for "Segmentation
fault", it's nmore or less the same

So general explanations can be found here:
https://www.quantum-espresso.org/faq/faq-frequent-errors-during-execution/#5.2 
<https://www.quantum-espresso.org/faq/faq-frequent-errors-during-execution/#5.2>

More specific help could be provided if you include input and/or output and/or 
any other information might help to give help!

Giovanni


-- 

Giovanni Cantele, PhD
CNR-SPIN
c/o Dipartimento di Fisica
Universita' di Napoli "Federico II"
Complesso Universitario M. S. Angelo - Ed. 6
Via Cintia, I-80126, Napoli, Italy
e-mail: giovanni.cant...@spin.cnr.it
Phone: +39 081 676910
Skype contact: giocan74

ResearcherID: http://www.researcherid.com/rid/A-1951-2009
Web page: https://sites.google.com/view/giovanni-cantele/home

> On 18 Feb 2022, at 02:54, DHILSHADA V.N.  wrote:
> 
> Hai
> 
> I have calculated the band structure of the system which is repeating in 1 
> direction. In this system Hs are present at the end positions. I removed this 
> Hs and repeated in two directions. But it shows an error in output file of 
> band structure calculations.The error is bad termination error. Could you 
> please help me to resolve this problem?
> ___
> Quantum ESPRESSO is supported by MaX (www.max-centre.eu)
> users mailing list users@lists.quantum-espresso.org
> https://lists.quantum-espresso.org/mailman/listinfo/users

___
Quantum ESPRESSO is supported by MaX (www.max-centre.eu)
users mailing list users@lists.quantum-espresso.org
https://lists.quantum-espresso.org/mailman/listinfo/users

Re: [QE-users] Fe (110) slab isn't converging

2022-02-10 Thread Giovanni Cantele
   2.02688649600   0   > 0
> Fe6.82619145690.90460291606.7661838994
> Fe1.87248687704.4014805858   -0.2143812725
> Fe1.84855078214.29947022754.38249539040   0   > 0
> Fe1.82402846414.73444942049.2345798280
> Fe2.75826761603.12062532402.02688649600   0   > 0
> Fe2.66831697403.17398881906.8797363658
> Fe4.34744494334.3631009861   -0.1911888976
> Fe4.32468502094.42133812534.36680906560   0   > 0
> Fe5.27538046833.10172304262.1588431075
> Fe5.14356815803.24675216476.63027217250   0   > 0
> Fe6.88665532264.2727158920   -0.2151129276
> Fe6.81481771944.36019957344.3973105493
> Fe6.49798970944.60880150719.24475839300   0   > 0
> Fe7.75861794483.10036483782.1698200674
> Fe7.63174223023.24307266056.86871938500   0   > 0
> Fe2.72055630836.7460510219   -0.2000658247
> Fe2.76084547576.61729878294.5215696688
> Fe3.58577215605.46116441802.02688649600   0   > 0
> Fe3.61153787195.45957365496.6122865627
> Fe5.24075777506.63136386400.000   0   > 0
> Fe5.25482067396.70982716364.3861096452
> Fe4.86910760636.90508379809.2329583714
> Fe6.06828842205.46116441802.02688649600   0   > 0
> Fe6.10286947615.54983384136.8798135598
> Fe7.72319968806.63136386400.000   0   > 0
> Fe7.74264385446.62510841924.3864901090
> Fe7.39774462396.96840542079.2402605725
> Fe8.5507305580    5.46116441802.02688649600   0   > 0
> Fe8.60145814345.54448357006.8763383415
> ___
> Quantum ESPRESSO is supported by MaX (www.max-centre.eu)
> users mailing list users@lists.quantum-espresso.org
> https://lists.quantum-espresso.org/mailman/listinfo/users

-- 

Giovanni Cantele, PhD
CNR-SPIN
c/o Dipartimento di Fisica
Universita' di Napoli "Federico II"
Complesso Universitario M. S. Angelo - Ed. 6
Via Cintia, I-80126, Napoli, Italy
e-mail: giovanni.cant...@spin.cnr.it
Phone: +39 081 676910
Skype contact: giocan74

ResearcherID: http://www.researcherid.com/rid/A-1951-2009
Web page: https://sites.google.com/view/giovanni-cantele/home

___
Quantum ESPRESSO is supported by MaX (www.max-centre.eu)
users mailing list users@lists.quantum-espresso.org
https://lists.quantum-espresso.org/mailman/listinfo/users

Re: [QE-users] Density of states and Volume

2021-11-14 Thread Giovanni Cantele
Dear Salman, to give you an exhaustive response regarding the behaviour of your 
DOS, outputs and/or plots would be necessary. However, the only thing I would 
say you can take for sure is that the values you get from the code are in eV. 

The differences of the values you observe just by changing the cell size are 
due to a change of the physics and chemistry of your system with the cell side. 
You have made a virtual experiment in which you expand the side of the cell in 
all three direction by as much as 100%. It would have been strange if you had 
get very similar values.

Just as a check, verify that the three integrated DOS are the same.

Giovanni

> On 14 Nov 2021, at 16:37, Salman Zarrini  wrote:
> 
> Thanks all for your responses.
> 
> @Giovanni:
> Actually, as an example I had considered the FCC Ni in three hypothetical 
> systems with primitive cells with celldm(1) of A0 = 3.511 angstrom, A1 = 
> 1.5*A0 and A2 = 2*A0, respectively. So, only the celldm increases from one 
> system to another one. Please note that they are not super-cell.
> And ran Quantum-Espresso to calculate the density of states for each of the 
> three systems using similar convergence criterias. To avoid any complication 
> regarding the spin, I hypothetically just used the spin--nonpolarized mode in 
> my calculations.
> 
> The results for density of states at the Fermi level (N(E_F)) for each system 
> is as follows:
> A0 :> N(E_F) = 5 
> A1 = 1.5*A0 :> N(E_F) = 33.78
> A2 = 2*A0:> N(E_F) = 159
> 
> And then the ratio of N(E_F) / V where V is the volume of primitive cell of 
> each system is as follows:
> A0 :> N(E_F) / V0 = 0.462
> A1 = 1.5A0 :>  N(E_F) / V1 = 0.924
> A2 = 2*A0:> N(E_F) / V2 = 1.836
> 
> So, comparing the density of states at the Fermi level, which indeed is a 
> critical quantity in some concepts, shows that the N(E_F) / V change from one 
> to another around two times from system A0 to system A1 and foure times from 
> system A0 to system A2. Although the general N(E) / V are not as similar as I 
> plotted them. 
> 
> I understand the chemistry point of view as by enlarging the cell dimension, 
> the overlap of d wave functions decreases so consequently the band width 
> decreases and that increases the N(E_F) ultimately. However, I still have the 
> problem in understanding the unit(s) used for density of states.
> 
> Regards,
> Salman Zarrini
> 
> On Sun, Nov 14, 2021 at 6:23 AM Giovanni Cantele 
> mailto:giovanni.cant...@spin.cnr.it>> wrote:
> Dear Salman,
> 
> if a quantity is extensive, so the larger the volume/mass/size the larger 
> that quantity, it should NOT have Volume^-1 in its units.
> 
> Indeed, the DOS as calculated by Quantum-ESPRESSO is in eV^-1. Let us suppose 
> that you calculate the ground state of a given crystal within its primitive 
> cell and obtain a certain DOS. If you now compute the ground state of the 
> SAME crystal  but with twice the unit cell, the number of electrons doubles 
> and as does the DOS. On the other hand, if you divide both DOSs by the 
> respective unit cell volumes, you’ll get a quantity with Volume^-1 in its 
> units that will be the exactly same for both calculations (provided the 
> convergence of both calculations with respect to the used parameters is the 
> same).
> 
> Giovanni
> 
>> On 13 Nov 2021, at 21:06, Salman Zarrini > <mailto:salman.zarr...@gmail.com>> wrote:
>> 
>> So, that means Quantum-Espresso gives an extensive density of states, right? 
>> if so, then it should have a Volume^-1 in its unit.
>> 
>> Regards,
>> Salman
>> 
>> 
>> On Sat, Nov 13, 2021 at 2:46 PM Stefano Baroni > <mailto:bar...@sissa.it>> wrote:
>> it depends on the volume of the unit cell. once you divide by it, you get an 
>> intensive (volume-independent) quantity. sb
>> 
>> ___
>> Stefano Baroni, Trieste -- http://stefano.baroni.me 
>> <http://stefano.baroni.me/>
>> 
>>> On 13 Nov 2021, at 20:29, Salman Zarrini >> <mailto:salman.zarr...@gmail.com>> wrote:
>>> 
>>> 
>>> 
>>> Dear Giovanni,
>>> 
>>> Thanks for your response.
>>> 
>>> Then, considering the density of states in an electronic system and what 
>>> Quantum-Espresso calculates as the density of states, should we expect to 
>>> have a volume-independent quantity? if I understood you correctly!
>>> 
>>> Regards,
>>> Salman
>>> 
>>> 
>>> On Sat, Nov 13, 2021 at 1:30 PM Giovanni Cantele 
>>> mailto:giovanni.cant...@spin.cnr.it>> wrote:
>>> Dear Salman,
>>> 
>>> Actually, th

Re: [QE-users] Density of states and Volume

2021-11-14 Thread Giovanni Cantele
Dear Salman,

if a quantity is extensive, so the larger the volume/mass/size the larger that 
quantity, it should NOT have Volume^-1 in its units.

Indeed, the DOS as calculated by Quantum-ESPRESSO is in eV^-1. Let us suppose 
that you calculate the ground state of a given crystal within its primitive 
cell and obtain a certain DOS. If you now compute the ground state of the SAME 
crystal  but with twice the unit cell, the number of electrons doubles and as 
does the DOS. On the other hand, if you divide both DOSs by the respective unit 
cell volumes, you’ll get a quantity with Volume^-1 in its units that will be 
the exactly same for both calculations (provided the convergence of both 
calculations with respect to the used parameters is the same).

Giovanni

> On 13 Nov 2021, at 21:06, Salman Zarrini  wrote:
> 
> So, that means Quantum-Espresso gives an extensive density of states, right? 
> if so, then it should have a Volume^-1 in its unit.
> 
> Regards,
> Salman
> 
> 
> On Sat, Nov 13, 2021 at 2:46 PM Stefano Baroni  <mailto:bar...@sissa.it>> wrote:
> it depends on the volume of the unit cell. once you divide by it, you get an 
> intensive (volume-independent) quantity. sb
> 
> ___
> Stefano Baroni, Trieste -- http://stefano.baroni.me 
> <http://stefano.baroni.me/>
> 
>> On 13 Nov 2021, at 20:29, Salman Zarrini > <mailto:salman.zarr...@gmail.com>> wrote:
>> 
>> 
>> 
>> Dear Giovanni,
>> 
>> Thanks for your response.
>> 
>> Then, considering the density of states in an electronic system and what 
>> Quantum-Espresso calculates as the density of states, should we expect to 
>> have a volume-independent quantity? if I understood you correctly!
>> 
>> Regards,
>> Salman
>> 
>> 
>> On Sat, Nov 13, 2021 at 1:30 PM Giovanni Cantele 
>> mailto:giovanni.cant...@spin.cnr.it>> wrote:
>> Dear Salman,
>> 
>> Actually, the two definitions are not mutually exclusive. The first you 
>> speak about, is the density of states per unit volume and, as you correctly 
>> mention, has units Energy^-1 Volume^-1. However, the definition of density 
>> of states a system of electrons and has units Energy^-1:
>> 
>> DOS(E) = sum_i Dirac_delta(E-E_i)
>> 
>> Integral( dE DOS(E) ) = number of electrons
>> 
>> What Quantum-Espresso calculates, is the density of states of the electron 
>> system in the unit cell of a given Bravais lattice (due to periodicity, you 
>> refer to the primitive cell). If you plot it as is, you should give it units 
>> eV^-1. However, you could need the density of states per unit volume. In 
>> that case, you can easily obtain the unit cell volume of your system, divide 
>> the computed density of states by it, and then the resulting 
>> density-of-states-per-unit-volume has units eV^-1 au^-3 (if you express the 
>> volume in au^3).
>> 
>> In this case, if you integrate over the energy, you obtain number of 
>> electrons per unit volume, that is, electron density.
>> 
>> Giovanni
>> 
>> > On 13 Nov 2021, at 19:14, Salman Zarrini > > <mailto:salman.zarr...@gmail.com>> wrote:
>> > 
>> > Dearl all,
>> > 
>> > As the density of states's definition implies, the electronic density of 
>> > states has a unit of "Number of electronic states per Energy per Volume" 
>> > or simply Volume^-1 Energy^-1.  However, the "Volume^-1" is apparently 
>> > missing in the unit of density of states in literatures as well as here in 
>> > manual/tutorials of Quantum-Espresso. So that the Energy^-1 is used as the 
>> > unit for total density of states, atomic site projected density of states 
>> > and orbital projected density of states. 
>> > 
>> > I guess it is just a misunderstanding from my side, so, I would be 
>> > thankful if one could elaborate further on that.
>> > 
>> > Regards,
>> > Salman
>> > ___
>> > Quantum ESPRESSO is supported by MaX (www.max-centre.eu 
>> > <http://www.max-centre.eu/>)
>> > users mailing list users@lists.quantum-espresso.org 
>> > <mailto:users@lists.quantum-espresso.org>
>> > https://lists.quantum-espresso.org/mailman/listinfo/users 
>> > <https://lists.quantum-espresso.org/mailman/listinfo/users>
>> 
>> ___
>> Quantum ESPRESSO is supported by MaX (www.max-centre.eu 
>> <http://www.max-centre.eu/>)
>> users mailing list users@lists.quantum-espresso.org 
>> <ma

Re: [QE-users] Density of states and Volume

2021-11-13 Thread Giovanni Cantele
Dear Salman,

Actually, the two definitions are not mutually exclusive. The first you speak 
about, is the density of states per unit volume and, as you correctly mention, 
has units Energy^-1 Volume^-1. However, the definition of density of states a 
system of electrons and has units Energy^-1:

DOS(E) = sum_i Dirac_delta(E-E_i)

Integral( dE DOS(E) ) = number of electrons

What Quantum-Espresso calculates, is the density of states of the electron 
system in the unit cell of a given Bravais lattice (due to periodicity, you 
refer to the primitive cell). If you plot it as is, you should give it units 
eV^-1. However, you could need the density of states per unit volume. In that 
case, you can easily obtain the unit cell volume of your system, divide the 
computed density of states by it, and then the resulting 
density-of-states-per-unit-volume has units eV^-1 au^-3 (if you express the 
volume in au^3).

In this case, if you integrate over the energy, you obtain number of electrons 
per unit volume, that is, electron density.

Giovanni

> On 13 Nov 2021, at 19:14, Salman Zarrini  wrote:
> 
> Dearl all,
> 
> As the density of states's definition implies, the electronic density of 
> states has a unit of "Number of electronic states per Energy per Volume" or 
> simply Volume^-1 Energy^-1.  However, the "Volume^-1" is apparently missing 
> in the unit of density of states in literatures as well as here in 
> manual/tutorials of Quantum-Espresso. So that the Energy^-1 is used as the 
> unit for total density of states, atomic site projected density of states and 
> orbital projected density of states. 
> 
> I guess it is just a misunderstanding from my side, so, I would be thankful 
> if one could elaborate further on that.
> 
> Regards,
> Salman
> ___
> Quantum ESPRESSO is supported by MaX (www.max-centre.eu)
> users mailing list users@lists.quantum-espresso.org
> https://lists.quantum-espresso.org/mailman/listinfo/users

___
Quantum ESPRESSO is supported by MaX (www.max-centre.eu)
users mailing list users@lists.quantum-espresso.org
https://lists.quantum-espresso.org/mailman/listinfo/users


Re: [QE-users] Inconsistency in electronic band structure of GeP3 monolayer

2021-07-20 Thread Giovanni Cantele
Hi Marcelo,

I do not see an evident reason why you should find a discrepancy. However, by 
looking to Fig. 1b of the first reference and to the side view of you structure 
(as obtained by opening the scf input directly with XcrysDEN), it seems to me 
that there is some difference in the z coordinates. Also, I tried (but I don’t 
know if in the correct way) to calculate "the puckered height between P–P and 
Ge–Ge” as mentioned in the reference. Again, your structure seems to be 
different. Did you relax the structure before making the scf? What is the value 
of “Total force” at the end of the scp output?

Giovanni

PS is there any reason why not trying with PAW pseudo potentials requiring a 
much smaller cutoff?
(this has of course nothing to do with your issue)

> On 20 Jul 2021, at 17:42, Marcelo Albuquerque  wrote:
> 
> Dear users,
> 
> I'm trying to calculate the electronic band structure of GeP3, but I haven't 
> been able to reproduce it, as shown in both references below. 
> 
> a) Bowen Zeng et al 2018 J. Phys. D: Appl. Phys. 51 235302;
> b) Yu Jing, et al, Nano Lett. 2017, 17, 3, 1833–1838.
> 
> BTW, they were computed using VASP code, and I don't understand why I'm 
> facing such an issue. I have already tried many different approaches to 
> calculate this band structure, but it simply doesn't work.
> 
> Here follows the inputs I used for the scf and the bands calculations. 
> 
> A) SCF
> 
> prefix='gep',
> calculation='scf'
> nstep=0
> tstress=.false.
> tprnfor= .true.
> etot_conv_thr = 1.0D-5
> forc_conv_thr = 1.0D-7
> pseudo_dir = '/home/malbuquerque/pseudos/',
> outdir='./'
>  /
>  
> ibrav=  4,
> a=  6.982664008,
> c = 16.0,
> nat= 8,
>  ntyp= 2,
> ecutwfc = 120,
>  nbnd = 40
>vdw_corr='grimme-d2'
> occupations='fixed',
> /
> 
> electron_maxstep = 400
> conv_thr = 1.0d-12
> diagonalization='david'
> mixing_mode='plain'
> mixing_beta= 0.7
>  /
> 
> ion_dynamics='bfgs'
> /
> 
>   cell_dynamics = 'bfgs',
>press_conv_thr = 0.5,
>cell_factor = 2.0,
>cell_dofree = '2Dxy',
> /
> ATOMIC_SPECIES
> Ge  72.640   Ge.pbe-hgh.UPF
> P   30.974   P.pbe-hgh.UPF
> ATOMIC_POSITIONS crystal
> Ge   0.836927687   0.360474777   0.413486925
> Ge   0.503586578   0.693798663   0.359352022
> P0.325445130   0.871903831   0.425075713
> P0.859830506   0.871910609   0.425081984
> P0.325454709   0.337550097   0.425072126
> P0.014988446   0.182362365   0.347653353
> P0.014997809   0.716639703   0.347643675
> P0.480752135   0.182372957   0.347659202
> K_POINTS automatic
> 13 13 1 0 0 0
> 
> 
> B) BANDS
> 
> prefix='gep',
> calculation='bands'
> nstep=0
> tstress=.false.
> tprnfor= .true.
> etot_conv_thr = 1.0D-5
> forc_conv_thr = 1.0D-7
> pseudo_dir = '/home/malbuquerque/pseudos/',
> outdir='./'
>  /
> 
> ibrav=  4,
> a=  6.982664008,
> c = 16.0,
> nat= 8,
>  ntyp= 2,
> ecutwfc = 120,
>  nbnd = 40
>vdw_corr='grimme-d2'
> occupations='fixed',
>  /
>  
> electron_maxstep = 400
> conv_thr = 1.0d-12
> diagonalization='david'
> mixing_mode='plain'
> mixing_beta= 0.7
>  /
> 
> ion_dynamics='bfgs'
> /
> 
>   cell_dynamics = 'bfgs',
>press_conv_thr = 0.5,
>cell_factor = 2.0,
>cell_dofree = '2Dxy',
> /
> ATOMIC_SPECIES
> Ge  72.640   Ge.pbe-hgh.UPF
> P   30.974   P.pbe-hgh.UPF
> ATOMIC_POSITIONS (crystal)
> Ge   0.836927687   0.360474777   0.413486925
> Ge   0.503586578   0.693798663   0.359352022
> P0.325445130   0.871903831   0.425075713
> P0.859830506   0.871910609   0.425081984
> P0.325454709   0.337550097   0.425072126
> P0.014988446   0.182362365   0.347653353
> P0.014997809   0.716639703   0.347643675
> P0.480752135   0.182372957   0.347659202
> K_POINTS crystal_b
> 4
> 0.00.00.0  30 !! Gamma
> 0.30.30.0  30 !! K
> 0.50.00.0  30 !! M
> 0.00.00.0  30 !! Gamma
> 
> 
> 
> In fact, I used the the QE versions6.7, 6.6 and 6.4.1, which gave me the same 
> strange result. I'm also sending a PNG file showing the obtained bands. The 
> red line is the VBM, and the blue one is the CBM (or it was supposed to be). 
> What is weird is that I was expecting (according to the aforementioned 
> reference) that the blue band was located a little bit above where it is in 
> this figure. Also the orange one should be a little bit above, close to the 
> VBM.
> 
> Could anyone help me or give me some advice about this problem. I really 
> appreciate it in advance.
> 
> 
> With all the best regards,
> Marcelo Albuquerque
> Ph.D. Candidate
> Institute of Physics
> Fluminense Federal University
> Niterói/RJ - Brazil
> ___
> Quantum ESPRESSO is supported by MaX (www.max-centre.eu)
> users mailing list 

Re: [QE-users] Help on QE + GPU + OpenMP compilation

2021-04-16 Thread Giovanni Cantele
With Paolo’s environmental variables, combined with Pietro’s suggestion 
--with-cuda=yes --enable-cuda-env-check=no (even though I cannot understand why 
the CUDA environment check fails if the CUDA libraries are there) everything 
works fine! I’m able to run the development version on GPUs, and see the 
related message in the output file, together with that associated with threads.

I thank all of you for your valuable support!

Giovanni
 

> On 16 Apr 2021, at 11:33, Sergey Lisenkov  wrote:
> 
>  
>  
> 15.04.2021, 15:45, "Paolo Giannozzi"  <mailto:p.gianno...@gmail.com>>:
> On Thu, Apr 15, 2021 at 2:01 PM Giovanni Cantele 
> mailto:giovanni.cant...@spin.cnr.it>> wrote:
>  
> I’m trying to compile the latest QE version
>  
> "latest stable" or "latest development"? GPU support is work in progress (or 
> maybe "in regress"): don't count on the stable version to support the latest 
> compiler.
>  
> I can tell you for sure that with a correct nvidia hpc_sdk 21.3 installation 
> + all needed environment variables defined + all needed configure options (+ 
> MKL libraries: I don't trust too much blas and lapack from the nvidia 
> compiler) the latest development version compiles and works.
> blas from nvidia compiler is actually OpenBLAS now, and scalapack is 
> distributed as well. On Power9, for example,  there are not much choice - no 
> MKL, of course,  ESSL is missing some lapack functions needed for QE, plus 
> FFT from ESSL library (-D__LINUX_ESSL) crashes on several runs, while FFTW 
> works.
>  
> Sergey
>  
> Paolo
> --
> Paolo Giannozzi, Dip. Scienze Matematiche Informatiche e Fisiche,
> Univ. Udine, via delle Scienze 206, 33100 Udine, Italy
> Phone +39-0432-558216, fax +39-0432-558222
>  
> ,
> ___
> Quantum ESPRESSO is supported by MaX (www.max-centre.eu 
> <https://www.max-centre.eu/>)
> users mailing list users@lists.quantum-espresso.org 
> <mailto:users@lists.quantum-espresso.org>
> https://lists.quantum-espresso.org/mailman/listinfo/users 
> <https://lists.quantum-espresso.org/mailman/listinfo/users>___
> Quantum ESPRESSO is supported by MaX (www.max-centre.eu 
> <http://www.max-centre.eu/>)
> users mailing list users@lists.quantum-espresso.org 
> <mailto:users@lists.quantum-espresso.org>
> https://lists.quantum-espresso.org/mailman/listinfo/users 
> <https://lists.quantum-espresso.org/mailman/listinfo/users>
___
Quantum ESPRESSO is supported by MaX (www.max-centre.eu)
users mailing list users@lists.quantum-espresso.org
https://lists.quantum-espresso.org/mailman/listinfo/users

Re: [QE-users] Help on QE + GPU + OpenMP compilation

2021-04-16 Thread Giovanni Cantele
Thank you, I’m working with your input variables and the latest DEVELOPMENT 
version downloaded yesterday.

There should be something wrong with configure. The error I’m facing is
configure: error: in `/home/cantele/q-e-develop':
configure: error: Couldn't find libcuda
See `config.log' for more details

If I look within config.log it tries to make the following compilation:
pgcc -o conftest -g -O2   conftest.c -lcuda   
-L/opt/nvidia/hpc_sdk/Linux_x86_64/21.3/cuda/11.2/lib64 -lcuda -lcudart 
-lcublas -lcufft  >&5

And indeed, at the end, the summary of the variables reports, among the others:
CUDA_LDLIBS=' -L/opt/nvidia/hpc_sdk/Linux_x86_64/21.3/cuda/11.2/lib64 -lcuda 
-lcudart -lcublas -lcufft’

However, /opt/nvidia/hpc_sdk/Linux_x86_64/21.3/cuda/11.2/lib64 does NOT contain 
libcufft, libcublas but only libcudart. The former are within 
/opt/nvidia/hpc_sdk/Linux_x86_64/21.3/REDIST/math_libs/11.2/lib64/
but it seems that configure does not look through all the directories in 
LD_LIBRARY_PATH.

Is that possible?

Thanks in any case, I’ll let you know if I manage to solve those issues.

Giovanni

> On 15 Apr 2021, at 22:22, Paolo Giannozzi  wrote:
> 
> This is what I have used, with some success. If you do not have MKL 
> libraries, you should get
>BLAS_LIBS=-llapack -lblas 
> or something like that
> 
> Paolo
> 
> On Thu, Apr 15, 2021 at 5:27 PM Giovanni Cantele 
> mailto:giovanni.cant...@spin.cnr.it>> wrote:
> I downloaded the latest development version. Could you share the needed 
> configure options and environment variables that allowed you to get a clean 
> compilation?
> 
> Thank you again,
> 
> Giovanni
> 
>> On 15 Apr 2021, at 14:50, Giovanni Cantele > <mailto:giovanni.cant...@spin.cnr.it>> wrote:
>> 
>> It was the lates stable (qe-6.7-ReleasePack.tgz).
>> 
>> Where can I find the all needed environment variables?
>> 
>> Thanks,
>> 
>> Giovanni
>> 
>>> On 15 Apr 2021, at 14:44, Paolo Giannozzi >> <mailto:p.gianno...@gmail.com>> wrote:
>>> 
>>> On Thu, Apr 15, 2021 at 2:01 PM Giovanni Cantele 
>>> mailto:giovanni.cant...@spin.cnr.it>> wrote:
>>> 
>>> I’m trying to compile the latest QE version
>>> 
>>> "latest stable" or "latest development"? GPU support is work in progress 
>>> (or maybe "in regress"): don't count on the stable version to support the 
>>> latest compiler. 
>>>  
>>> I can tell you for sure that with a correct nvidia hpc_sdk 21.3 
>>> installation + all needed environment variables defined + all needed 
>>> configure options (+ MKL libraries: I don't trust too much blas and lapack 
>>> from the nvidia compiler) the latest development version compiles and works.
>>> 
>>> Paolo
>>> -- 
>>> Paolo Giannozzi, Dip. Scienze Matematiche Informatiche e Fisiche,
>>> Univ. Udine, via delle Scienze 206, 33100 Udine, Italy
>>> Phone +39-0432-558216, fax +39-0432-558222
>>> 
>>> ___
>>> Quantum ESPRESSO is supported by MaX (www.max-centre.eu 
>>> <http://www.max-centre.eu/>)
>>> users mailing list users@lists.quantum-espresso.org 
>>> <mailto:users@lists.quantum-espresso.org>
>>> https://lists.quantum-espresso.org/mailman/listinfo/users 
>>> <https://lists.quantum-espresso.org/mailman/listinfo/users>
>> ___
>> Quantum ESPRESSO is supported by MaX (www.max-centre.eu 
>> <http://www.max-centre.eu/>)
>> users mailing list users@lists.quantum-espresso.org 
>> <mailto:users@lists.quantum-espresso.org>
>> https://lists.quantum-espresso.org/mailman/listinfo/users 
>> <https://lists.quantum-espresso.org/mailman/listinfo/users>
> ___
> Quantum ESPRESSO is supported by MaX (www.max-centre.eu 
> <http://www.max-centre.eu/>)
> users mailing list users@lists.quantum-espresso.org 
> <mailto:users@lists.quantum-espresso.org>
> https://lists.quantum-espresso.org/mailman/listinfo/users 
> <https://lists.quantum-espresso.org/mailman/listinfo/users>
> 
> -- 
> Paolo Giannozzi, Dip. Scienze Matematiche Informatiche e Fisiche,
> Univ. Udine, via delle Scienze 206, 33100 Udine, Italy
> Phone +39-0432-558216, fax +39-0432-558222
> 
> ___
> Quantum ESPRESSO is supported by MaX (www.max-centre.eu)
> users mailing list users@lists.quantum-espresso.org
> https://lists.quantum-espresso.org/mailman/listinfo/users

___
Quantum ESPRESSO is supported by MaX (www.max-centre.eu)
users mailing list users@lists.quantum-espresso.org
https://lists.quantum-espresso.org/mailman/listinfo/users

Re: [QE-users] Help on QE + GPU + OpenMP compilation

2021-04-15 Thread Giovanni Cantele
I downloaded the latest development version. Could you share the needed 
configure options and environment variables that allowed you to get a clean 
compilation?

Thank you again,

Giovanni

> On 15 Apr 2021, at 14:50, Giovanni Cantele  
> wrote:
> 
> It was the lates stable (qe-6.7-ReleasePack.tgz).
> 
> Where can I find the all needed environment variables?
> 
> Thanks,
> 
> Giovanni
> 
>> On 15 Apr 2021, at 14:44, Paolo Giannozzi > <mailto:p.gianno...@gmail.com>> wrote:
>> 
>> On Thu, Apr 15, 2021 at 2:01 PM Giovanni Cantele 
>> mailto:giovanni.cant...@spin.cnr.it>> wrote:
>> 
>> I’m trying to compile the latest QE version
>> 
>> "latest stable" or "latest development"? GPU support is work in progress (or 
>> maybe "in regress"): don't count on the stable version to support the latest 
>> compiler. 
>>  
>> I can tell you for sure that with a correct nvidia hpc_sdk 21.3 installation 
>> + all needed environment variables defined + all needed configure options (+ 
>> MKL libraries: I don't trust too much blas and lapack from the nvidia 
>> compiler) the latest development version compiles and works.
>> 
>> Paolo
>> -- 
>> Paolo Giannozzi, Dip. Scienze Matematiche Informatiche e Fisiche,
>> Univ. Udine, via delle Scienze 206, 33100 Udine, Italy
>> Phone +39-0432-558216, fax +39-0432-558222
>> 
>> ___
>> Quantum ESPRESSO is supported by MaX (www.max-centre.eu 
>> <http://www.max-centre.eu/>)
>> users mailing list users@lists.quantum-espresso.org 
>> <mailto:users@lists.quantum-espresso.org>
>> https://lists.quantum-espresso.org/mailman/listinfo/users 
>> <https://lists.quantum-espresso.org/mailman/listinfo/users>
> ___
> Quantum ESPRESSO is supported by MaX (www.max-centre.eu)
> users mailing list users@lists.quantum-espresso.org
> https://lists.quantum-espresso.org/mailman/listinfo/users

___
Quantum ESPRESSO is supported by MaX (www.max-centre.eu)
users mailing list users@lists.quantum-espresso.org
https://lists.quantum-espresso.org/mailman/listinfo/users

Re: [QE-users] Help on QE + GPU + OpenMP compilation

2021-04-15 Thread Giovanni Cantele
It was the lates stable (qe-6.7-ReleasePack.tgz).

Where can I find the all needed environment variables?

Thanks,

Giovanni

> On 15 Apr 2021, at 14:44, Paolo Giannozzi  wrote:
> 
> On Thu, Apr 15, 2021 at 2:01 PM Giovanni Cantele 
> mailto:giovanni.cant...@spin.cnr.it>> wrote:
> 
> I’m trying to compile the latest QE version
> 
> "latest stable" or "latest development"? GPU support is work in progress (or 
> maybe "in regress"): don't count on the stable version to support the latest 
> compiler. 
>  
> I can tell you for sure that with a correct nvidia hpc_sdk 21.3 installation 
> + all needed environment variables defined + all needed configure options (+ 
> MKL libraries: I don't trust too much blas and lapack from the nvidia 
> compiler) the latest development version compiles and works.
> 
> Paolo
> -- 
> Paolo Giannozzi, Dip. Scienze Matematiche Informatiche e Fisiche,
> Univ. Udine, via delle Scienze 206, 33100 Udine, Italy
> Phone +39-0432-558216, fax +39-0432-558222
> 
> ___
> Quantum ESPRESSO is supported by MaX (www.max-centre.eu)
> users mailing list users@lists.quantum-espresso.org
> https://lists.quantum-espresso.org/mailman/listinfo/users

___
Quantum ESPRESSO is supported by MaX (www.max-centre.eu)
users mailing list users@lists.quantum-espresso.org
https://lists.quantum-espresso.org/mailman/listinfo/users

[QE-users] Help on QE + GPU + OpenMP compilation

2021-04-15 Thread Giovanni Cantele
Hi all,I’m trying to compile the latest QE version with OpenMP and GPU support. The latest NVIDIA hpc package is in use (nvidia/hpc_sdk/Linux_x86_64/21.3). 4xTesla V100 SXM2 32GB are mounted on the computing node.There are some points that are not clear to me and I would be grateful if someone might help me in this respect:i) configure commandexport PATH=/opt/nvidia/hpc_sdk/Linux_x86_64/21.3/comm_libs/openmpi/openmpi-3.1.5/bin/:/opt/nvidia/hpc_sdk/Linux_x86_64/21.3/compilers/bin/:$PATH./install/configure FC=nvfortran CC=nvcc --enable-openmp --with-cuda=/usr/local/cuda --with-cuda-cc=70 --with-cuda-runtime=11.2 LIBDIRS=/opt/nvidia/hpc_sdk/Linux_x86_64/21.3/compilers/lib/ /opt/nvidia/hpc_sdk/Linux_x86_64/21.3/math_libs/11.2/targets/x86_64-linux/lib/stubs /opt/nvidia/hpc_sdk/Linux_x86_64/21.3/comm_libs/openmpi/openmpi-3.1.5/lib/ /opt/nvidia/hpc_sdk/Linux_x86_64/21.3/comm_libs/mpi/lib/I’ve given the needed paths and libraries that should suffice to a proper installation with CUDA, lapack, blas and so on. The following compilation does work. However, I find something strange inmake.inc :LAPACK_LIBS    = -L/opt/nvidia/hpc_sdk/Linux_x86_64/21.3/comm_libs/openmpi/openmpi-3.1.5/lib/ -llapack  -lblasThe -L…. directory, among those provided through LIBDIRS, is NOT the one containing lapack and blas libraries, so although the subsequent compilation seems to work, the link as above seems to be wrong (it shouldn’t work!). Is that normal? For example, it seems that a library containing dgeem is not found. However, /opt/nvidia/hpc_sdk/Linux_x86_64/21.3/compilers/lib/  contains both libblas.a and liblapack.a .The install/config.log is attached for further checks.ii) CUDA support seems to have been correctly included. I tried to run pw.x and it seems to work. Also, using nvidia-smi, I see that the pw.x processes are running on the GPUs. However, while on Marconi100 the output file contains a section like    GPU acceleration is ACTIVE.     GPU used by master process:        Device Number: 0        Device name: Tesla V100-SXM2-16GB        Compute capability : 70        Ratio of single to double precision performance  : 2        Memory Clock Rate (KHz): 877000        Memory Bus Width (bits): 4096        Peak Memory Bandwidth (GB/s): 898.05, I do not find any message like this in my output. Does it mean that the GPU support has some troubles?iii) last point, it seems that OpenMP support has not been included. I cannot find any indication of that either in config.log or in make.inc (also attached to this message), despite —enable-openmp was specified to the configure command. I also do not find any information in the output concerning the number of threads. Is there anything else I should specify at compiling / running time ?I’m very sorry for overburdening you with so many questions, but any help would be very much appreciated.Thanks,    Giovanni

config.log
Description: Binary data


make.inc
Description: Binary data

-- Giovanni Cantele, PhDCNR-SPINc/o Dipartimento di FisicaUniversita' di Napoli "Federico II"Complesso Universitario M. S. Angelo - Ed. 6Via Cintia, I-80126, Napoli, Italye-mail: giovanni.cant...@spin.cnr.itPhone: +39 081 676910Skype contact: giocan74ResearcherID: http://www.researcherid.com/rid/A-1951-2009Web page: https://sites.google.com/view/giovanni-cantele/home

___
Quantum ESPRESSO is supported by MaX (www.max-centre.eu)
users mailing list users@lists.quantum-espresso.org
https://lists.quantum-espresso.org/mailman/listinfo/users

Re: [QE-users] Structure with symmetry

2021-03-18 Thread Giovanni Cantele
maybe you could provide the input file for further check
Giovanni

> On 18 Mar 2021, at 14:34, Offermans Willem  wrote:
> 
> Hi Giovanni and Quantum Espresso friends,
> 
> 
> Thank you for your response. It is very much appreciated.
> 
> You are right, by specifying the unit cell in the following
> way, the warning message disappeared.
> 
> 
> ...
>ibrav= 8
>celldm(1)= 10.6279
>celldm(2)= 1.5
>celldm(3)= 4.5961940777125589086
> …
> 
> 
> Though, I specified the cell dimensions in Bohr units, but this is 
> just a detail for this communication.
> 
> As bonus, I can now find the following in the output:
> 
> 
> ...
>  point group C_i (-1)   
>  there are  2 classes
>  the character table:
> 
>E i
> A_g1.00  1.00
> A_u1.00 -1.00
> 
>  the symmetry operations in each class and the name of the first element:
> 
>  E1
>   identity   
>  i2
>   inversion  
> …
> 
> 
> I wonder whether this is compatible to the following output, obtained in the 
> old fashioned way.
> 
> 
> ...
> Found symmetry operation: I + ( -0.5000  0. -0.)
>  This is a supercell, fractional translations are disabled
>  Message from routine find_sym:
>  Not a group! Trying with lower acceptance parameter...
>  Message from routine find_sym:
>  Symmetry operations form a group
> …
> 
> 
> The last line says: "Symmetry operations form a group”.
> Great, which group is it?
> 
> 
> 
> Met vriendelijke groeten,
> Mit freundlichen Grüßen,
> With kind regards,
> 
> 
> Willem Offermans
> Researcher Electrocatalysis SCT
> VITO NV | Boeretang 200 | 2400 Mol
> Phone:+32(0)14335263 Mobile:+32(0)492182073 
> 
> willem.offerm...@vito.be <mailto:willem.offerm...@vito.be>
> 
> 
>> On 18 Mar 2021, at 10:13, Giovanni Cantele > <mailto:giovanni.cant...@spin.cnr.it>> wrote:
>> 
>> Here, a list of the ibrav values that you can choose
>> https://www.quantum-espresso.org/Doc/INPUT_PW.html#idm200 
>> <https://eur02.safelinks.protection.outlook.com/?url=https%3A%2F%2Fwww.quantum-espresso.org%2FDoc%2FINPUT_PW.html%23idm200=04%7C01%7C%7C7db3d1168d674e9bc47b08d8e9ee7e1a%7C9e2777ed82374ab992782c144d6f6da3%7C0%7C1%7C637516557853981073%7CUnknown%7CTWFpbGZsb3d8eyJWIjoiMC4wLjAwMDAiLCJQIjoiV2luMzIiLCJBTiI6Ik1haWwiLCJXVCI6Mn0%3D%7C3000=6NM1WCXt2ogavB5wbKfDwv1wmeybmvhrmyyh2PPpEnM%3D=0>
>> 
>> It seems that you have a simple orthorhombic lattice, that should correspond 
>> to ibrav = 8.
>> Than you can specify the variables
>> a
>> b
>> c
>> as crystal axes lengths in Angstrom units.
>> 
>> Giovanni
>> 
>> 
>>> On 18 Mar 2021, at 10:03, Offermans Willem >> <mailto:willem.offerm...@vito.be>> wrote:
>>> 
>>> 
>>> Dear Quantum Espresso friends,
>>> 
>>> In most of my calculations, I’m dealing with metallic slabs, representing
>>> surfaces, derived from fcc metals, such as (221) or (533).
>>> 
>>> In these calculations, I’m dealing with structures with symmetry.
>>> The single symmetry element is an inversion center. S_2 is the Schönfließ 
>>> symbol,
>>> if I recall correctly. 
>>> 
>>> In the output file of pw.x, I encounter the following note:
>>> 
>>> 
>>> ...
>>>  Message from routine setup:
>>>  using ibrav=0 with symmetry is DISCOURAGED, use correct ibrav instead
>>> …
>>> 
>>> 
>>> The unitcell is defined as:
>>> 
>>> 
>>> ...
>>>  celldm(1)=  10.627880  celldm(2)=   0.00  celldm(3)=   0.00
>>>  celldm(4)=   0.00  celldm(5)=   0.00  celldm(6)=   0.00
>>> 
>>>  crystal axes: (cart. coord. in units of alat)
>>>a(1) = (   1.00   0.00   0.00 )  
>>>a(2) = (   0.00   1.50   0.00 )  
>>>a(3) = (   0.00   0.00   4.596194 )  
>>> …
>>> 
>>> 
>>> I would like to follow the advice to use the correct ibrav, but I need some 
>>> help
>>> to determine the correct ibrav. 
>>> 
>>> Can someone help me to determine the correct Ibrav?
>>> 
>>> 
>>> 
>>> 
>>> 
>>> Met vriendelijke groeten,
>>> Mit freundlichen Grüßen,
>>> With k

Re: [QE-users] Structure with symmetry

2021-03-18 Thread Giovanni Cantele
Here, a list of the ibrav values that you can choose
https://www.quantum-espresso.org/Doc/INPUT_PW.html#idm200 
<https://www.quantum-espresso.org/Doc/INPUT_PW.html#idm200>

It seems that you have a simple orthorhombic lattice, that should correspond to 
ibrav = 8.
Than you can specify the variables
a
b
c
as crystal axes lengths in Angstrom units.

Giovanni


> On 18 Mar 2021, at 10:03, Offermans Willem  wrote:
> 
> 
> Dear Quantum Espresso friends,
> 
> In most of my calculations, I’m dealing with metallic slabs, representing
> surfaces, derived from fcc metals, such as (221) or (533).
> 
> In these calculations, I’m dealing with structures with symmetry.
> The single symmetry element is an inversion center. S_2 is the Schönfließ 
> symbol,
> if I recall correctly. 
> 
> In the output file of pw.x, I encounter the following note:
> 
> 
> ...
>  Message from routine setup:
>  using ibrav=0 with symmetry is DISCOURAGED, use correct ibrav instead
> …
> 
> 
> The unitcell is defined as:
> 
> 
> ...
>  celldm(1)=  10.627880  celldm(2)=   0.00  celldm(3)=   0.00
>  celldm(4)=   0.00  celldm(5)=   0.00  celldm(6)=   0.00
> 
>  crystal axes: (cart. coord. in units of alat)
>a(1) = (   1.00   0.00   0.00 )  
>a(2) = (   0.00   1.50   0.00 )  
>a(3) = (   0.00   0.00   4.596194 )  
> …
> 
> 
> I would like to follow the advice to use the correct ibrav, but I need some 
> help
> to determine the correct ibrav. 
> 
> Can someone help me to determine the correct Ibrav?
> 
> 
> 
> 
> 
> Met vriendelijke groeten,
> Mit freundlichen Grüßen,
> With kind regards,
> 
> 
> Willem Offermans
> Researcher Electrocatalysis SCT
> VITO NV | Boeretang 200 | 2400 Mol
> Phone:+32(0)14335263 Mobile:+32(0)492182073 
> 
> willem.offerm...@vito.be <mailto:willem.offerm...@vito.be>
> 
> 
> _______
> Quantum ESPRESSO is supported by MaX (www.max-centre.eu)
> users mailing list users@lists.quantum-espresso.org
> https://lists.quantum-espresso.org/mailman/listinfo/users

-- 

Giovanni Cantele, PhD
CNR-SPIN
c/o Dipartimento di Fisica
Universita' di Napoli "Federico II"
Complesso Universitario M. S. Angelo - Ed. 6
Via Cintia, I-80126, Napoli, Italy
e-mail: giovanni.cant...@spin.cnr.it
Phone: +39 081 676910
Skype contact: giocan74

ResearcherID: http://www.researcherid.com/rid/A-1951-2009
Web page: https://sites.google.com/view/giovanni-cantele/home

___
Quantum ESPRESSO is supported by MaX (www.max-centre.eu)
users mailing list users@lists.quantum-espresso.org
https://lists.quantum-espresso.org/mailman/listinfo/users

[QE-users] info QE-GPU on Marconi100 infrastructures

2020-07-09 Thread Giovanni Cantele
Dear all,

I’m writing to ask whether anybody is (successfully) trying to run qe-gpu on 
the new CINECA infrastructure MARCONI100.

While at the beginning thinks seemed to work fine, in the last days there are 
lots of instabilities that I think are partly 
related to the system instability.

However, I’m also facing other maybe code-related issues. For example, in some 
runs I’m not managing to carry out
a restart from a (cleanly exited) previous run.
In particular, in one of the last runs I found this error, obtained when a 
freshly started run was trying to write the needed
files to outdir:
ERROR(FoX)
xml_AddCharacters: Invalid character in chars
FORTRAN STOP

Did anybody experience such kind of errors? I would like just to be sure that 
there are not known issues on the 6.5 version of qe-gpu,
since I suspect that most of the problem I’m facing are related to the computer 
rather than the code.

Thank you in advance,

Giovanni



 
-- 

Giovanni Cantele, PhD

CNR-SPIN
c/o Dipartimento di Fisica
Universita' di Napoli "Federico II"
Complesso Universitario M. S. Angelo - Ed. 6
Via Cintia, I-80126, Napoli, Italy

e-mail: giovanni.cant...@spin.cnr.it
gcant...@gmail.com
Phone: +39 081 676910
Skype contact: giocan74
Web page: https://sites.google.com/view/giovanni-cantele

___
Quantum ESPRESSO is supported by MaX (www.max-centre.eu/quantum-espresso)
users mailing list users@lists.quantum-espresso.org
https://lists.quantum-espresso.org/mailman/listinfo/users

Re: [QE-users] Doping

2020-07-09 Thread Giovanni Cantele
It is possible to simulate a doped system. However, the only way to do it is 
using a supercell approach.

Indeed, doping is usually meant as a “random” distribution of non-interacting 
(or almost non-interacting)
impurities. As such, you should choose a sufficiently large supercell (for 
example, try 2x2x2 or 3x3x3 for
a three-dimensional cubic lattice), to prevent the impurity defects in 
neighbour supercells from interacting.

Consider that if in the bulk unit cell you sample the Brillouin zone with a n x 
n x n k-point grid and study
the impurity with an m x m x m supercell, the latter requires, to obtain the 
same accuracy with
respect to the sampling, a n/m x n/m x n/m k-point grid (for example, if you 
use a 6x6x6 k-point grid in the
1x1x1 bulk unit cell, and study the impurity within a 2x2x2 supercell, for the 
latter you sample the BZ with
a 3x3x3 k-point grid).

This being said, the larger the supercell, the larger the computational 
resources (also in terms of RAM)
you need.

If you use, as you say, a “unit cell structure” (I think you mean the same unit 
cell as the bulk crystal),
you are simulating a possibly interacting distribution of impurities, with 
concentrations usually much larger
that the experimental ones and with defect levels that possibly exhibit 
dispersion (“defect bands”).

Giovanni

PS people usually do sign their posts in the forum, together with their 
affiliation.

> On 8 Jul 2020, at 20:32, Neelam Swarnkar  wrote:
> 
> Dear expert and all
> 
> Doping of an atom is possible in a unit cell structure, because supercell 
> taking a lot RAM for calculation.
> 
> If it is possible , than please let me know.
> 
> Thanks in advance
> ___
> Quantum ESPRESSO is supported by MaX (www.max-centre.eu/quantum-espresso)
> users mailing list users@lists.quantum-espresso.org
> https://lists.quantum-espresso.org/mailman/listinfo/users

-- 

Giovanni Cantele, PhD

CNR-SPIN
c/o Dipartimento di Fisica
Universita' di Napoli "Federico II"
Complesso Universitario M. S. Angelo - Ed. 6
Via Cintia, I-80126, Napoli, Italy

e-mail: giovanni.cant...@spin.cnr.it
gcant...@gmail.com
Phone: +39 081 676910
Skype contact: giocan74
Web page: https://sites.google.com/view/giovanni-cantele

___
Quantum ESPRESSO is supported by MaX (www.max-centre.eu/quantum-espresso)
users mailing list users@lists.quantum-espresso.org
https://lists.quantum-espresso.org/mailman/listinfo/users

Re: [QE-users] Dipole moment of the CO2 molecule

2020-05-26 Thread Giovanni Cantele
An issue with your input is that you place the point where the external 
saw-tooth potential
has a discontinuity in its derivative at emaxpos = 0.5 (it is in crystal units, 
so because edir = 3,
it means that the potential depends on z and the discontinuity is at (0,0,a/2)).
However, the CO2 molecule is placed along the x axis at fixed y = a/2 and z = 
a/2. As such,
the discontinuity, which is unphysical since in serves to compensate the 
spurious dipole field that
would arise as a consequence of the periodic boundary conditions, is located in 
a region where the
charge density is not zero. emaxpos should be set in such a way that the 
discontinuity il located
in the vacuum, in the region where the charge density is zero or in any case 
very small. In your setup
it seems that a good choice would be emaxpos=0.

Giovanni

PS since you’re studying an isolated molecule, provided the size of your 
supercell is sufficiently large,
the eigenvalues should exhibit no dependence on k. As a consequence, in this 
case, a 4x4x4 sampling
of the Brillouin zone, should provide results equivalent to a gamma only 
sampling. So, you can switch to
KPOINTS automatic
1 1 10 0 0
or, even better,
KPOINTS gamma
to make your calculation faster while keeping the same accuracy.

> On 27 May 2020, at 03:42, ENDALE ABEBE  wrote:
> 
> Dear Experts, users and all
> 
> I found the dipoles of CO2 calculated by Quantum ESPRESSO as :
> 
> Computed dipole along edir(3) :
> Elec. dipole 0.3112 Ry au, 0.7909 Debye
> Ion. dipole 0.8137 Ry au, 2.0683 Debye
> Dipole 41.9812 Ry au, 106.7055 Debye
> Dipole field 0.5025 Ry au, 
> 
> I assumed the third value is the sum of the electronic and ionic 
> contributions.
> The input and output files are attached herewith.
> Since CO2 is non-polar molecule (with polar bonds), shouldn't the total 
> dipole moment be zero?
> 
> -- 
> Endale Abebe
> Program coordinator and Lecturer
> Faculty of Materials Science and Engineering
> Jimma Institute of Technology
> Jimma University
> P.O.Box 378, Jimma, Ethiopia
> Mobile: +251921381598
> ___
> Quantum ESPRESSO is supported by MaX (www.max-centre.eu/quantum-espresso)
> users mailing list users@lists.quantum-espresso.org
> https://lists.quantum-espresso.org/mailman/listinfo/users

-- 

Giovanni Cantele, PhD

CNR-SPIN
c/o Dipartimento di Fisica
Universita' di Napoli "Federico II"
Complesso Universitario M. S. Angelo - Ed. 6
Via Cintia, I-80126, Napoli, Italy

e-mail: giovanni.cant...@spin.cnr.it
gcant...@gmail.com
Phone: +39 081 676910
Skype contact: giocan74
Web page: https://sites.google.com/view/giovanni-cantele

___
Quantum ESPRESSO is supported by MaX (www.max-centre.eu/quantum-espresso)
users mailing list users@lists.quantum-espresso.org
https://lists.quantum-espresso.org/mailman/listinfo/users

Re: [QE-users] POSCAR to QE input

2020-05-06 Thread Giovanni Cantele
Dear Poonam,

this POSCAR section
>1. 
>  5.9487492159093023   -0.   -0.
> -2.97437460795465115.15176794171171260.
> -0.0.   11.7211034581915818
means that the cell parameters are in Angstrom.
So, it can be translated into QE input directives as:
CELL_PARAMETERS { angstrom }
 5.9487492159093023   -0.   -0.
-2.97437460795465115.15176794171171260.
-0.0.   11.7211034581915818

You can explore whether it is better to use, in place of the CELL_PARAMETERS 
card,
the variables designed to build up the crystal unit cell, that is, ibrav, a, b, 
c, cosAB, cosAC, cosBC .

As far as the atomic positions are concerned, POSCAR “Direct” means crystal 
coordinates,
as such you should add to the QE input file a card
ATOMIC_POSITIONS { crystal }

followed by the list of the atomic coordinates, as in the POSCAR. However, QE 
requires each atomic coordinate 
triplet (xat, yat, zat) be preceded by its atomic symbol. In your POSCAR the 
first 12 atoms are Fe, the next 12 S,
so you will have 12 lines looking like
Fe x  y  z
and 12 lines looking like
S  x  y  z


Of course, this is the “translation” of the only POSCAR file. You then need to 
build up all the rest of the QE input file,
concerning the CONTROL, SYSTEM and ELECTRONS name lists, as well as the 
ATOMIC_SPECIES card. For these,
you might refer to the QE user guide and to the many examples included therein.

Giovanni
-- 

Giovanni Cantele, PhD

CNR-SPIN
c/o Dipartimento di Fisica
Universita' di Napoli "Federico II"
Complesso Universitario M. S. Angelo - Ed. 6
Via Cintia, I-80126, Napoli, Italy

e-mail: giovanni.cant...@spin.cnr.it
gcant...@gmail.com
Phone: +39 081 676910
Skype contact: giocan74
Web page: https://sites.google.com/view/giovanni-cantele

> On 6 May 2020, at 07:13, Poonam Kaushik  wrote:
> 
> Dear all,
> I want to convert a POSCAR file into the QE input file. I would be very 
> grateful if somebody helps or give suggestions.
> Fe1 S1  
>1. 
>  5.9487492159093023   -0.   -0.
> -2.97437460795465115.15176794171171260.
> -0.0.   11.7211034581915818
>Fe   S 
> 1212
> Direct
>   0.3954866316692124  0.0721204949184797  0.1217724210678790
>   0.9278795160815250  0.3233661487507409  0.1217724210678790
>   0.6766338512492591  0.6045133683307876  0.1217724210678790
>   0.3954866316692124  0.0721204949184797  0.3782275859321179
>   0.9278795160815250  0.3233661487507409  0.3782275859321179
>   0.6766338512492591  0.6045133683307876  0.3782275859321179
>   0.0721204949184797  0.3954866316692124  0.8782275859321180
>   0.3233661487507409  0.9278795160815250  0.8782275859321180
>   0.6045133683307876  0.6766338512492591  0.8782275859321180
>   0.0721204949184797  0.3954866316692124  0.6217724140678820
>   0.3233661487507409  0.9278795160815250  0.6217724140678820
>   0.6045133683307876  0.6766338512492591  0.6217724140678820
>  -0. -0.  0.
>  -0. -0.  0.5000
>   0.33342996  0.66687029  0.0251416953082458
>   0.33342996  0.66687029  0.4748582936917498
>   0.66687029  0.33342996  0.9748582936917498
>   0.66687029  0.33342996  0.5251417063082502
>   0.6643064194527374  0.477489745914  0.2500
>   0.522750254107  0.6643586704781460  0.2500
>   0.3356412995218515  0.3356935805472627  0.2500
>   0.477489745914  0.6643064194527374  0.7500
>   0.6643586704781460  0.522750254107  0.7500
>   0.3356935805472627  0.3356412995218515  0.7500
> Thanks,
> Poonam S
> 
> 
> 
> -
> Poonam Sharma
> Research Scholar 
> Department of Physics
> Indian Institute of Technology Bombay
> Mumbai - 400076
> India.
> 
> ___
> Quantum ESPRESSO is supported by MaX (www.max-centre.eu/quantum-espresso)
> users mailing list users@lists.quantum-espresso.org
> https://lists.quantum-espresso.org/mailman/listinfo/users

___
Quantum ESPRESSO is supported by MaX (www.max-centre.eu/quantum-espresso)
users mailing list users@lists.quantum-espresso.org
https://lists.quantum-espresso.org/mailman/listinfo/users

Re: [QE-users] Si band structure using 8 atoms and SC lattice

2020-04-20 Thread Giovanni Cantele
Independently of whether one, both or no calculation(s) is (are) wrong, how do 
you judge whether or not a band structure is wrong?

A way to have a direct comparison between the two calculations is to compute 
the density of states of the two systems, since
the two density of states should be exactly the same (if both calculations are 
converged) , except for a scaling factor.

On the other hand, if you want to compare the band structures you should for 
example measure energy gaps. Because if you mean
that the second band structure looks different, this is indeed exactly what you 
expect. The second band structure is computed on
a larger unit cell (compare the unit cell volume  of the FCC and SC lattices), 
as such it has a smaller Brillouin zone and the
resulting bands are folded. Also, using the same special k-points does not give 
a direct term of comparison, since the high symmetry
points, usually used for plotting band structures, are different for different 
lattices. Last but not least, if in the second case you want to
plot the band structure along exactly the same path of the former, you should 
figure out how the k-points of the FCC Brillouin zone
read in terms of reciprocal lattico vectors of the FCC system.

Giovanni


-- 

Giovanni Cantele, PhD

CNR-SPIN
c/o Dipartimento di Fisica
Universita' di Napoli "Federico II"
Complesso Universitario M. S. Angelo - Ed. 6
Via Cintia, I-80126, Napoli, Italy

e-mail: giovanni.cant...@spin.cnr.it
gcant...@gmail.com
Phone: +39 081 676910
Skype contact: giocan74
Web page: https://sites.google.com/view/giovanni-cantele

> On 19 Apr 2020, at 15:00, Mohamed Saleh  wrote:
> 
> Dear all,
> 
> I have successfully produced the band structure of bulk Si using FCC lattice 
> with 2 atoms per basis. The relevant input sections are as follows:
> 
> 
> ibrav=2, 
> celldm(1)=10.410909236,
> nat=2, 
> ntyp=1,
> nbnd=8
> /
> 
> ATOMIC_POSITIONS (alat)
> Si 0.00 0.00 0.00
> Si 0.25 0.25 0.25
> 
> However, when I try to get the same band structure using a SC unit cell that 
> contain 8 Si atoms, I get completely wrong results. The relevant input 
> sections are as follows:
> 
> 
> ibrav=1, 
> celldm(1)=10.3557,
> nat=8, 
> ntyp=1,
> nbnd=32
> /
> 
> ATOMIC_POSITIONS (alat)
> Si 0 0 0
> Si 0 0.5 0.5
> Si 0.5 0.5 0
> Si 0.5 0 0.5
> Si 0.75 0.25 0.75
> Si 0.25 0.25 0.25
> Si 0.25 0.75 0.75
> Si 0.75 0.75 0.25
> 
> Both calculations use the same special k-points. The resulting band 
> structures (the correct one and the wrong one) are attached. So what am I 
> doing wrong?
> 
> Thanks in advance
> 
> 
> ___
> Quantum ESPRESSO is supported by MaX (www.max-centre.eu/quantum-espresso 
> <http://www.max-centre.eu/quantum-espresso>)
> users mailing list users@lists.quantum-espresso.org 
> <mailto:users@lists.quantum-espresso.org>
> https://lists.quantum-espresso.org/mailman/listinfo/users 
> <https://lists.quantum-espresso.org/mailman/listinfo/users>
___
Quantum ESPRESSO is supported by MaX (www.max-centre.eu/quantum-espresso)
users mailing list users@lists.quantum-espresso.org
https://lists.quantum-espresso.org/mailman/listinfo/users

[QE-users] Issue on LDA+U test calculation on Gd

2020-02-10 Thread Giovanni Cantele
Dear all,

I’m trying to make a test calculation of a crystal containing the Gd atom. I 
use the DFT+U scheme.
However, as the program starts, I’ve immediately facing the following error:

%%
 Error in routine offset_atom_wfc (2):
 wrong offset
 %%


I’ve browsed the PW/src/offset_atom_wfc.f90 routine. It seems (that is what I 
have understood) that while in Modules/set_hubbard_l.f90 it is set 
hubbard_l = 3  for that atomic species, in the pseudo potential file (see e.g. 
http://www.quantum-espresso.org/upf_files/Gd.pbe-spdn-kjpaw_psl.1.0.0.UPF 
<http://www.quantum-espresso.org/upf_files/Gd.pbe-spdn-kjpaw_psl.1.0.0.UPF>)
the l=3 wave function is not found.
The atomic configuration of Gd is Xe 4f7 5d1 6s2.

So my question is:

i) it is sufficient to solve the problem or wrong to set hubbard_l = 2 in 
Modules/set_hubbard_l.f90?
ii) more importantly (sorry for the basic question, I’v never considered 
systems with f electrons): why f electrons are not considered in the pseudo 
file?
Should the Hubbard U be more important for those l=3 electronic states than for 
l=2 states?

Thanks for any answer you could provide.

Giovanni



-- 

Giovanni Cantele, PhD

CNR-SPIN
c/o Dipartimento di Fisica
Universita' di Napoli "Federico II"
Complesso Universitario M. S. Angelo - Ed. 6
Via Cintia, I-80126, Napoli, Italy

e-mail: giovanni.cant...@spin.cnr.it
gcant...@gmail.com
Phone: +39 081 676910
Skype contact: giocan74
Web page: https://sites.google.com/view/giovanni-cantele

___
Quantum ESPRESSO is supported by MaX (www.max-centre.eu/quantum-espresso)
users mailing list users@lists.quantum-espresso.org
https://lists.quantum-espresso.org/mailman/listinfo/users

Re: [QE-users] Physical constants conversion inconsistency

2020-01-23 Thread Giovanni Cantele
Dear S. Kumar,

maybe some developer will give you a more precise answer. However, just to make 
a remark, what I would say is that what one should also check
is to what extent a change in the conversion and/or in the alat would modify 
your results. 
For example, as far as the the number
of decimal places is concerned, let us suppose that we are making a calculation 
of a cubic crystal with a = 3.40613 A and in place of it
we specify a = 3.4061 A. That would correspond to an underestimation of the 
lattice constant of less that 0.0009 %, much less than typical accuracy that
can be reached by DFT in estimating the lattice constants. With such a small 
error, if you compute for example the band structure with
the two values of the lattice parameter, you would appreciate no difference 
between the two (I would guess with changes in the eigenvalues
much less that 1 meV).

Just to make an exercise, I tried with the following input, that concerns bulk 
silicon:

calculation  = 'scf'
title= 'Silicon'
restart_mode = 'from_scratch'
outdir   = './tmp/'
prefix   = 'Si'
tprnfor  = .true.
pseudo_dir   = './'
/

ibrav= 2
a  = 5.43
nat  = 2
ntyp = 1
ecutwfc= 44
ecutrho= 200
nbnd   = 8
/
 
/
ATOMIC_SPECIES
Si   28.09 Si.pbe-n-kjpaw_psl.1.0.0.UPF
ATOMIC_POSITIONS { alat }
Si   0.00  0.00  0.00
Si   0.25  0.25  0.25
K_POINTS { automatic }
4 4 40  0  0


With the shown value of a you get a total energy of -93.43935682 Ry and 
eigenvalues at Gamma point equal to
-5.7252   6.2495   6.2495   6.2495   8.7945   8.7945   8.7945   9.6160

If I change a from 5.43 to 5.43005 (on the 5th digit, which corresponds to a 
change of 0.0009%) the previous quantities become
-93.43935815 Ry and
-5.7253   6.2492   6.2492   6.2492   8.7941   8.7942   8.7942   9.6154.
The total energy changes by -0.0133 Ry (-0.018 meV), the gap at Gamma point 
from 2.545 to 2.5449 (-0.1 meV).


Giovanni


> On 23 Jan 2020, at 06:00, Sonu Kumar <1009uku...@gmail.com> wrote:
> 
> Dear QE developers,
> 
> I found that physical constant conversion factors are incorrect in the q-e 
> version 6.2 and 6.4.1. For example, Bohr to ang conversion factor is around 
> 0.529177208591, while nist codata value 
> (https://physics.nist.gov/cgi-bin/cuu/Value?bohrrada0 
> <https://physics.nist.gov/cgi-bin/cuu/Value?bohrrada0>) is 5.291 772 109 03 
> e-11 m.  This error is introduced in data-file-schema.xml files as lattice 
> parameters are written in Bohr (provided input file contain lattice 
> parameters in angstrom units). This error might lead to other erroneous 
> physical quantities involving angstrom units. 
> 
> Also, output file contains 'lattice parameter (alat)  =' values upto 4 
> decimal places; It will be better (in my opinion) to add more decimal places 
> so as to check conversions to angstrom units or so.
> 
> Looking into the constants.f90 files in Modules in the src directory, I found 
> the same. This is true for other constants mentioned in this file. 
> 
> With kind regards, sonu
> 
> 
> With kind regards,
> S Kumar |  PhD.
> ICReDD
> ___
> Quantum ESPRESSO is supported by MaX (www.max-centre.eu/quantum-espresso)
> users mailing list users@lists.quantum-espresso.org
> https://lists.quantum-espresso.org/mailman/listinfo/users

-- 

Giovanni Cantele, PhD

CNR-SPIN
c/o Dipartimento di Fisica
Universita' di Napoli "Federico II"
Complesso Universitario M. S. Angelo - Ed. 6
Via Cintia, I-80126, Napoli, Italy

e-mail: giovanni.cant...@spin.cnr.it
    gcant...@gmail.com
Phone: +39 081 676910
Skype contact: giocan74
Web page: https://sites.google.com/view/giovanni-cantele

___
Quantum ESPRESSO is supported by MaX (www.max-centre.eu/quantum-espresso)
users mailing list users@lists.quantum-espresso.org
https://lists.quantum-espresso.org/mailman/listinfo/users

Re: [QE-users] X-CrysDen atomic position error

2019-12-17 Thread Giovanni Cantele
sorry, at some point of my message I (or the automatic correction) wrote
CaO has a cubic bcc lattice
that instead was meant
CaO has a cubic fcc lattice

Giovanni

> On 17 Dec 2019, at 10:47, Giovanni Cantele  
> wrote:
> 
> You’re using ibrav=2, which corresponds to a cubic F (fcc). The atomic 
> positions are in alat units. As such, as clearly stated by the error message,
> atoms 1 and 2 are equivalent. Indeed, their positions are:
> #1 --> a/2 a/2   0
> #2 —>   00   0
> 
> Atom #1 lies at the center of a face, its position for a cubic bcc lattice is 
> obtained by translating the lattice site at the origin (where atom #2 lies)
> by a direct lattice vector. 
> 
> You must specify *ONLY* inequivalent atoms, their periodic replicas cannot be 
> included in the list of atoms. There are many other similar
> overlapping atoms, such as
> Ca —> 0 0 0 with Ca —> 1 1 0 or Ca —> 0 0 1.
> 
> Actually, to be honest, you should probably remove *ALL ATOMS BUT TWO*. 
> Indeed, as far as I remember, CaO has a cubic bcc lattice
> with on Ca atom in 0 0 0 and on O atom in a/2 a/2 a/2 (but please check!).
> 
> Provided that, in my opinion, any question is welcome, my suggestion is that 
> you try to give a solution to error messages by yourself before
> asking people, because what you learn if you “try to solve” is priceless if 
> compared to what you learn if you “ask to solve”. In this respect,
> Quantum-ESPRESSO is an exceptional lab to make experience, since especially 
> (but not only) for the easiest tasks (such as build the band structure of
> simple solids) many error messages are self-explanatory and user-friendly!  
> ;-)
> 
> Giovanni
> 
> 
> 
>> On 17 Dec 2019, at 10:25, Pooja Vyas > <mailto:poojavyas1251...@gmail.com>> wrote:
>> 
>> Following is my input file. I obtained the atomic position using X-CrysDen. 
>> But when I run the file it shows me an error message.
>> 
>> Input file:
>> 
>> calculation = 'scf',
>> prefix = '9.1334'
>> tstress= .true.
>> tprnfor= .true.
>> outdir = '/home/userpooja/cao.oct/'
>> pseudo_dir = '/home/userpooja/cao.oct/pseudo/'
>>  /
>>  
>> ibrav =  2,
>> celldm(1) = 9.1334,
>> nat =  27,
>> ntyp = 2,
>> ecutwfc = 100,
>> /
>> 
>> mixing_beta = 0.7
>>  /
>> 
>> ATOMIC_SPECIES
>> 
>> Ca 40.078  Ca.pbe-nsp-van.UPF
>> O 15.999 O.pbe-van_ak.UPF
>> 
>> ATOMIC_POSITIONS (alat)
>> Ca 0.5 0.5 0.0
>> Ca 0.0 0.0 0.0
>> Ca 0.5 0.0 0.5
>> Ca 0.0 0.5 0.5
>> O  0.0 0.0 0.5
>> O  0.5 0.5 0.5
>> O  0.0 0.5 0.0
>> O  0.5 0.0 0.0
>> Ca 0.5 0.5 1.0
>> Ca 0.0 0.0 1.0
>> O  0.0 0.5 1.0
>> O  0.5 0.0 1.0
>> Ca 0.0 1.0 0.0
>> Ca 0.5 1.0 0.5
>> O  0.0 1.0 0.5
>> O  0.5 1.0 0.0
>> Ca 0.0 1.0 1.0
>> O  0.5 1.0 1.0
>> Ca 1.0 0.0 0.0
>> Ca 1.0 0.5 0.5
>> O  1.0 0.0 0.5
>> O  1.0 0.5 0.0
>> Ca 1.0 0.0 1.0
>> O  1.0 0.5 1.0
>> Ca 1.0 1.0 0.0
>> O  1.0 1.0 0.5
>> Ca 1.0 1.0 1.0
>> K_POINTS (automatic)
>>   11 11 11 1 1 1
>> 
>> Error: 
>> 
>>  
>> %%
>>  task #17
>>  from check_atoms : error # 1
>>  atoms #   1 and #   2 differ by lattice vector (-1, 1, 0) in crystal 
>> axis
>>  
>> %%
>> ___
>> Quantum ESPRESSO is supported by MaX (www.max-centre.eu/quantum-espresso 
>> <http://www.max-centre.eu/quantum-espresso>)
>> users mailing list users@lists.quantum-espresso.org 
>> <mailto:users@lists.quantum-espresso.org>
>> https://lists.quantum-espresso.org/mailman/listinfo/users 
>> <https://lists.quantum-espresso.org/mailman/listinfo/users>
> -- 
> 
> Giovanni Cantele, PhD
> 
> CNR-SPIN
> c/o Dipartimento di Fisica
> Universita' di Napoli "Federico II"
> Complesso Universitario M. S. Angelo - Ed. 6
> Via Cintia, I-80126, Napoli, Italy
> 
> e-mail: giovanni.cant...@spin.cnr.it <mailto:giovanni.cant...@spin.cnr.it>
> gcant...@gmail.com <mailto:gcant...@gmail.com>
> Phone: +39 081 676910
> Skype contact: giocan74
> Web page: https://sites.google.com/view/giovanni-cantele 
> <https://sites.google.com/view/giovanni-cantele>
> ___
> Quantum ESPRESSO is supported by MaX (www.max-centre.eu/quantum-espresso)
> users mailing list users@lists.quantum-espresso.org
> https://lists.quantum-espresso.org/mailman/listinfo/users

-- 

Giovanni Cantele, PhD

CNR-SPIN
c/o Dipartimento di Fisica
Universita' di Napoli "Federico II"
Complesso Universitario M. S. Angelo - Ed. 6
Via Cintia, I-80126, Napoli, Italy

e-mail: giovanni.cant...@spin.cnr.it
gcant...@gmail.com
Phone: +39 081 676910
Skype contact: giocan74
Web page: https://sites.google.com/view/giovanni-cantele

___
Quantum ESPRESSO is supported by MaX (www.max-centre.eu/quantum-espresso)
users mailing list users@lists.quantum-espresso.org
https://lists.quantum-espresso.org/mailman/listinfo/users

Re: [QE-users] X-CrysDen atomic position error

2019-12-17 Thread Giovanni Cantele
You’re using ibrav=2, which corresponds to a cubic F (fcc). The atomic 
positions are in alat units. As such, as clearly stated by the error message,
atoms 1 and 2 are equivalent. Indeed, their positions are:
#1 --> a/2 a/2   0
#2 —>   00   0

Atom #1 lies at the center of a face, its position for a cubic bcc lattice is 
obtained by translating the lattice site at the origin (where atom #2 lies)
by a direct lattice vector. 

You must specify *ONLY* inequivalent atoms, their periodic replicas cannot be 
included in the list of atoms. There are many other similar
overlapping atoms, such as
Ca —> 0 0 0 with Ca —> 1 1 0 or Ca —> 0 0 1.

Actually, to be honest, you should probably remove *ALL ATOMS BUT TWO*. Indeed, 
as far as I remember, CaO has a cubic bcc lattice
with on Ca atom in 0 0 0 and on O atom in a/2 a/2 a/2 (but please check!).

Provided that, in my opinion, any question is welcome, my suggestion is that 
you try to give a solution to error messages by yourself before
asking people, because what you learn if you “try to solve” is priceless if 
compared to what you learn if you “ask to solve”. In this respect,
Quantum-ESPRESSO is an exceptional lab to make experience, since especially 
(but not only) for the easiest tasks (such as build the band structure of
simple solids) many error messages are self-explanatory and user-friendly!  ;-)

Giovanni



> On 17 Dec 2019, at 10:25, Pooja Vyas  wrote:
> 
> Following is my input file. I obtained the atomic position using X-CrysDen. 
> But when I run the file it shows me an error message.
> 
> Input file:
> 
> calculation = 'scf',
> prefix = '9.1334'
> tstress= .true.
> tprnfor= .true.
> outdir = '/home/userpooja/cao.oct/'
> pseudo_dir = '/home/userpooja/cao.oct/pseudo/'
>  /
>  
> ibrav =  2,
> celldm(1) = 9.1334,
> nat =  27,
> ntyp = 2,
> ecutwfc = 100,
> /
> 
> mixing_beta = 0.7
>  /
> 
> ATOMIC_SPECIES
> 
> Ca 40.078  Ca.pbe-nsp-van.UPF
> O 15.999 O.pbe-van_ak.UPF
> 
> ATOMIC_POSITIONS (alat)
> Ca 0.5 0.5 0.0
> Ca 0.0 0.0 0.0
> Ca 0.5 0.0 0.5
> Ca 0.0 0.5 0.5
> O  0.0 0.0 0.5
> O  0.5 0.5 0.5
> O  0.0 0.5 0.0
> O  0.5 0.0 0.0
> Ca 0.5 0.5 1.0
> Ca 0.0 0.0 1.0
> O  0.0 0.5 1.0
> O  0.5 0.0 1.0
> Ca 0.0 1.0 0.0
> Ca 0.5 1.0 0.5
> O  0.0 1.0 0.5
> O  0.5 1.0 0.0
> Ca 0.0 1.0 1.0
> O  0.5 1.0 1.0
> Ca 1.0 0.0 0.0
> Ca 1.0 0.5 0.5
> O  1.0 0.0 0.5
> O  1.0 0.5 0.0
> Ca 1.0 0.0 1.0
> O  1.0 0.5 1.0
> Ca 1.0 1.0 0.0
> O  1.0 1.0 0.5
> Ca 1.0 1.0 1.0
> K_POINTS (automatic)
>   11 11 11 1 1 1
> 
> Error: 
> 
>  
> %%
>  task #17
>  from check_atoms : error # 1
>  atoms #   1 and #   2 differ by lattice vector (-1, 1, 0) in crystal axis
>  
> %%
> _______
> Quantum ESPRESSO is supported by MaX (www.max-centre.eu/quantum-espresso)
> users mailing list users@lists.quantum-espresso.org
> https://lists.quantum-espresso.org/mailman/listinfo/users

-- 

Giovanni Cantele, PhD

CNR-SPIN
c/o Dipartimento di Fisica
Universita' di Napoli "Federico II"
Complesso Universitario M. S. Angelo - Ed. 6
Via Cintia, I-80126, Napoli, Italy

e-mail: giovanni.cant...@spin.cnr.it
gcant...@gmail.com
Phone: +39 081 676910
Skype contact: giocan74
Web page: https://sites.google.com/view/giovanni-cantele

___
Quantum ESPRESSO is supported by MaX (www.max-centre.eu/quantum-espresso)
users mailing list users@lists.quantum-espresso.org
https://lists.quantum-espresso.org/mailman/listinfo/users

Re: [QE-users] electric field units

2019-12-04 Thread Giovanni Cantele
thanks for help, just another quick question:
from a test I’m doing, it seems (but I could be wrong, I’ll make more careful 
checks in the next days) that eamp and esm_efield
have opposite sign, that is to get the results with eamp>0 I should use 
esm_efield<0 and viceversa. Does it make sense or I’m getting
unreasonable results for some reason?

Thanks again,

Giovanni


> On 4 Dec 2019, at 16:31, Paolo Giannozzi  wrote:
> 
> I am quite confident that what is written in the documentation is correct, 
> but I have no evidence to support (or disprove) this
> 
> Paolo
> 
> On Wed, Dec 4, 2019 at 11:48 AM Giovanni Cantele 
> mailto:giovanni.cant...@spin.cnr.it>> wrote:
> Dear all,
> 
> I would like just to have confirmation about the units used by QE for 
> electric field within two different approaches:
> 
> 1) sawtooth potential 
> from the documentation I see Amplitude of the electric field, in 
> ***Hartree*** a.u.;
> 1 a.u. = 51.4220632*10^10 V/m
> 
> That means that 1 V / A = 1 / 51.4220632 = 0.019447 => eamp = 0.019447 in 
> this example
> 
> 
> 2) ESM method with esm_bc = ‘bc2’
> from the documentation I read "magnitude of the electric field [Ry/a.u.] to 
> be applied
> between semi-infinite ESM electrodes.”
> 
> That means that 1 V / A = 2 / 51.4220632 = 0.038894
> 
> 
> So, if I want to test both methods, I should use in the former case a value 
> of eamp, in the latter twice this value.
> Is it right?
> 
> Thanks for support,
> 
> Giovanni
> 
> -- 
> 
> Giovanni Cantele, PhD
> 
> CNR-SPIN
> c/o Dipartimento di Fisica
> Universita' di Napoli "Federico II"
> Complesso Universitario M. S. Angelo - Ed. 6
> Via Cintia, I-80126, Napoli, Italy
> 
> e-mail: giovanni.cant...@spin.cnr.it <mailto:giovanni.cant...@spin.cnr.it>
> gcant...@gmail.com <mailto:gcant...@gmail.com>
> Phone: +39 081 676910
> Skype contact: giocan74
> Web page: https://sites.google.com/view/giovanni-cantele 
> <https://sites.google.com/view/giovanni-cantele>
> ___
> Quantum ESPRESSO is supported by MaX (www.max-centre.eu/quantum-espresso 
> <http://www.max-centre.eu/quantum-espresso>)
> users mailing list users@lists.quantum-espresso.org 
> <mailto:users@lists.quantum-espresso.org>
> https://lists.quantum-espresso.org/mailman/listinfo/users 
> <https://lists.quantum-espresso.org/mailman/listinfo/users>
> 
> -- 
> Paolo Giannozzi, Dip. Scienze Matematiche Informatiche e Fisiche,
> Univ. Udine, via delle Scienze 208, 33100 Udine, Italy
> Phone +39-0432-558216, fax +39-0432-558222
> 
> ___
> Quantum ESPRESSO is supported by MaX (www.max-centre.eu/quantum-espresso)
> users mailing list users@lists.quantum-espresso.org
> https://lists.quantum-espresso.org/mailman/listinfo/users

-- 

Giovanni Cantele, PhD

CNR-SPIN
c/o Dipartimento di Fisica
Universita' di Napoli "Federico II"
Complesso Universitario M. S. Angelo - Ed. 6
Via Cintia, I-80126, Napoli, Italy

e-mail: giovanni.cant...@spin.cnr.it
gcant...@gmail.com
Phone: +39 081 676910
Skype contact: giocan74
Web page: https://sites.google.com/view/giovanni-cantele

___
Quantum ESPRESSO is supported by MaX (www.max-centre.eu/quantum-espresso)
users mailing list users@lists.quantum-espresso.org
https://lists.quantum-espresso.org/mailman/listinfo/users

Re: [QE-users] elements contribution in projected band structure

2019-12-04 Thread Giovanni Cantele


> On 4 Dec 2019, at 14:13, 후신 부지드  wrote:
> 
> Dear QE users
> I've plotted the spin up and down band structures (from the file .dat.gnu) 
> for a magnetic material (it is a Diluted magnetic semiconductor) but I would 
> like to know to which atom or element belong each band and make it color 
> coded. The problem I cannot do this simply from the DOS since I have a 
> supercell containing many atoms and the bands are too many.
> I have seen people do this in VASP, is it possible to do the same in QE?
> 
> Houcine Bouzid
> Sungkyunkwan University, 
>  
> ___
> Quantum ESPRESSO is supported by MaX (www.max-centre.eu/quantum-espresso 
> <http://www.max-centre.eu/quantum-espresso>)
> users mailing list users@lists.quantum-espresso.org 
> <mailto:users@lists.quantum-espresso.org>
> https://lists.quantum-espresso.org/mailman/listinfo/users 
> <https://lists.quantum-espresso.org/mailman/listinfo/users>


It is not possible at all to do it simply from the DOS, you need the 
atom-resolved DOS, that is the projected DOS as obtained from projwfc.x

Once you’ve calculated the PDOS, I do not know whether there is some available 
script, but what you should do is, for each k-point and band you’re interested 
in,
read the corresponding projections (from standard output or from filproj if you 
set it up in the input file of projwfc.x), sum them up and use the result as a 
color or point size code. The script accomplishing this task should be a bit 
tricky but not that hard to code.

Giovanni

-- 

Giovanni Cantele, PhD

CNR-SPIN
c/o Dipartimento di Fisica
Universita' di Napoli "Federico II"
Complesso Universitario M. S. Angelo - Ed. 6
Via Cintia, I-80126, Napoli, Italy

e-mail: giovanni.cant...@spin.cnr.it
gcant...@gmail.com
Phone: +39 081 676910
Skype contact: giocan74
Web page: https://sites.google.com/view/giovanni-cantele

___
Quantum ESPRESSO is supported by MaX (www.max-centre.eu/quantum-espresso)
users mailing list users@lists.quantum-espresso.org
https://lists.quantum-espresso.org/mailman/listinfo/users

[QE-users] electric field units

2019-12-04 Thread Giovanni Cantele
Dear all,

I would like just to have confirmation about the units used by QE for electric 
field within two different approaches:

1) sawtooth potential 
from the documentation I see Amplitude of the electric field, in ***Hartree*** 
a.u.;
1 a.u. = 51.4220632*10^10 V/m

That means that 1 V / A = 1 / 51.4220632 = 0.019447 => eamp = 0.019447 in this 
example


2) ESM method with esm_bc = ‘bc2’
from the documentation I read "magnitude of the electric field [Ry/a.u.] to be 
applied
between semi-infinite ESM electrodes.”

That means that 1 V / A = 2 / 51.4220632 = 0.038894


So, if I want to test both methods, I should use in the former case a value of 
eamp, in the latter twice this value.
Is it right?

Thanks for support,

Giovanni

-- 

Giovanni Cantele, PhD

CNR-SPIN
c/o Dipartimento di Fisica
Universita' di Napoli "Federico II"
Complesso Universitario M. S. Angelo - Ed. 6
Via Cintia, I-80126, Napoli, Italy

e-mail: giovanni.cant...@spin.cnr.it
gcant...@gmail.com
Phone: +39 081 676910
Skype contact: giocan74
Web page: https://sites.google.com/view/giovanni-cantele

___
Quantum ESPRESSO is supported by MaX (www.max-centre.eu/quantum-espresso)
users mailing list users@lists.quantum-espresso.org
https://lists.quantum-espresso.org/mailman/listinfo/users

Re: [QE-users] Convergency problem

2019-11-23 Thread Giovanni Cantele
I cannot properly check your input because I don’t have a computer now, however 
I can suggest to you some things to try and some to be changed.

1) you use PAW pseudo potentials , you should specify ecutrho, since the 
default value of 4*ecutwfc is much likely insufficient, typical values are from 
6 to 12 times ecutwfc, maybe this might result in an uncorverged total charge 
density

2) also the k-point sampling could be checked against convergence, I would try 
2x2x1

3) mixing_beta can be reduced down to 0.01 in the worst cases 

4) maybe it could be worth converging a calculation with no dipole correction, 
then restart it (calculation=‘restart’) after turning it on, pw.x should not 
complain as far as I remember

5) increase degauss, converge the calculation with the higher (less converged 
and more unphysical) smearing and then restart with the initial degauss 

Giovanni

Inviato da iPhone

> Il giorno 23 nov 2019, alle ore 11:15, Ganes Shukri  ha 
> scritto:
> 
> Dear QE users, 
> 
> I am trying to optimize a hexagonal WO3 (001) surface (a quite common system 
> though). However, I am facing difficulty in making the electronic 
> optimization converge (let alone the ionic optimization). I have been trying 
> to change some parameters that may affect the convergency (i.e. mixing_beta, 
> conv_thr, mixing_mode, diagonalization, dipole correction parameters (eamp, 
> emaxpos)), but so far, none is working. The total energy always fluctuates 
> abruptly and does not seem to go to a particular value. I also already used 
> both types of pseudo potential (US and PAW), but still my calculation failed 
> to reach convergency.
> 
> I would be grateful if there are any suggestions or hints from other fellow 
> QE users.
> 
> Below is the (latest) input file that I used and the corresponding structure. 
> 
> ---
> 
> calculation='relax'
> restart_mode='from_scratch'
> pseudo_dir='/home/hakade/ganes/qe_pp'
> outdir='./temp'
> prefix='h-wo3'
> tprnfor=.true.
> dipfield=.true.
> tefield=.true. 
> /
>  
> ibrav=0,
> nat=102,
> ntyp=2,
> ecutwfc=42.0,
> occupations='smearing',
> smearing='gauss',
> degauss=0.02,
> nspin=1,
> starting_magnetization(1)=1,
> edir=3,
> emaxpos=0.80,
> eopreg=0.05,
> eamp=0.00,
> !lda_plus_u=.TRUE.,
> !lda_plus_u_kind=0,
> !Hubbard_U(1)=,
>  /
>  
> diagonalization='cg',
> conv_thr=1.d-4,
> mixing_beta=0.2,
> mixing_ndim=8,
> mixing_mode='local-TF',
> electron_maxstep=150,
>  /
>  
> ion_dynamics='bfgs'
>  /
> ATOMIC_SPECIES
> W 183.84 W.pbe-spn-kjpaw_psl.1.0.0.UPF
> O 15.999 O.pbe-n-kjpaw_psl.1.0.0.UPF
> 
> ATOMIC_POSITIONS angstrom
> W -1.853873   3.21100118.336259   1   1   1
> W 3.7077450.0018.336259   1   1   1
> W 1.8538733.21100118.336259   1   1   1
> W 5.5616183.21100118.336259   1   1   1
> W 11.123235   0.0018.336259   1   1   1
> W 9.2693633.21100118.336259   1   1   1
> W -1.853873   3.21100114.525900   1   1   1
> W 3.7077450.0014.525900   1   1   1
> W 1.8538733.21100114.525900   1   1   1
> W 5.5616183.21100114.525900   1   1   1
> W 11.123235   0.0014.525900   1   1   1
> W 9.2693633.21100114.525900   1   1   1
> W -1.853873   3.21100110.715540   0   0   0
> W 3.7077450.0010.715540   0   0   0
> W 1.8538733.21100110.715540   0   0   0
> W 5.5616183.21100110.715540   0   0   0
> W 11.123235   0.0010.715540   0   0   0
> W 9.2693633.21100110.715540   0   0   0
> W -1.853873   3.2110016.9051800   0   0
> W 3.7077450.006.9051800   0   0
> W 1.8538733.2110016.9051800   0   0
> W 5.5616183.2110016.9051800   0   0
> W 11.123235   0.006.9051800   0   0
> W 9.2693633.2110016.9051800   0   0
> O -1.853873   3.21100120.241440   1   1   1
> O 3.7077450.0020.241440   1   1   1
> O 1.8538733.21100120.241440   1   1   1
> O 5.5616183.21100120.241440   1   1   1
> O 11.123235   0.0020.241440   1   1   1
> O 9.2693633.21100120.241440  

Re: [QE-users] quantum espressso

2019-11-20 Thread Giovanni Cantele
1) you should sign a message with your name and affiliation

2) it is quite unlikely that anyone will give you a tutorial in this forum 

3) you can browse the examples included in the distribution as well as go to 
the web site where you can find tutorials 



Inviato da iPhone

> Il giorno 20 nov 2019, alle ore 14:48, GAMERS STREAM 
>  ha scritto:
> 
> 
> I am beginning larner quantum espresso. first of all how to use quantum 
> espresso?
> ___
> Quantum ESPRESSO is supported by MaX (www.max-centre.eu/quantum-espresso)
> users mailing list users@lists.quantum-espresso.org
> https://lists.quantum-espresso.org/mailman/listinfo/users

___
Quantum ESPRESSO is supported by MaX (www.max-centre.eu/quantum-espresso)
users mailing list users@lists.quantum-espresso.org
https://lists.quantum-espresso.org/mailman/listinfo/users

Re: [QE-users] =>> PBS: job killed: walltime 172854 exceeded limit 172800 . how can i correct this error.

2019-09-12 Thread Giovanni Cantele
The error message means that either you submitted a job by defining a finite 
wall time
that has been exceeded (that is, you job would have required more time than 
required),
or that the default wall time il too small for jour job or else that the 
queuing setup has
a maximum wall time, that cannot be exceeded in any case.

In the first two cases, you should require a larger time for your job. In the 
latter,
the only solution is to restart your job several times, until it lasts.
Also consider that QE has a max_seconds variable (you can learn its usage
from the user guide) that allows you to setup a maximum execution time,
independently of the queuing system. In general you set max_seconds to
a time that is a bit smaller than the maximum wall time imposed by the
queuing system (if any). In this way the code exits smoothly before it is killed
by the queuing system. Indeed, abrupt killing might result in an incomplete
writing of the files necessary for subsequent restart.

As a general remark, messages posted to the forum are usually signed with 
affiliation.
Moreover, I think it is not that efficient to include the body of the message 
in its subject.

Giovanni


> On 12 Sep 2019, at 12:21, Kwaoallan Blaze  wrote:
> 
> 
> ___
> Quantum ESPRESSO is supported by MaX (www.max-centre.eu/quantum-espresso)
> users mailing list users@lists.quantum-espresso.org
> https://lists.quantum-espresso.org/mailman/listinfo/users

-- 

Giovanni Cantele, PhD

CNR-SPIN
c/o Dipartimento di Fisica
Universita' di Napoli "Federico II"
Complesso Universitario M. S. Angelo - Ed. 6
Via Cintia, I-80126, Napoli, Italy

e-mail: giovanni.cant...@spin.cnr.it
gcant...@gmail.com
Phone: +39 081 676910
Skype contact: giocan74
Web page: https://sites.google.com/view/giovanni-cantele

___
Quantum ESPRESSO is supported by MaX (www.max-centre.eu/quantum-espresso)
users mailing list users@lists.quantum-espresso.org
https://lists.quantum-espresso.org/mailman/listinfo/users

Re: [QE-users] Fwd: Error while parsing atomic position card

2019-09-04 Thread Giovanni Cantele
I don’t know if this is the problem,
but if I copy and paste your ATOMIC_POSITIONS card, a hidden character shows up 
in the last
line (Co y coordinate). This i what I see in my text file:
2.127711552<202c>0
Try to delete it and rewrite without cut
Giovanni

> On 4 Sep 2019, at 12:36, hamed asadi  wrote:
> 
> 
> Dear qe users;
> 
> I'm trying to relax cobalt doped graphene (the input file is attached bellow) 
> but after starting calculations the fallowing error appeared.
> could someone help me with this issue?
>  
> %%
>  task # 0
>  from card_atomic_positions : error # 1
>  Error while parsing atomic position card.
>   
>   
>   
>  
> %%
> 
>  
>  
> calculation = 'relax' ,
> restart_mode = 'from_scratch' ,
> etot_conv_thr = 1.0E-8  , 
> forc_conv_thr = 1.0D-8 ,
> outdir='/root/Desktop/PROJECT/g22Co/out',
> pseudo_dir = '/root/Desktop/PROJECT/g22Co/pp',   
> wf_collect=.true.  
> tprnfor   = .true.
> tstress = .true.
> verbosity= 'low'
> /
> 
> ibrav = 4,
> celldm(1) = 9.285,
> celldm(3) = 3.053,
> nbnd = 100,
> nat = 9 ,
> ntyp = 2 ,
> ecutwfc = 40 ,
> ecutrho = 400,   
> occupations = 'smearing' ,
> degauss= 0.01 ,
> smearing= 'gaussian',
> /
> 
> mixing_beta = 0.7
> conv_thr = 1.D-8 ,
> /
> 
>  ion_dynamics= 'bfgs'
>
> /
>  
>cell_dynamics = 'bfgs' ,
>cell_factor = 2
>  /
> ATOMIC_SPECIES
> C12.0107C.pbe-n-rrkjus_psl.0.1.UPF
> Co58.9332Co.pbe-spn-rrkjus_psl.0.3.1.UPF
> ATOMIC_POSITIONS (angstrom)
> C1.22842728800.70931659307.50
> C0.7255801.41850751207.50
> C0.5743302.83691559207.50
> C   -1.22829729703.54610651107.50
> C3.68516699700.70931659307.50
> C2.45681226701.41850751207.50
> C2.45679714202.83691559207.50
> C1.22844241203.54610651107.50
> Co   1.22843485002.127711552‬010.50
> K_POINTS {automatic}
> 8 8 1   0 0 0
> 
> Best regards,
> Hamed Asadi,
> KN Toosi university of technology
> +989126193984
> 
> 
> _______
> Quantum ESPRESSO is supported by MaX (www.max-centre.eu/quantum-espresso)
> users mailing list users@lists.quantum-espresso.org
> https://lists.quantum-espresso.org/mailman/listinfo/users

-- 

Giovanni Cantele, PhD

CNR-SPIN
c/o Dipartimento di Fisica
Universita' di Napoli "Federico II"
Complesso Universitario M. S. Angelo - Ed. 6
Via Cintia, I-80126, Napoli, Italy

e-mail: giovanni.cant...@spin.cnr.it
gcant...@gmail.com
Phone: +39 081 676910
Skype contact: giocan74
Web page: https://sites.google.com/view/giovanni-cantele

___
Quantum ESPRESSO is supported by MaX (www.max-centre.eu/quantum-espresso)
users mailing list users@lists.quantum-espresso.org
https://lists.quantum-espresso.org/mailman/listinfo/users

Re: [QE-users] Implementing a bias potential in a Si slab

2019-07-30 Thread Giovanni Cantele
There are three issues that might be among the causes of the strange behaviour 
you observe:

1) you should add dipfield = .true. , the dipole correction is needed due to 
the periodic boundary corrections

2) you set edir=3, emaxpos=0.9 and eopreg=0.1: the saw-like potential decreases 
from emaxpos within an eopreg interval,
that is, from 0.9 to 0.9 + 0.1 in units of the unit cell parameter along edir. 
Since along the z direction you have atoms at z=0
that, due to the periodic boundary conditions are replicated at z=1, the 
(unphysical but needed) decrease of the saw-like potential
occurs in a region of not zero charge density, thus adding spurious interaction 
to the calculation.
Usually you set emaxpos in the middle of the vacuum space separating two 
periodic replicas along z. For example, if I’m not wrong,
you have atoms ranging from z=0 to z=0.5, so I would set emaxpos=0.75. 
Moreover, I would choose a small eopreg.

3) You should also check whether the 28 Ry cutoff is enough to get converged 
properties with all the pseudo potentials included in your
calculation.

Giovanni

> On 30 Jul 2019, at 13:31, Helen Eisenberg  wrote:
> 
> Dear Users,
> 
> I am trying to implement a bias potential on a Si slab with a Li ion in the 
> vacuum, and I am getting unphysical things happening to the Si slab (it gets 
> squashed or stretched depending on which bias I use). This makes me think I'm 
> doing something wrong...Do I need to use a dipole correction (dipfield=TRUE)? 
> Am I doing something else wrong? Does it not work with molecular dynamics 
> only ionic relaxation?
> 
> Thank you for your help (I have copied my input file below, note the various 
> parameters, such as kpoints and ecutwfc are not yet converged, as these are 
> initial calculations )
> 
> Dr. Helen Eisenberg,
> 
> Fritz Haber Center for Molecular Dynamics,
> 
> Hebrew University
> 
> 
> 
>   calculation = "md",
>   restart_mode='from_scratch',
>   pseudo_dir  = "$PSEUDO_DIR",
>   prefix  = "Energy_Si_2x2",
>   tstress = .true.
>   tprnfor = .true.
>   outdir  = "$TMP_DIR"
> dt=40,
> nstep=800
> tefield=TRUE
> /
> 
>   ibrav   = 0,
>   nat = 33,
>   ntyp= 3,
>   ecutwfc = 28.D0,
>   occupations = 'smearing',
>   smearing= 'mv',
>   degauss = 0.0001,
>   tot_charge  = 0.0,
>   nspin   = 2,
>   starting_magnetization(1)=0.0
> edir=3
> emaxpos=0.9
> eopreg=0.1
> eamp=0.01
> /
> 
> electron_maxstep=500
>  mixing_mode = 'plain'
>   mixing_beta = 0.7
>   conv_thr =  1.0d-6
> /
>  
> pot_extrapolation='second-order'
> wfc_extrapolation='second-order'
>  /
> 
> ATOMIC_SPECIES
> Si  28.086  Si.pbe-mt_gipaw.UPF
> H 1.00794 H.pbe-vbc.UPF
> Li   6.941  Li.pbe-s-mt.UPF
> ATOMIC_POSITIONS (crystal)
> H 0.25  -0.005170 0 0 0
> H 0.75  -0.005170 0 0 0
> Si 0.51097  0.49804 0.16473
> Si 0.35376  0.49734 0.35774
> Si 0.25 0.24742 0.05541 0 0 0
> Si 0.24944  0.24765 0.27061
> Si 0.75 0.24742 0.05541 0 0 0
> Si 0.75919  0.24761 0.28048
> Si 6.33534E-4   0.49808 0.16495
> Si 0.07141  0.49748 0.38744
> Si -5.17962E-4  0.2482  0.10971
> Si 0.01902  0.24763 0.33382
> Si 0.75553  0.49764 0.22403
> Si 0.50177  0.24816 0.10965
> Si 0.4958   0.24746 0.33137
> Si 0.25633  0.49758 0.21703
> H 0.25  0.49483 0 0 0 0
> H 0.75  0.49483 0 0 0 0
> Si 0.51097  0.99804 0.16473
> Si 0.35376  0.99734 0.35774
> Si 0.25 0.74742 0.05541 0 0 0
> Si 0.24944  0.74765 0.27061
> Si 0.75 0.74742 0.05541 0 0 0
> Si 0.75919  0.74761 0.28048
> Si 6.33534E-4   0.99808 0.16495
> Si 0.07141  0.99748 0.38744
> Si -5.17962E-4  0.7482  0.10971
> Si 0.01902  0.74763 0.33382
> Si 0.75553  0.99764 0.22403
> Si 0.50177  0.74816 0.10965
> Si 0.4958   0.74746 0.33137
> Si 0.25633  0.99758 0.21703
> Li   0.625   0.5   0.5
> K_POINTS automatic
> 2 2  1  0  0  0
> CELL_PARAMETERS {angstrom}
> 7.68016959320.000.00
> 0.00    7.760340.00
> 0.000.00   24.503725
> EOF
> 
> ___
> Quantum ESPRESSO is supported by MaX (www.max-centre.eu/quantum-espresso)
> users mailing list users@lists.quantum-espresso.org
> https://lists.quantum-espresso.org/mailman/listinfo/users

-- 

Giovanni Cantele, PhD

CNR-SPIN
c/o Dipartimento di Fisica
Universita' di Napoli "Federico II"
Complesso Universitario M. S. Angelo - Ed. 6
Via Cintia, I-80126, Napoli, Italy

e-mail: giovanni.cant...@spin.cnr.it
gcant...@gmail.com
Phone: +39 081 676910
Skype contact: giocan74
Web page: https://sites.google.com/view/giovanni-cantele

___
Quantum ESPRESSO is supported by MaX (www.max-centre.eu/quantum-espresso)
users mailing list users@lists.quantum-espresso.org
https://lists.quantum-espresso.org/mailman/listinfo/users

Re: [QE-users] a query on weights of k-points

2019-07-24 Thread Giovanni Cantele
I think that the sum of the weights is correct because it takes into account 
the spin degeneration.
If I’m not wrong, what it is happening is that the code first normalises the 
weights to on and then multiplies them
by two in the case the run you are carrying out is not spin polarised.
If you see PW/src/setup.f90 you can find the line
  ELSE
 !
 ! ... LDA case: the two spin polarizations are identical
 !
 wk(1:nkstot)= wk(1:nkstot) * degspin



So, did you ad the factor of 2 due to the spin degeneracy in your own script?

Giovanni

> On 22 Jul 2019, at 09:41, Михаил Петров  wrote:
> 
> Dear users,
> 
> I am calculating a 1D system (stripe) and I have a problem calculating its 
> DOS.
> There is exactly 2 times difference between DOS calculated with my own script 
> and the one that I get from the dos.x.
> I have noticed that in my scf/nscf out files the weights of all the k-points 
> involved add up not to 1 but to 2 and maybe that is where the trouble is. 
> Is that ok that weights add up not to one?
> I added nscf.out file just in case. 
> 
> Best regards, Mikhail 
> 
> 
> 
> ___
> Quantum ESPRESSO is supported by MaX (www.max-centre.eu/quantum-espresso)
> users mailing list users@lists.quantum-espresso.org
> https://lists.quantum-espresso.org/mailman/listinfo/users

-- 

Giovanni Cantele, PhD

CNR-SPIN
c/o Dipartimento di Fisica
Universita' di Napoli "Federico II"
Complesso Universitario M. S. Angelo - Ed. 6
Via Cintia, I-80126, Napoli, Italy

e-mail: giovanni.cant...@spin.cnr.it
gcant...@gmail.com
Phone: +39 081 676910
Skype contact: giocan74
Web page: https://sites.google.com/view/giovanni-cantele

___
Quantum ESPRESSO is supported by MaX (www.max-centre.eu/quantum-espresso)
users mailing list users@lists.quantum-espresso.org
https://lists.quantum-espresso.org/mailman/listinfo/users

Re: [QE-users] kpoints.x input file

2019-07-12 Thread Giovanni Cantele
just run it interactively!

it asks:
i) the Bravais lattice (form 1 to 14)
ii) the name of an output file (mesh_k is the default)
iii) some structural parameters (e.g. celldm(3) for cases where  a /= c)
iv) the mesh (e.g. 4 4 4)
v) the mesh shift ( 0 0 0 if Gamma point has to be included in your mesh)
vi) a final variable, set to false or true (usually true is what you need), 
according to whether you want to write the full mesh or
give a weight different from 1 to some k-points by grouping those which are 
equivalent

Giovanni

> On 12 Jul 2019, at 14:51, Rajender Prasad Tiwari  
> wrote:
> 
> Dear All,
> 
> I want to generate k-points in
> K_POINTS tpiba | crystal | tpiba_b | crystal_b | tpiba_c | crystal_c 
> nks <https://www.quantum-espresso.org/Doc/INPUT_PW.html#idm1108>  
>  <>
>  xk_x(1) <https://www.quantum-espresso.org/Doc/INPUT_PW.html#idm1113> 
>  xk_y(1) <https://www.quantum-espresso.org/Doc/INPUT_PW.html#idm1114>
> xk_z(1) <https://www.quantum-espresso.org/Doc/INPUT_PW.html#idm1115>wk(1) 
> <https://www.quantum-espresso.org/Doc/INPUT_PW.html#idm1116> 
>  xk_x(2) <https://www.quantum-espresso.org/Doc/INPUT_PW.html#idm1113> 
>  xk_y(2) <https://www.quantum-espresso.org/Doc/INPUT_PW.html#idm1114>
> xk_z(2) <https://www.quantum-espresso.org/Doc/INPUT_PW.html#idm1115>wk(2) 
> <https://www.quantum-espresso.org/Doc/INPUT_PW.html#idm1116> 
>  . . .
>  xk_x(nks) <https://www.quantum-espresso.org/Doc/INPUT_PW.html#idm1113>   
>  xk_y(nks) <https://www.quantum-espresso.org/Doc/INPUT_PW.html#idm1114>   
>xk_z(nks) <https://www.quantum-espresso.org/Doc/INPUT_PW.html#idm1115> 
>  wk(nks) <https://www.quantum-espresso.org/Doc/INPUT_PW.html#idm1116>
> 
> format and I believe this can be done using kpoints.x in QE. I tried to find 
> document describing the input format, however, I couldn't find it.
> Could you kindly help me with kpoints.x input file format or with any 
> document describing kpoints.x input format?
> 
> Many thanks!
> -- 
> Regards,
> Rajender Tiwari
> 
> Ph.D. Scholar
> Special Center for Nano Science
> Jawaharlal Nehru University, New Delhi
> #+91-9868970410/ 9873513329
> 
> ___
> Quantum ESPRESSO is supported by MaX (www.max-centre.eu/quantum-espresso)
> users mailing list users@lists.quantum-espresso.org
> https://lists.quantum-espresso.org/mailman/listinfo/users

-- 

Giovanni Cantele, PhD

CNR-SPIN
c/o Dipartimento di Fisica
Universita' di Napoli "Federico II"
Complesso Universitario M. S. Angelo - Ed. 6
Via Cintia, I-80126, Napoli, Italy

e-mail: giovanni.cant...@spin.cnr.it
gcant...@gmail.com
Phone: +39 081 676910
Skype contact: giocan74
Web page: https://sites.google.com/view/giovanni-cantele

___
Quantum ESPRESSO is supported by MaX (www.max-centre.eu/quantum-espresso)
users mailing list users@lists.quantum-espresso.org
https://lists.quantum-espresso.org/mailman/listinfo/users

Re: [QE-users] 2D band surface plot

2019-05-03 Thread Giovanni Cantele
you should provide the input file of the nscf calculation to figure out if 
something strange is happening
Giovanni

> On 30 Apr 2019, at 08:50, elch...@auth.gr wrote:
> 
> 
> Quoting Oleksandr Motornyi :
> 
>> Hello Eleni
>> 
>> Could you please provide an input as well as the strange outputs? It is hard 
>> to give any advice or example without knowing what are you dealing with 
>> exactly.
>> 
>> Regards,
>> 
>> Oleksandr
>> 
>> Oleksandr Motornyi
>> PhD
>> 
>> Laboratoire des Solides Irradies
>> Ecole Polytechnique (Palaiseau, France)
>> 
>> On 29/04/19 18:12, elch...@auth.gr wrote:
>>> Hello everyone,
>>> 
>>> Do you have an example with a grid used for 2D surface plot in bands.x? I 
>>> am getting some really strange output.
>>> 
>>> Regards,
>>> 
>>> Eleni
>>> 
>>> 
>> 
>> ___
>> Quantum Espresso is supported by MaX (www.max-centre.eu/quantum-espresso)
>> users mailing list users@lists.quantum-espresso.org
>> https://lists.quantum-espresso.org/mailman/listinfo/users
> 
> 
> 
> -- 
> Dr. Eleni Chatzikyriakou
> Computational Physics lab
> Aristotle University of Thessaloniki
> elch...@auth.gr - tel:+30 2310 998109
> ___
> Quantum Espresso is supported by MaX (www.max-centre.eu/quantum-espresso)
> users mailing list users@lists.quantum-espresso.org
> https://lists.quantum-espresso.org/mailman/listinfo/users

-- 

Giovanni Cantele, PhD

CNR-SPIN
c/o Dipartimento di Fisica
Universita' di Napoli "Federico II"
Complesso Universitario M. S. Angelo - Ed. 6
Via Cintia, I-80126, Napoli, Italy

e-mail: giovanni.cant...@spin.cnr.it
gcant...@gmail.com
Phone: +39 081 676910
Skype contact: giocan74
Web page: https://sites.google.com/view/giovanni-cantele

___
Quantum Espresso is supported by MaX (www.max-centre.eu/quantum-espresso)
users mailing list users@lists.quantum-espresso.org
https://lists.quantum-espresso.org/mailman/listinfo/users

Re: [QE-users] Enquiry about the Computation of Graphene/Co

2019-04-16 Thread Giovanni Cantele
in the Dropbox file there is only the band structure plot, please provide also 
the input files
Giovanni

> On 16 Apr 2019, at 11:38, Kenan Song  wrote:
> 
> Dear Dr. Cantele,
> 
> Thank you for the reply.
> 
> I have already compressed my files and share it with you through dropbox. 
> Would you please have a look at it and give me some suggestions? Thank you 
> very much for your help.
> 
> Kind regards,
> 
> Kenan Song
> 
> On Tue, Apr 16, 2019 at 10:58 AM Giovanni Cantele 
> mailto:giovanni.cant...@spin.cnr.it>> wrote:
> Dear Kenan,
> 
> unfortunately, it is not that easy to get access to the files that you tried 
> to share one by one through Dropbox (Dropbox asks to request access). It 
> would be much
> more efficient to zip them in a single file.
> 
> Maybe, after providing the input files, it would be much easier to answer you 
> question. It would be quite important to understand what are the differences
> you find between the VASP and the QE band structure. Indeed, if you get 
> completely different bands, it should be quite unlikely that this can be 
> attributed
> to the pseudo potentials. Maybe one of the two calculations has a wrong 
> setup. A plot of the two band structures together with the input files would 
> be
> helpful in this respect.
> 
> Giovanni
> 
>> On 15 Apr 2019, at 21:24, Kenan Song > <mailto:kenan.s...@kaust.edu.sa>> wrote:
>> 
>> Dear All,
>> 
>> I am trying to use Quantum Espresso (QE) to compute Graphene/Co system. 
>> However, QE gives me a quite different band structure from that given by 
>> VASP. I think that I did not choose the proper pseudopotential to describe 
>> this system. I upload my computational files into dropbox so that you could 
>> refer to them through the dropbox.
>> 
>> Would anyone please help me double check my input files from QE and see 
>> whether the problem comes from the pseudopotential or something else? If the 
>> problem mainly comes from the pseudopotential, would anyone please recommend 
>> a proper pseudopotential for Graphene/Co system to me? Thank you very much 
>> for your help.
>> 
>> Kind regards,
>> 
>> Kenan Song
>> 
>> This message and its contents, including attachments are intended solely for 
>> the original recipient. If you are not the intended recipient or have 
>> received this message in error, please notify me immediately and delete this 
>> message from your computer system. Any unauthorized use or distribution is 
>> prohibited. Please consider the environment before printing this 
>> email._______
>> users mailing list
>> users@lists.quantum-espresso.org <mailto:users@lists.quantum-espresso.org>
>> https://lists.quantum-espresso.org/mailman/listinfo/users 
>> <https://lists.quantum-espresso.org/mailman/listinfo/users>
> -- 
> 
> Giovanni Cantele, PhD
> 
> CNR-SPIN
> c/o Dipartimento di Fisica
> Universita' di Napoli "Federico II"
> Complesso Universitario M. S. Angelo - Ed. 6
> Via Cintia, I-80126, Napoli, Italy
> 
> e-mail: giovanni.cant...@spin.cnr.it <mailto:giovanni.cant...@spin.cnr.it>
> gcant...@gmail.com <mailto:gcant...@gmail.com>
> Phone: +39 081 676910
> Skype contact: giocan74
> Web page: https://sites.google.com/view/giovanni-cantele 
> <https://sites.google.com/view/giovanni-cantele>
> ___
> users mailing list
> users@lists.quantum-espresso.org <mailto:users@lists.quantum-espresso.org>
> https://lists.quantum-espresso.org/mailman/listinfo/users 
> <https://lists.quantum-espresso.org/mailman/listinfo/users>
> This message and its contents, including attachments are intended solely for 
> the original recipient. If you are not the intended recipient or have 
> received this message in error, please notify me immediately and delete this 
> message from your computer system. Any unauthorized use or distribution is 
> prohibited. Please consider the environment before printing this 
> email.___
> users mailing list
> users@lists.quantum-espresso.org
> https://lists.quantum-espresso.org/mailman/listinfo/users

-- 

Giovanni Cantele, PhD

CNR-SPIN
c/o Dipartimento di Fisica
Universita' di Napoli "Federico II"
Complesso Universitario M. S. Angelo - Ed. 6
Via Cintia, I-80126, Napoli, Italy

e-mail: giovanni.cant...@spin.cnr.it
gcant...@gmail.com
Phone: +39 081 676910
Skype contact: giocan74
Web page: https://sites.google.com/view/giovanni-cantele

___
users mailing list
users@lists.quantum-espresso.org
https://lists.quantum-espresso.org/mailman/listinfo/users

Re: [QE-users] Enquiry about the Computation of Graphene/Co

2019-04-16 Thread Giovanni Cantele
Dear Kenan,

unfortunately, it is not that easy to get access to the files that you tried to 
share one by one through Dropbox (Dropbox asks to request access). It would be 
much
more efficient to zip them in a single file.

Maybe, after providing the input files, it would be much easier to answer you 
question. It would be quite important to understand what are the differences
you find between the VASP and the QE band structure. Indeed, if you get 
completely different bands, it should be quite unlikely that this can be 
attributed
to the pseudo potentials. Maybe one of the two calculations has a wrong setup. 
A plot of the two band structures together with the input files would be
helpful in this respect.

Giovanni

> On 15 Apr 2019, at 21:24, Kenan Song  wrote:
> 
> Dear All,
> 
> I am trying to use Quantum Espresso (QE) to compute Graphene/Co system. 
> However, QE gives me a quite different band structure from that given by 
> VASP. I think that I did not choose the proper pseudopotential to describe 
> this system. I upload my computational files into dropbox so that you could 
> refer to them through the dropbox.
> 
> Would anyone please help me double check my input files from QE and see 
> whether the problem comes from the pseudopotential or something else? If the 
> problem mainly comes from the pseudopotential, would anyone please recommend 
> a proper pseudopotential for Graphene/Co system to me? Thank you very much 
> for your help.
> 
> Kind regards,
> 
> Kenan Song
> 
> This message and its contents, including attachments are intended solely for 
> the original recipient. If you are not the intended recipient or have 
> received this message in error, please notify me immediately and delete this 
> message from your computer system. Any unauthorized use or distribution is 
> prohibited. Please consider the environment before printing this 
> email.___
> users mailing list
> users@lists.quantum-espresso.org
> https://lists.quantum-espresso.org/mailman/listinfo/users

-- 

Giovanni Cantele, PhD

CNR-SPIN
c/o Dipartimento di Fisica
Universita' di Napoli "Federico II"
Complesso Universitario M. S. Angelo - Ed. 6
Via Cintia, I-80126, Napoli, Italy

e-mail: giovanni.cant...@spin.cnr.it
gcant...@gmail.com
Phone: +39 081 676910
Skype contact: giocan74
Web page: https://sites.google.com/view/giovanni-cantele

___
users mailing list
users@lists.quantum-espresso.org
https://lists.quantum-espresso.org/mailman/listinfo/users

Re: [QE-users] query on number of k points in nscf

2019-04-08 Thread Giovanni Cantele
Dear Lorenzo,
thanks you for your prompt response. This is actually the case:
 Parallel version (MPI & OpenMP), running on2400 processor cores
 Number of MPI processes:  2400
 Threads/MPI process: 1

 MPI processes distributed on   100 nodes
 K-points division: npool =   2
 R & G space division:  proc/nbgrp/npool/nimage =1200
 wavefunctions fft division:  Y-proc x Z-proc =   2600
 wavefunctions fft division:  task group distribution
  #TGx Z-proc =   2600


So, it is explained also why the time used to compute 10 k-points coincides 
with the whole calculation
computational time.

Maybe, for the sake of clarity, it could be really explanatory if in the output 
in would be explicitly 
mentioned that only the output from pool #1 is give.

Thank you again,

Giovanni



> On 8 Apr 2019, at 12:37, Lorenzo Paulatto  wrote:
> 
> Dear Giovanni,
> if you are using pools, only a fraction of the k-points will be enumerated in 
> the NSCF calculation.
> kind regards
> 
> On 08/04/2019 12:07, Giovanni Cantele wrote:
>> Dear all,
>> I’ve experienced a “strange” (?) behaviour in an NSCF calculation. I’m using 
>> Quantum-ESPRESSO v. 6.4 on CINECA-Marconi.
>> The calculation is with no symmetry ("No symmetry found”) and lists 19 
>> k-points at the beginning of the calculation.
>> In the input the k-point specification is obtained through
>> K_POINTS { automatic }
>> 36  1  1   0 0 0
>> If I
>> grep "k =.*bands " NSCF.out | wc
>> 19 lines are correctly found in the output.
>> However
>> grep "Computing" NSCF.out
>> returns
>>  Computing kpt #: 1
>>  Computing kpt #: 2
>>  Computing kpt #: 3
>>  Computing kpt #: 4
>>  Computing kpt #: 5
>>  Computing kpt #: 6
>>  Computing kpt #: 7
>>  Computing kpt #: 8
>>  Computing kpt #:
>>  Computing kpt #:10
>> as if only 10 k-points were considered in the calculation. Even more 
>> strangely, by analizing the cpu-time information, I get
>>  Computing kpt #: 1
>>  c_bands: 56 eigenvalues not converged
>>  total cpu time spent up to now is13728.9 secs
>> [….]
>>  Computing kpt #:10
>>  c_bands: 48 eigenvalues not converged
>>  total cpu time spent up to now is   133710.2 secs
>> that is, the time for 10 k-points, is about 10 times that for a single 
>> k-point. However, the full calculation takes
>>  PWSCF:   1d13h59m CPU   1d15h18m WALL
>> about 1d and 13 hours, that is 37 hours that correspond to about 133000 
>> seconds. So it seems that the whole calculation
>> takes the same time as taken for the first 10 k-points.
>> So, the questions are:
>> 1) why only 10 “Computing ….” lines are printed in output?
>> 2) How can I be sure that the code effectively computed the k-points from 11 
>> to 19 (and if it did not do it, what are
>> the eigenvalues printed at the and of the output for those k-points)?
>> Maybe I’m missing something very trivial (in the case I apologise for 
>> that!), but I cannot figure out what!
>> I also checked PW/src/c_bands.f90 and it seems that the line
>>  IF ( iverbosity > 0 ) WRITE( stdout, 9001 ) ik
>> is within the loop
>> k_loop: DO ik = ik_+1, nks
>> so the printing of the message with format
>> 9001 FORMAT(/' Computing kpt #: ',I5 )
>> should occur for either all k-points or none of them.
>> I thank you in advance for any suggestion.
>> Giovanni
>> -- 
>> Giovanni Cantele, PhD
>> CNR-SPIN
>> c/o Dipartimento di Fisica
>> Universita' di Napoli "Federico II"
>> Complesso Universitario M. S. Angelo - Ed. 6
>> Via Cintia, I-80126, Napoli, Italy
>> e-mail: giovanni.cant...@spin.cnr.it <mailto:giovanni.cant...@spin.cnr.it> 
>> <mailto:giovanni.cant...@spin.cnr.it <mailto:giovanni.cant...@spin.cnr.it>>
>> gcant...@gmail.com <mailto:gcant...@gmail.com> <mailto:gcant...@gmail.com 
>> <mailto:gcant...@gmail.com>>
>> Phone: +39 081 676910
>> Skype contact: giocan74
>> Web page:https://sites.google.com/view/giovanni-cantele 
>> <https://sites.google.com/view/giovanni-cantele>
>> ___
>> users mailing list
>> users@lists.quantum-espresso.org <mailto:users@lists.quantum-espresso.org>
>> https://lists.quantum-espresso.org/mailman/listinfo/users 
>> <https://lists.quantum-espresso.org/mailman/listinfo/us

[QE-users] query on number of k points in nscf

2019-04-08 Thread Giovanni Cantele
Dear all,

I’ve experienced a “strange” (?) behaviour in an NSCF calculation. I’m using 
Quantum-ESPRESSO v. 6.4 on CINECA-Marconi.

The calculation is with no symmetry ("No symmetry found”) and lists 19 k-points 
at the beginning of the calculation.
In the input the k-point specification is obtained through
K_POINTS { automatic }
36  1  1   0 0 0 

If I
grep "k =.*bands " NSCF.out | wc
19 lines are correctly found in the output.
However
grep "Computing" NSCF.out 
returns
 Computing kpt #: 1
 Computing kpt #: 2
 Computing kpt #: 3
 Computing kpt #: 4
 Computing kpt #: 5
 Computing kpt #: 6
 Computing kpt #: 7
 Computing kpt #: 8
 Computing kpt #: 
 Computing kpt #:10
as if only 10 k-points were considered in the calculation. Even more strangely, 
by analizing the cpu-time information, I get
 Computing kpt #: 1
 c_bands: 56 eigenvalues not converged
 total cpu time spent up to now is13728.9 secs
[….]
 Computing kpt #:10
 c_bands: 48 eigenvalues not converged
 total cpu time spent up to now is   133710.2 secs

that is, the time for 10 k-points, is about 10 times that for a single k-point. 
However, the full calculation takes
 PWSCF:   1d13h59m CPU   1d15h18m WALL
about 1d and 13 hours, that is 37 hours that correspond to about 133000 
seconds. So it seems that the whole calculation
takes the same time as taken for the first 10 k-points.

So, the questions are:

1) why only 10 “Computing ….” lines are printed in output?
2) How can I be sure that the code effectively computed the k-points from 11 to 
19 (and if it did not do it, what are
the eigenvalues printed at the and of the output for those k-points)?


Maybe I’m missing something very trivial (in the case I apologise for that!), 
but I cannot figure out what!

I also checked PW/src/c_bands.f90 and it seems that the line 
 IF ( iverbosity > 0 ) WRITE( stdout, 9001 ) ik
is within the loop
k_loop: DO ik = ik_+1, nks
so the printing of the message with format
9001 FORMAT(/' Computing kpt #: ',I5 )
should occur for either all k-points or none of them.

I thank you in advance for any suggestion.

Giovanni


-- 

Giovanni Cantele, PhD

CNR-SPIN
c/o Dipartimento di Fisica
Universita' di Napoli "Federico II"
Complesso Universitario M. S. Angelo - Ed. 6
Via Cintia, I-80126, Napoli, Italy

e-mail: giovanni.cant...@spin.cnr.it
gcant...@gmail.com
Phone: +39 081 676910
Skype contact: giocan74
Web page: https://sites.google.com/view/giovanni-cantele

___
users mailing list
users@lists.quantum-espresso.org
https://lists.quantum-espresso.org/mailman/listinfo/users

Re: [QE-users] i have doubt

2019-03-22 Thread Giovanni Cantele
1) please include your affiliation to the message, as usually done by all 
people taking part to this forum

2) I would use more meaningful and explanatory subjects, this usually results 
in a faster and more appropriate answer

This being said, the code is complaining because a run with calculation = 
‘bands’ should follow a self-consistent or relax
run. The latter saves a number of files needed for the next calculations. Those 
files must exist and being accessible/readable
by the code when the band calculation run is started.

Maybe you either did not perform the scf run, or deleted some of the needed 
files, or else those files are stored in a directory
different from “./“ (as you specified in the outdoor variable) or with a prefix 
name different from that specified in the input
file you sent.

It is quite hard, if not impossible, to provide more help without more details.

Giovanni

PS in your run you might need to set the value of ecutrho (currently not 
specified) to a value larger than
its default, 4*ecutwfc, because you’re using PAW pseudo potentials

> On 22 Mar 2019, at 10:39, yasmin kani  wrote:
> 
> i have an error while running band structure calculation for InSb. i need a 
> clarification
> ___
> users mailing list
> users@lists.quantum-espresso.org
> https://lists.quantum-espresso.org/mailman/listinfo/users

-- 

Giovanni Cantele, PhD

CNR-SPIN
c/o Dipartimento di Fisica
Universita' di Napoli "Federico II"
Complesso Universitario M. S. Angelo - Ed. 6
Via Cintia, I-80126, Napoli, Italy

e-mail: giovanni.cant...@spin.cnr.it
gcant...@gmail.com
Phone: +39 081 676910
Skype contact: giocan74
Web page: https://sites.google.com/view/giovanni-cantele

___
users mailing list
users@lists.quantum-espresso.org
https://lists.quantum-espresso.org/mailman/listinfo/users

Re: [QE-users] Error in average.x

2019-03-12 Thread Giovanni Cantele
quite strange, the file looks ok, are you sure that this is just the input 
you’re using? Maybe any special (hidden) character
is present in the file?

I tried to run average.x with your input and it works, in that it passes the 
line that gives you the error and of course complains
about the inexistent WS2.pot file (as it should do).

Giovanni

> On 12 Mar 2019, at 12:41, Ubaid Mohd  wrote:
> 
> Dear All QE Users,
> I am using the average.x now and the read the input file as follow :
> 1
> WS2.pot
> 1.D0
> 1440
> 3
> 1.461144408
> but when run this it has the follow error:
> 
>  Error in routine average (1):
>  nfile is wrong
> 
> nfile should be right, I have read the source file, only when nfile <=0 or >7 
> will show this error, but in my input file it is set to be 1. Would you 
> please give me some suggestion on what might be the issue? Thanks very much.
> 
> Best wishes
> Mohammad Ubaid
> PhD Research Scholar
> Department of Physics
> Jamia Millia Islamia University
> New Delhi - 110025
> ___
> users mailing list
> users@lists.quantum-espresso.org
> https://lists.quantum-espresso.org/mailman/listinfo/users

-- 

Giovanni Cantele, PhD

CNR-SPIN
c/o Dipartimento di Fisica
Universita' di Napoli "Federico II"
Complesso Universitario M. S. Angelo - Ed. 6
Via Cintia, I-80126, Napoli, Italy

e-mail: giovanni.cant...@spin.cnr.it
gcant...@gmail.com
Phone: +39 081 676910
Skype contact: giocan74
Web page: https://sites.google.com/view/giovanni-cantele

___
users mailing list
users@lists.quantum-espresso.org
https://lists.quantum-espresso.org/mailman/listinfo/users

Re: [QE-users] NEB calculation no symmetry found

2019-02-14 Thread Giovanni Cantele
from PW/src/symm_base.f90 :

  ! ... these are acceptance criteria
  !
  REAL(DP), PARAMETER :: eps1 = 1.0d-6, eps2 = 1.0d-5


I guess you can change those values and recompile the code to force it to use a 
certain tolerance
Giovanni


> On 14 Feb 2019, at 09:58, Fabrizio Cossu  wrote:
> 
> 
> 
> On Thu, 14 Feb 2019 at 17:37, Paolo Giannozzi  <mailto:p.gianno...@gmail.com>> wrote:
> On Thu, Feb 14, 2019 at 8:34 AM Offermans Willem  <mailto:willem.offerm...@vito.be>> wrote:
> 
> So neb.x is able to detect symmetry. Only in my particular case, it is not 
> able to and I wonder why.
> 
> the reason has been explained no less than 1001 times: if the code does not 
> find a symmetry, it's not there, according to the criteria implemented in the 
> code. 
> 
> Paolo
> 
> A DFT code should recognise symmetry operations within a certain tolerance 
> given the coordinates of the atoms. A quick search ("symmetry", "tolerance") 
> in the pw.x input options does not return any result for a tunable parameter. 
> What is the default value for the tolerance, and what should I do if I had to 
> define a certain tolerance?
> Thank you,
> Fabrizio
> 
> -- 
> Fabrizio Cossu
> postdoctoral fellow at APCTP (Asia Pacific Center for Theoretical Physics),
> Hogil Kim Memorial Building #501 
> POSTECH, 67 Cheongam-Ro, Nam-Gu, 
> Pohang-si, Gyeongsangbuk-do,
> 790-784 (37673), Republic of Korea
>|
>   .. .. .. |  ..   ===
>   ,| || |  |  ||   http://www.apctp.org/?JrgId=16 
> <http://www.apctp.org/?JrgId=16>
>   `^ |' `' `' |'   ===
>  ||
> _______
> users mailing list
> users@lists.quantum-espresso.org
> https://lists.quantum-espresso.org/mailman/listinfo/users

-- 

Giovanni Cantele, PhD

CNR-SPIN
c/o Dipartimento di Fisica
Universita' di Napoli "Federico II"
Complesso Universitario M. S. Angelo - Ed. 6
Via Cintia, I-80126, Napoli, Italy

e-mail: giovanni.cant...@spin.cnr.it
gcant...@gmail.com
Phone: +39 081 676910
Skype contact: giocan74
Web page: https://sites.google.com/view/giovanni-cantele

___
users mailing list
users@lists.quantum-espresso.org
https://lists.quantum-espresso.org/mailman/listinfo/users

Re: [QE-users] Wrong number of k-points?

2019-01-31 Thread Giovanni Cantele
The symmetry k -> -k is still present even though your system has no spatial 
symmetry. As such the number of k-points is halved.
Giovanni

PS from a quick google search I found for example this document you might find 
of help
https://people.sissa.it/~degironc/FIST/Slides/8%20Symmetry.pdf

> On 31 Jan 2019, at 10:09, Laurens Siemons  wrote:
> 
> Dear QE users,
> 
> I'm running a relax calculation and I encountered something that I do not 
> understand in the output file. I used a 2 2 1 1 1 0 Monkhorst-Pack grid in my 
> input file (which is shown below) but the output file tells me I only have 2 
> k-points:
> 
> 
>116   H   tau( 116) = (   3.8845153   0.8033970  -0.3177301  )
>117   H   tau( 117) = (   3.8658215   1.2769290  -0.3107519  )
>118   H   tau( 118) = (   4.1682098   1.0557090  -0.6030099  )
> 
>  number of k points= 2
>cart. coord. in units 2pi/alat
> k(1) = (   0.000   0.083  -0.0984507), wk =   1.000
> k(2) = (   0.000   0.083   0.0984507), wk =   1.000
> 
>  Dense  grid:  5866317 G-vectors FFT dimensions: ( 180, 160, 480)
> 
>  Smooth grid:  1484065 G-vectors FFT dimensions: ( 108,  96, 320)
> 
> I would expect at least 4 k-point right? The system possesses no symmetry.
> 
> Thanks in advance,
> Laurens Siemons,
> PhD, University of Antwerp
> 
> 
>   calculation = 'relax'
>   restart_mode = 'from_scratch'
>   prefix = 'a101mpa1'
>   tprnfor = .true.
>   pseudo_dir = 
> '/data/antwerpen/204/vsc20442/pseudo/pslibrary.1.0.0/wc/PSEUDOPOTENTIALS'
>   outdir = '/scratch/antwerpen/204/vsc20442/TMP'
>   wf_collect = .true.
>   nstep = 100
> /
> 
>   ibrav = 0
>   A =3.77449
>   nat = 118
>   ntyp = 5
>   ecutwfc = 60
>   ecutrho = 600
>   spline_ps = .true.
> /
> 
>   electron_maxstep = 500
>   diagonalization = 'david'
>   conv_thr =  1.0d-10
>   mixing_beta = 0.70
> /
> 
>   ion_dynamics = 'bfgs'
> /
> ATOMIC_SPECIES
>O   15.99900  O.wc-n-kjpaw_psl.1.0.0.UPF
>   Ti   47.86700  Ti.wc-spn-kjpaw_psl.1.0.0.UPF
>H   1.0079H.wc-kjpaw_psl.1.0.0.UPF
>C   12.0107   C.wc-n-kjpaw_psl.1.0.0.UPF
>P   30.973761 P.wc-n-kjpaw_psl.1.0.0.UPF
> ATOMIC_POSITIONS {crystal}
> Ti  0   0.000.026316 0 0 0
> Ti  2/3 0.000.026316 0 0 0
> Ti  1/3 0.000.026316 0 0 0
> Ti  1/6 1/2 0.026316 0 0 0
> Ti  1/2 1/2 0.026316 0 0 0
> Ti  5/6 1/2 0.026316 0 0 0
> Ti  0   0.9958560.266326
> Ti  1/3 0.9958560.266326
> Ti  2/3 0.9958560.266326
> Ti  1/6 0.4958560.266326
> Ti  1/2 0.4958560.266326
> Ti  5/6 0.4958560.266326
> Ti  0   0.9892020.145250
> Ti  1/3 0.9892020.145250
> ...
> ...
> ...
> CELL_PARAMETERS {alat}
>   0.000   3.000   0.000
>   1.000  -0.000  -2.539341738883929
>   8.551   0.000   0.000
> K_POINTS {automatic}
> 2 2 1 1 1 0
> 
>   
>   
> 
>  
> <http://www.avg.com/email-signature?utm_medium=email_source=link_campaign=sig-email_content=webmail>
>  Virus-free. www.avg.com 
> <http://www.avg.com/email-signature?utm_medium=email_source=link_campaign=sig-email_content=webmail>
>  
> ___
> users mailing list
> users@lists.quantum-espresso.org <mailto:users@lists.quantum-espresso.org>
> https://lists.quantum-espresso.org/mailman/listinfo/users 
> <https://lists.quantum-espresso.org/mailman/listinfo/users>
-- 

Giovanni Cantele, PhD

CNR-SPIN
c/o Dipartimento di Fisica
Universita' di Napoli "Federico II"
Complesso Universitario M. S. Angelo - Ed. 6
Via Cintia, I-80126, Napoli, Italy

e-mail: giovanni.cant...@spin.cnr.it
gcant...@gmail.com
Phone: +39 081 676910
Skype contact: giocan74
Web page: https://sites.google.com/view/giovanni-cantele

___
users mailing list
users@lists.quantum-espresso.org
https://lists.quantum-espresso.org/mailman/listinfo/users

Re: [QE-users] ibrav number check

2018-11-19 Thread Giovanni Cantele
maybe you’re visualising the conventional, rather than the primitive unit cell.
Try to set Display -> Primitive Cell Mode

The bct unit cel should show up.

Giovanni

-- 

Giovanni Cantele, PhD

CNR-SPIN
c/o Dipartimento di Fisica
Universita' di Napoli "Federico II"
Complesso Universitario M. S. Angelo - Ed. 6
Via Cintia, I-80126, Napoli, Italy

e-mail: giovanni.cant...@spin.cnr.it
gcant...@gmail.com
Phone: +39 081 676910
Skype contact: giocan74
Web page: https://sites.google.com/view/giovanni-cantele

> On 19 Nov 2018, at 19:38, Chan, Edmund  wrote:
> 
> Dear all users,
>  
> I am currently trying to relax a methylammonium lead iodide CH3NH3PbI3 
> structure which has tetragonal structure I4/mcm. Thus, in my input file, I 
> select ibrav = 7 (body centred teragonal). However, as I visualise the input 
> and output on Xcrysden, the unit cell becomes orthorhombic. Why is that?
>  
>  
> 
>title = 'MAPI' ,
>  calculation = 'relax' ,
> restart_mode = 'from_scratch' ,
>   outdir = './' ,
>   pseudo_dir = './' ,
>   prefix = 'CH3NH3PbI3' ,
> /
> 
>ibrav = 7,
>A = 8.83920,
>C = 12.6948,
>  nat = 12,
> ntyp = 5,
>  ecutwfc = 80 ,
>ecutrho = 800,
> /
> 
> electron_maxstep = 5000 ,
> conv_thr = 1.0d-6 ,
>  mixing_beta = 0.3 ,
> /
> 
> ion_dynamics = 'bfgs' ,
> /
> ATOMIC_SPECIES
> C   12.01070  c_pbesol_v1.2.uspp.F.UPF
> H1.00794  H.pbesol-rrkjus_psl.0.1.UPF
> N   14.00670  N.pbesol-theos.UPF
>Pb  207.2  Pb.pbesol-dn-kjpaw_psl.0.2.2.UPF
> I  126.90450  i_pbesol_v1.uspp.F.UPF
> ATOMIC_POSITIONS alat
>Pb  0.00.00.5   
> I  0.00.00.25000   
> I  0.294200.205800.5   
> I -0.29420   -0.207600.5   
> C  0.544000.921000.78400   
> H  0.609201.002100.76990   
> H  0.557300.845500.72460   
> H  0.551500.875200.86430   
> N  0.389000.987000.76700   
> H  0.324700.913900.73030   
> H  0.381401.068100.73430   
> H  0.339300.990800.83100   
> K_POINTS automatic
>   1 1 1   0 0 0
>  
> Thank you all.
>  
>  
> Kind regards,
>  
> Edmund Chan
> 
> Edmund Chan
> PhD Renewable Energy Student
> Environment & Sustainablity Institute
> College of Engineering, Mathematics and Physical Sciences
> University of Exeter, Penryn Campus
> Penryn, Cornwall
> United Kingdom
> TR10 9FE
> 
> Tel: +44 (0)7455 235 701 
> Email: thc...@exeter.ac.uk <mailto:thc...@exeter.ac.uk>
> LinkedIn: https://uk.linkedin.com/in/edmundchan1 
> <https://uk.linkedin.com/in/edmundchan1>
> 
>  
> 
>  
> ___
> users mailing list
> users@lists.quantum-espresso.org <mailto:users@lists.quantum-espresso.org>
> https://lists.quantum-espresso.org/mailman/listinfo/users 
> <https://lists.quantum-espresso.org/mailman/listinfo/users>
___
users mailing list
users@lists.quantum-espresso.org
https://lists.quantum-espresso.org/mailman/listinfo/users

Re: [QE-users] Unit for the output of average.x

2018-10-26 Thread Giovanni Cantele
Are you sure about you Fermi level? How did you get it? 

GIOVANNI 

Inviato da iPhone

> Il giorno 26 ott 2018, alle ore 19:33, Dingfu Shao  ha 
> scritto:
> 
> Dear Giovanni and Paolo,
> 
> Thanks very much for your suggestions.
> 
> I reconsidered the definition of the local density of states (say, 
> LD(x,y,z,E)). Since it is "local", the unit of it should be states/eV/bohr^3 
> or electrons/eV/bohr^3. Therefore, the integration of it within an energy 
> window should lead to the charge density in this energy window: 
> electrons/bohr^3 . Therefore, if we choose the energy window from the lowest 
> energy to the Fermi energy, we should get exactly the total charge density.
> 
> So I did some tests following Giovanni's suggestion, using a simple case of 
> momolayer MoS2, which has "number of electrons  =   26.00". The area of its 
> xy plane is S. For those tests, the previous scf and nscf calculations are 
> the same, with LDA USPP,  the occupation= 'fixed', and k-points of 40*40*1 
> for scf and 100*100*1 for nscf.
> 
> 1. I  calculated the total charge density (rho(x,y,z)) using pp.x with 
> plot_num = 0, and then calculate the planar average of it (rho_avg(z)). Then 
> I integrated it by \int (S* rho_avg(z)) dz and I got 25.89. It is close to 
> 26.00, maybe a more accurate value can be obtained by a calculation with 
> denser k points.
> 
> 2. I calculated the integrated local density of states  ( ILD(x,y,z)) from 
> -65 eV (this energy is below the lower band) to Fermi energy using pp.x with 
> plot_num =10, and and then calculate the planar average of it ( ILD_avg(z) ). 
> When I integrated it by \int (S* ILD_avg(z)) dz, I got 29.25, which is much 
> larger than the total number of electrons of 26.
> 
>  So it seems the the test 2 doesn't correctly reflect the  reality. I am not 
> sure it is due to something happened with plot_num = 10 in pp.x, or just I 
> understand this incorrectly. 
> 
> Any suggestions? Thank you very much!
> 
> Best,
> 
> Ding-Fu
> 
> 
> 
> 
> 
> 
>> From: Giovanni Cantele 
>> To: Quantum Espresso users Forum 
>> Cc: 
>> Bcc: 
>> Date: Fri, 26 Oct 2018 09:51:40 +0200
>> Subject: Re: [QE-users] Unit for the output of average.x
>> Dear Ding-Fu,
>> as far as I remember there is a surface factor that you need to adjust units.
>> For sure on the ascissa axis the coordinate is in bohr.
>> The planar average give you back a quantity with the same units as the 
>> averaged quantity
>> (e.g. if you star from charge density in electrons/bohr^3 you get an 
>> averaged electron density in electrons/bohr^3),
>> being defined as (let us suppose that you average in the plane defined by a1 
>> and a2 vectors):
>> rho_avg(z) = ( 1 / S ) * integral( dx dy rho(x,y,z) )
>> That means that if you perform 
>>  integral( dz rho_avg(z) )
>> you get 
>> number of electrons / S
>> If you need number of electrons than just multiply by S with
>> S = cross_product( a1, a2 )
>> (in bohr^2)
>> Just try, better if you do it with the total charge density, to check if the 
>> integral returns you
>> the number of electrons.
>> I’m sorry but I cannot check directly if I remember correctly at the moment, 
>> but
>> this should work.
>> Giovanni
>> -- 
>> Giovanni Cantele, PhD
>> CNR-SPIN
>> c/o Dipartimento di Fisica
>> Universita' di Napoli "Federico II"
>> Complesso Universitario M. S. Angelo - Ed. 6
>> Via Cintia, I-80126, Napoli, Italy
>> e-mail: giovanni.cant...@spin.cnr.it
>> gcant...@gmail.com
>> Phone: +39 081 676910
>> Skype contact: giocan74
>> Web page: https://sites.google.com/view/giovanni-cantele
>> On 26 Oct 2018, at 03:31, Dingfu Shao  wrote:
>> Dear QE developers and users:
>> I am wondering what should be the unit of the planar average data got from 
>> the average.x
>>  I am calculating the planar average of charge density within a energy 
>> window. What I did is firstly using pp to get the integrated local density 
>> of states (ILDOS) of that energy window with plot_num=10, then using 
>> average.x to get the planar average. 
>> In this case, what is the unit of the second column (say, rho(z)) of  the 
>> output file? I thought since the DOS has a unit of states/eV, the 
>> integration of DOS within a energy window should get some states or 
>> electrons. Then the unit of rho(z) should be electron/bohr. But seems it is 
>> not. In my case the energy window I concerned contains one electron, 
>> However, if I directly integrate  rho(z), I can only get a very small value

Re: [QE-users] Unit for the output of average.x

2018-10-26 Thread Giovanni Cantele
Dear Ding-Fu,

as far as I remember there is a surface factor that you need to adjust units.

For sure on the ascissa axis the coordinate is in bohr.

The planar average give you back a quantity with the same units as the averaged 
quantity
(e.g. if you star from charge density in electrons/bohr^3 you get an averaged 
electron density in electrons/bohr^3),
being defined as (let us suppose that you average in the plane defined by a1 
and a2 vectors):

rho_avg(z) = ( 1 / S ) * integral( dx dy rho(x,y,z) )

That means that if you perform 
 integral( dz rho_avg(z) )
you get 
number of electrons / S

If you need number of electrons than just multiply by S with
S = cross_product( a1, a2 )
(in bohr^2)

Just try, better if you do it with the total charge density, to check if the 
integral returns you
the number of electrons.

I’m sorry but I cannot check directly if I remember correctly at the moment, but
this should work.

Giovanni

-- 

Giovanni Cantele, PhD

CNR-SPIN
c/o Dipartimento di Fisica
Universita' di Napoli "Federico II"
Complesso Universitario M. S. Angelo - Ed. 6
Via Cintia, I-80126, Napoli, Italy

e-mail: giovanni.cant...@spin.cnr.it
gcant...@gmail.com
Phone: +39 081 676910
Skype contact: giocan74
Web page: https://sites.google.com/view/giovanni-cantele

> On 26 Oct 2018, at 03:31, Dingfu Shao  wrote:
> 
> Dear QE developers and users:
> 
> I am wondering what should be the unit of the planar average data got from 
> the average.x
> 
>  I am calculating the planar average of charge density within a energy 
> window. What I did is firstly using pp to get the integrated local density of 
> states (ILDOS) of that energy window with plot_num=10, then using average.x 
> to get the planar average. 
> 
> In this case, what is the unit of the second column (say, rho(z)) of  the 
> output file? I thought since the DOS has a unit of states/eV, the integration 
> of DOS within a energy window should get some states or electrons. Then the 
> unit of rho(z) should be electron/bohr. But seems it is not. In my case the 
> energy window I concerned contains one electron, However, if I directly 
> integrate  rho(z), I can only get a very small value. If I assume the unit is 
> electron/(bohr^3), the integretion of  rho(z)*A is also smaller than one 
> (here A is the area of xy plane).  
> 
> Can you help me about it? Thank you very much!
> 
> Best,
> 
> Ding-Fu
> 
> 
> 
> Ding-Fu Shao, Ph. D.
> Department of Physics and Astronomy, University of Nebraska-Lincoln
> Lincoln, NE 68588-0299
> Email: dingfu.s...@gmail.com 
> <mailto:dingfu.s...@gmail.com>___
> users mailing list
> users@lists.quantum-espresso.org
> https://lists.quantum-espresso.org/mailman/listinfo/users

___
users mailing list
users@lists.quantum-espresso.org
https://lists.quantum-espresso.org/mailman/listinfo/users

Re: [QE-users] Adsorption energy studies

2018-07-25 Thread Giovanni Cantele
In you are dealing with asymmetric slabs, dipole correction might be important. 
Some adsorbates are just conceived to change the work function of a material, 
thus representing dipole layers added on the top of a surface. The induced 
dipole can make the vacuum levels in the two sides of the slab rather different 
Giovanni 

Inviato da iPhone

> Il giorno 25 lug 2018, alle ore 20:05, Paolo Costa  
> ha scritto:
> 
> Hi Giovanni,
> 
> thanks a lot for your quick reply. 
> I guess you are definetely right, I did not recognize such error. I will put 
> right way the calculation of the surface allowing the relax of all the atoms. 
> I will let you know the results.
> 
> By the way, do you think that the dipole correction is always necessary for 
> adsorption studies?
> 
> Thanks again.
> 
> Paolo
> 
> 2018-07-25 13:21 GMT-04:00 Giovanni Cantele :
>> I cannot visualise your structures at the moment, however one first issue, 
>> concerning the adsorption energy calculation is that it seems that in the 
>> system WITOUTH
>> the molecule you fixed some Ti/O layers, that are instead free to relax in 
>> the adsorbed system. As such, the run with the molecule is able to lower the 
>> total energy
>> also by allowing those atoms (a lot!), that were fixed in the other run, to 
>> change their positions. The result is that the -0.66 Ry contain also the 
>> energy gain of
>> the “free” surface when you let those atoms to relax towards a lower energy 
>> configuration. So, one first check would bee to see whether the atomic 
>> positions
>> of those atoms changed a lot (for example I see the z of one Ti atom equal 
>> to 15.819807000 when it is fixed, and 16.095206282 when it is allowed to 
>> relax, this is a change
>> of ~0.3 A -> 2% of the initial coordinate, that summed over all those atoms 
>> might make a difference!).
>> In any case I would try to make a surface (with no molecule) run where all 
>> atoms are allowed to relax and see how much energy is gained with respect to 
>> your run.
>> 
>> I could give you more hints tomorrow, if I’m able to visualise the structures
>> 
>> Giovanni
>> 
>> -- 
>> 
>> Giovanni Cantele, PhD
>> 
>> CNR-SPIN
>> c/o Dipartimento di Fisica
>> Universita' di Napoli "Federico II"
>> Complesso Universitario M. S. Angelo - Ed. 6
>> Via Cintia, I-80126, Napoli, Italy
>> 
>> e-mail: giovanni.cant...@spin.cnr.it
>> gcant...@gmail.com
>> Phone: +39 081 676910
>> Skype contact: giocan74
>> Web page: https://sites.google.com/view/giovanni-cantele
>> 
>>> On 25 Jul 2018, at 19:07, Paolo Costa  wrote:
>>> 
>>> Dear experts,
>>> 
>>> I got nonsense value of adsorption energy by computing 4-iodoanisole on top 
>>> of TiO2 anastase surface (101). 
>>> By calculating : (Esurface+anisole)-Eanisole-Esurface, I got an adsoprtion 
>>> energy of 0.66 ry (8.98 eV), which is too high. 
>>> Now I am trying to apply the dipole correction to see if this is the 
>>> problem causing an unusual adsorption energy. 
>>> 
>>> In the following link you can find the output files:
>>> https://www.dropbox.com/sh/edijwg1kyp7gx31/AACD7Up1XIIULbJFToLNqsUra?dl=0
>>> 
>>> I would be glad to receive any suggestions to understand the problem. 
>>> 
>>> Thanks.
>>> 
>>> Paolo
>>> ___
>>> users mailing list
>>> users@lists.quantum-espresso.org
>>> https://lists.quantum-espresso.org/mailman/listinfo/users
>> 
>> 
>> ___
>> users mailing list
>> users@lists.quantum-espresso.org
>> https://lists.quantum-espresso.org/mailman/listinfo/users
> 
> 
> 
> -- 
> Paolo Costa, Ph.D.
> Postdoctoral Researcher
> Department of Chemistry and Biomolecular Sciences
> University of Ottawa
> 10 Marie Curie, Ottawa, ON K1N 6N5, Canada
> Room number: DRO 326 (D'Iorio Hall)
> ___
> users mailing list
> users@lists.quantum-espresso.org
> https://lists.quantum-espresso.org/mailman/listinfo/users
___
users mailing list
users@lists.quantum-espresso.org
https://lists.quantum-espresso.org/mailman/listinfo/users

Re: [QE-users] Adsorption energy studies

2018-07-25 Thread Giovanni Cantele
I cannot visualise your structures at the moment, however one first issue, 
concerning the adsorption energy calculation is that it seems that in the 
system WITOUTH
the molecule you fixed some Ti/O layers, that are instead free to relax in the 
adsorbed system. As such, the run with the molecule is able to lower the total 
energy
also by allowing those atoms (a lot!), that were fixed in the other run, to 
change their positions. The result is that the -0.66 Ry contain also the energy 
gain of
the “free” surface when you let those atoms to relax towards a lower energy 
configuration. So, one first check would bee to see whether the atomic positions
of those atoms changed a lot (for example I see the z of one Ti atom equal to 
15.819807000 when it is fixed, and 16.095206282 when it is allowed to relax, 
this is a change
of ~0.3 A -> 2% of the initial coordinate, that summed over all those atoms 
might make a difference!).
In any case I would try to make a surface (with no molecule) run where all 
atoms are allowed to relax and see how much energy is gained with respect to 
your run.

I could give you more hints tomorrow, if I’m able to visualise the structures

Giovanni

-- 

Giovanni Cantele, PhD

CNR-SPIN
c/o Dipartimento di Fisica
Universita' di Napoli "Federico II"
Complesso Universitario M. S. Angelo - Ed. 6
Via Cintia, I-80126, Napoli, Italy

e-mail: giovanni.cant...@spin.cnr.it
gcant...@gmail.com
Phone: +39 081 676910
Skype contact: giocan74
Web page: https://sites.google.com/view/giovanni-cantele

> On 25 Jul 2018, at 19:07, Paolo Costa  wrote:
> 
> Dear experts,
> 
> I got nonsense value of adsorption energy by computing 4-iodoanisole on top 
> of TiO2 anastase surface (101). 
> By calculating : (Esurface+anisole)-Eanisole-Esurface, I got an adsoprtion 
> energy of 0.66 ry (8.98 eV), which is too high. 
> Now I am trying to apply the dipole correction to see if this is the problem 
> causing an unusual adsorption energy. 
> 
> In the following link you can find the output files:
> https://www.dropbox.com/sh/edijwg1kyp7gx31/AACD7Up1XIIULbJFToLNqsUra?dl=0 
> <https://www.dropbox.com/sh/edijwg1kyp7gx31/AACD7Up1XIIULbJFToLNqsUra?dl=0>
> 
> I would be glad to receive any suggestions to understand the problem. 
> 
> Thanks.
> 
> Paolo
> ___
> users mailing list
> users@lists.quantum-espresso.org
> https://lists.quantum-espresso.org/mailman/listinfo/users

___
users mailing list
users@lists.quantum-espresso.org
https://lists.quantum-espresso.org/mailman/listinfo/users

Re: [QE-users] Band Structure and DOS contradiction

2018-05-16 Thread Giovanni Cantele
Maybe you could send input files and a plot of DOS and band structure to help 
people to figure out
what is happening.

If DOS and band structure do not agree with each other, I would not trust to 
either!!!

What do you mean that you repeat the calculation with **different theory** 

Giovanni



> On 16 May 2018, at 13:17, B S Bhushan <ecebhus...@gmail.com> wrote:
> 
> Dear Experts,
> 
> I am analyzing defected graphene with different dopants.
> In very few cases... I end up having contradictory DOS and Band structure.
> 
> The Band structure shows a small band gap at the fermi level, however, the 
> DOS shows states on the fermi level.
> Same result appears even if the calculation is repeated with different theory.
> In such cases, Should I depend on the Band structure or the DOS.
> What should I call the system, metallic or semi-metallic or semi-conducting 
> ???
> 
> Please suggest.
> 
> 
> Sincerely,
> B S Bhushan,
> Ph.D Scholar,
> ABV- Indian Institute of Information Technology and Management, Gwalior.
> www.bsbhushan.com <http://www.bsbhushan.com/>
> 
> 
> ___
> users mailing list
> users@lists.quantum-espresso.org
> https://lists.quantum-espresso.org/mailman/listinfo/users

-- 

Giovanni Cantele, PhD
CNR-SPIN
c/o Dipartimento di Fisica
Universita' di Napoli "Federico II"
Complesso Universitario M. S. Angelo - Ed. 6
Via Cintia, I-80126, Napoli, Italy
e-mail: giovanni.cant...@spin.cnr.it
Phone: +39 081 676910
Skype contact: giocan74

ResearcherID: http://www.researcherid.com/rid/A-1951-2009
Web page: http://people.fisica.unina.it/~cantele

___
users mailing list
users@lists.quantum-espresso.org
https://lists.quantum-espresso.org/mailman/listinfo/users

Re: [QE-users] Reg: need of Antimony (Sb) Ultrasoft Pseudopotentials

2018-05-16 Thread Giovanni Cantele
Actually, US pseudo potential for Sb is downloadable at the Pseudopotentials 
page of Quantum-espresso
http://www.quantum-espresso.org/pseudopotentials/ps-library/sb 
<http://www.quantum-espresso.org/pseudopotentials/ps-library/sb>

The files you find are built using pslibrary:
https://dalcorso.github.io/pslibrary/ <https://dalcorso.github.io/pslibrary/>


> On 16 May 2018, at 09:54, Hanuma Kumar <hanuma...@gmail.com> wrote:
> 
> Dear Quantum Espresso users,
> 
> I have calculated band structure of Sb (Antimony) based intermetallics, and I 
> have calculated band structure of the alloys using NORMCONS Pseudopotentials. 
> The density of states and band structure is matches with the reported 
> results, but their magnetic moments are not matching. In order to get good 
> results, I am interested to do with Ultrasoft pseudopotentials. These type of 
> pseudopotentials is not available for Sb (Antimonty, Atomic number 51) and I 
> don't know how to generate pseudopotentials. If possible, can anyone please 
> send me the USPP files for Sb.
> 
> 
> Thanks and regards
> 
> Hanuma.
> ___
> users mailing list
> users@lists.quantum-espresso.org
> https://lists.quantum-espresso.org/mailman/listinfo/users

-- 

Giovanni Cantele, PhD
CNR-SPIN
c/o Dipartimento di Fisica
Universita' di Napoli "Federico II"
Complesso Universitario M. S. Angelo - Ed. 6
Via Cintia, I-80126, Napoli, Italy
e-mail: giovanni.cant...@spin.cnr.it
Phone: +39 081 676910
Skype contact: giocan74

ResearcherID: http://www.researcherid.com/rid/A-1951-2009
Web page: http://people.fisica.unina.it/~cantele

___
users mailing list
users@lists.quantum-espresso.org
https://lists.quantum-espresso.org/mailman/listinfo/users

Re: [QE-users] Which celldm(i) to choose for DOS

2018-05-16 Thread Giovanni Cantele
Dear Sudip,

I’m not sure I’ve fully understood your question. You are saying that, after a 
vc-relax calculation, you use the final output lattice parameter(s) in a single 
scf run,
and you obtain a different pressure, aren’t you?

As far as I remember, during vc-relax the code uses a fixed number of plane 
waves (because it cannot change the basis set during a single calculation).
On the other hand, in a single scf run the number of plane waves depends on the 
lattice parameter. So, it might happen that the basis set of the scf run is 
different from
that of the vc-relax run, because the latter is setup based on the initial 
cell, and not the last one that is unknown. So maybe you could compare the two 
basis sets, and then
try to make for example the scf run with the vc-relax basis set, in place of 
the default one, just to check that you obtain exactly the same results. In any 
case, I think that if this is 
the case the differences you experience in the lattice parameter should be 
negligible or small.

Giovanni

PS by the way, after writing this message I’ve find the FAQ addressing just you 
issue!
http://www.quantum-espresso.org/resources/faq/self-consistency#6.11

> On 16 May 2018, at 09:25, Sudip Kumar Mondal <sudipkm1...@gmail.com> wrote:
> 
> Hi all ,
>  
> I'm currently using QE 5.4.0. & I have a general question regarding the 
> calculation of density of states (LDOS & PDOS).
> 
> Before estimating any physical observable we optimize (relax/vc-relax or 
> sometimes both) the structure in hand to obtain accurate lattice parameter(s) 
> corresponding to zero kBar pressure. Optimizations w.r.t. all other 
> parameters (k-point grid , ecutwfc , ecutrho etc.)  are also performed.
> However for a single scf run with the lattice parameter(s) corresponding to 
> zero pressure as obtained from relax/vc-relax , yields some different 
> pressure other than zero kBar.  In my case then I ran a series of single scf 
> calculations for converging to zero pressure by varying the lattice 
> parameter. The scf convergence to zero pressure was successful. But the 
> lattice parameter  turned out be smaller than its zero pressure equivalent 
> obtained from relaxation by ~ 0.9 bohr.
> 
> Now, my question is whether I should proceed to calculate DOS with the value 
> of the lattice parameter(s) corresponding to zero pressure obtained from 
> relaxation or with the same obtained from the series of single scfs?
> 
> 
> Regards.
> -- 
> Sudip Kumar Mondal
> DST INSPIRE Fellow
> High Pressure & Temperature Laboratory
> Dept. Of Physics/Geological Sc.
> Jadavpur University.
> ___
> users mailing list
> users@lists.quantum-espresso.org
> https://lists.quantum-espresso.org/mailman/listinfo/users

-- 

Giovanni Cantele, PhD
CNR-SPIN
c/o Dipartimento di Fisica
Universita' di Napoli "Federico II"
Complesso Universitario M. S. Angelo - Ed. 6
Via Cintia, I-80126, Napoli, Italy
e-mail: giovanni.cant...@spin.cnr.it
Phone: +39 081 676910
Skype contact: giocan74

ResearcherID: http://www.researcherid.com/rid/A-1951-2009
Web page: http://people.fisica.unina.it/~cantele

___
users mailing list
users@lists.quantum-espresso.org
https://lists.quantum-espresso.org/mailman/listinfo/users

Re: [QE-users] possible little bug in post-processing + electric field ?

2018-04-18 Thread Giovanni Cantele
Hi, thanks for your response. However, I do not think this is the problem, 
because
you can add a dipole correction in the cases where, for example, you are 
dealing with an asymmetric
slab introducing a spurious dipole due to periodic boundary conditions. 
eamp is needed when you what to introduce a “physical” electric field, that is, 
run simulations in the presence
of the electric field, and this is not my case.

Thanks,

Giovanni

> On 18 Apr 2018, at 13:02, Manoar Hossain <manoar...@niser.ac.in> wrote:
> 
> Hi,
> 
> 
> Your eamp=0.0 that's why you are getting zero fields.
> 
> 
> 
> 
> 
> Thanks and Regards,
> 
> On Wed, Apr 18, 2018 at 4:17 PM, Giovanni Cantele 
> <giovanni.cant...@spin.cnr.it <mailto:giovanni.cant...@spin.cnr.it>> wrote:
> Dear all,
> 
> I’m using pp.x, 6.2 version, and trying to plot the sawtooth electric field 
> potential (if present) (plotnum=12), in the presence of
> dipole correction. However, after successfully accomplishing the pw.x run, I 
> find that that the sawtooth potential saved by pp.x is
> everywhere zero and the following message appears in the output of pp.x:
>  Message from routine punch_plot:
>  e_field is not calculated
> 
> However, if using 5.4.0 with the SAME input files, the output of pp.x says, 
> for example (in a sample run)
>  Calling punch_plot, plot_num =  12
> 
>  Adding external electric field
> 
>  Computed dipole along edir(3) : 
> Dipole   -0.0216 Ry au, -0.0550 Debye
> Dipole field -0. Ry au
> 
> Potential amp.0.0014 Ry
> Total length 17.9524 bohr
> 
> 
> I realise that since 5.4.0 the format of some (or maybe all) files within 
> outdir has changed, because for example
> in 5.4.0 I find data-file.xml that contains a line such as
>  
> T
> 
> that is present NOWHERE in the filed saved by pw.x 6.2. On the other hand, 
> other fields of xml files, such as field_direction
> or maximum_position are present in the outputs of both 6.2 and 5.4.0 even 
> though with slightly different formats.
> 
> 
> 
> Just to let someone try if I’m incorrectly using pp.x or if a possible bug 
> has been introduced from 5.4.0 to 6.2, this is a sample, quick
> run that reproduces the error:
> 
>  prova.scf.in <http://prova.scf.in/>
> 
> calculation  = 'scf'
> title= 'prova'
> restart_mode = 'from_scratch'
> outdir   = './tmp/'
> prefix   = 'prova'
> pseudo_dir   = './'
> tefield  = .true.
> dipfield = .true.
> /
> 
> ibrav= 1
> a= 10.0
> nat  = 1
> ntyp = 1
> ecutwfc  = 20
> occupations  = 'smearing'
> smearing = 'mv' 
> degauss  = 0.015
> edir = 3
> eamp = 0.0
> emaxpos  = 0.638889
> eopreg   = 0.05
> /
>  
> /
> ATOMIC_SPECIES
>  O15.9994   O.pbe-n-kjpaw_psl.1.0.0.UPF
> ATOMIC_POSITIONS { crystal }
> O 0.0   0.0   0.0
> K_POINTS { gamma }
> 
> 
> 
> 
>  prova.pp.in <http://prova.pp.in/>
> 
> prefix= 'prova'
> outdir= './tmp/'
> filplot   = 'prova.pp'
> plot_num  = 12
> /
> 
> 
> 
> Could you please give me an advice on whether I’m using in the wrong way pp.x 
> or there is an issue in
> 6.2 versus 5.4.0? If the latter applies, is the problem only related to pp.x 
> or it affects also the results of
> scf/relax runs?
> 
> Thanks in advance,
> 
> Giovanni
> 
> -- 
> 
> Giovanni Cantele, PhD
> CNR-SPIN
> c/o Dipartimento di Fisica
> Universita' di Napoli "Federico II"
> Complesso Universitario M. S. Angelo - Ed. 6
> Via Cintia, I-80126, Napoli, Italy
> e-mail: giovanni.cant...@spin.cnr.it <mailto:giovanni.cant...@spin.cnr.it>
> Phone: +39 081 676910
> Skype contact: giocan74
> 
> ResearcherID: http://www.researcherid.com/rid/A-1951-2009 
> <http://www.researcherid.com/rid/A-1951-2009>
> Web page: http://people.fisica.unina.it/~cantele 
> <http://people.fisica.unina.it/~cantele>
> 
> 
> ___
> users mailing list
> users@lists.quantum-espresso.org <mailto:users@lists.quantum-espresso.org>
> https://lists.quantum-espresso.org/mailman/listinfo/users 
> <https://lists.quantum-espresso.org/mailman/listinfo/users>
> 
> 
> 
> -- 
> Manoar Hossain
> Research Scholar
> School of Physical Sciences
> National Institute of Science Education and Research (NISER)
&

[QE-users] possible little bug in post-processing + electric field ?

2018-04-18 Thread Giovanni Cantele
Dear all,

I’m using pp.x, 6.2 version, and trying to plot the sawtooth electric field 
potential (if present) (plotnum=12), in the presence of
dipole correction. However, after successfully accomplishing the pw.x run, I 
find that that the sawtooth potential saved by pp.x is
everywhere zero and the following message appears in the output of pp.x:
 Message from routine punch_plot:
 e_field is not calculated

However, if using 5.4.0 with the SAME input files, the output of pp.x says, for 
example (in a sample run)
 Calling punch_plot, plot_num =  12

 Adding external electric field

 Computed dipole along edir(3) : 
Dipole   -0.0216 Ry au, -0.0550 Debye
Dipole field -0. Ry au

Potential amp.0.0014 Ry
Total length 17.9524 bohr


I realise that since 5.4.0 the format of some (or maybe all) files within 
outdir has changed, because for example
in 5.4.0 I find data-file.xml that contains a line such as
 
T

that is present NOWHERE in the filed saved by pw.x 6.2. On the other hand, 
other fields of xml files, such as field_direction
or maximum_position are present in the outputs of both 6.2 and 5.4.0 even 
though with slightly different formats.



Just to let someone try if I’m incorrectly using pp.x or if a possible bug has 
been introduced from 5.4.0 to 6.2, this is a sample, quick
run that reproduces the error:

 prova.scf.in

calculation  = 'scf'
title= 'prova'
restart_mode = 'from_scratch'
outdir   = './tmp/'
prefix   = 'prova'
pseudo_dir   = './'
tefield  = .true.
dipfield = .true.
/

ibrav= 1
a= 10.0
nat  = 1
ntyp = 1
ecutwfc  = 20
occupations  = 'smearing'
smearing = 'mv' 
degauss  = 0.015
edir = 3
eamp = 0.0
emaxpos  = 0.638889
eopreg   = 0.05
/
 
/
ATOMIC_SPECIES
 O15.9994 O.pbe-n-kjpaw_psl.1.0.0.UPF
ATOMIC_POSITIONS { crystal }
O 0.0   0.0   0.0
K_POINTS { gamma }




 prova.pp.in

prefix= 'prova'
outdir= './tmp/'
filplot   = 'prova.pp'
plot_num  = 12
/



Could you please give me an advice on whether I’m using in the wrong way pp.x 
or there is an issue in
6.2 versus 5.4.0? If the latter applies, is the problem only related to pp.x or 
it affects also the results of
scf/relax runs?

Thanks in advance,

Giovanni

-- 

Giovanni Cantele, PhD
CNR-SPIN
c/o Dipartimento di Fisica
Universita' di Napoli "Federico II"
Complesso Universitario M. S. Angelo - Ed. 6
Via Cintia, I-80126, Napoli, Italy
e-mail: giovanni.cant...@spin.cnr.it
Phone: +39 081 676910
Skype contact: giocan74

ResearcherID: http://www.researcherid.com/rid/A-1951-2009
Web page: http://people.fisica.unina.it/~cantele

___
users mailing list
users@lists.quantum-espresso.org
https://lists.quantum-espresso.org/mailman/listinfo/users

  1   2   3   4   5   >